IBPS Agriculture Officers Examination-2016 Held on 29-1-2017 Quantitative Aptitude Question Paper With Answer Key

IBPS Agriculture Officers Examination-2016 Held on 29-1-2017 Quantitative Aptitude
IBPS Agriculture Officers Examination-2016 Held on 29-1-2017 Quantitative Aptitude Question Paper With Answer Key

IBPS Agriculture Officers Examination-2016 Held on 29-1-2017

Quantitative Aptitude

Directions – (Q. 1-5) The question consists of a question and two statements I and II given below it. You have to decide whether the data provided in the statements are sufficient to answer the question. Read the both the statements and choose the appropriate option.

(A) The data in both the statements I and II together are necessary to answer the question.

(B) The data either in statement I alone or in statement II alone are sufficient to answer the question.

(C) The data in both the statements I and II together are not sufficient to answer the question.

(D) The data in statement II alone are sufficient to answer the question, while the data in statement I alone are not sufficient to answer the question.

(E) The data in statement I alone are sufficient to answer the question, while the data in statement II alone are not sufficient to answer the question.

1. What is the cost price of the article ? (in Rs)

I. 8% of the selling price of the article is equal to the 12% of the cost price.

II. 15% of the cost price of the article is Rs 6 more than 9% of the selling price of the article.

Answer: (A)

2. In how many days A, B and C together can finish a piece of work?

I. A is twice as efficient as B.

II. A and B together can finish the piece of work in  days. B and C together can finish the piece of work in  17 A and C together can finish the same piece of work in   

Answer: (D)

3. What distance (in km) the boat can travel downstream in 20 minutes?

I. The distance travelled by the boat upstream in 48 minutes is equal to the distance travelled by the boat downstream in 36 minutes.

II. The respective ratio between speed of the boat in still water and speed of the water currents is 7 : 1.

Answer: (C)

4. What is M’s mother’s present age ? (in years)

I. The respective ratio between M’s father present age and M’s present age is 7 : 2.

II. The combined present age of M’s father and M is 6 years more than the combined present age of M’s mother and M. 12 years ago M’s father age that time was  M’s mother’s age that time.

Answer: (D)

5. Side of a square is equal to the length of a rectangle. What is the area of the square ? (in m2)

I. The difference between area of the rectangle and that of the square is 196 m2.

II. The difference between perimeter of the square and that of the rectangle is 14 m.

Answer: (A)

Directions – (Q. 6-11) Study the table and answer the given question.

   The given table shows the attendance at a stadium (having a seating capacity of 40,000) during all the days of a 5-day test match.

Note- Everyone left the stadium at the end of play. End of play occurs at the end of every day.

6. What was the average number of people who left the stadium before the end of play on Thursday, Friday and Sunday?

(A)  3700

(B)  3900

(C)  3300

(D)  3100

(E)  3500

Answer: (E)

7. What is respective ratio between the total number of people who left the stadium before the end of play on Friday and Saturday together and that who left the stadium before the end of play on Thursday and Monday together?

(A)  3 : 5

(B)  4 : 7

(C)  5 : 7

(D)  5 : 8

(E)  2 : 3

Answer: (C)

8. Out of the number of people who left the stadium before the end of play on Monday, the respective ratio between the number of children, adults and senior citizens was 4 : 3 : 1. What was the number of adults who left the stadium before the end of play on Monday?

(A)  840

(B)  750

(C)  1200

(D)  900

(E)  1020

Answer: (B)

9. What per cent the total seats of the stadium was sold for Saturday, if 1,080 people did not turn up for that day’s play ?

(A)  81.2

(B)  78.7

(C)  82.7

(D)  94.8

(E)  83.2

Answer: (C)

10. If males comprised 65% and 60% of those who attended the stadium on Friday and Sunday respectively, what was the difference between the number of males who attended the stadium on Friday and Sunday?

(A)  1800

(B)  3400

(C)  3200

(D)  5100

(E)  2400

Answer: (E)

11. The number of people who stayed back till the end of p lay on Friday was what per cent more than that on Thursday?

(A)  50

(B)  60

(C)  40

(D)  25

(E)  30

Answer: (A)

Directions – (Q. 12-16) What will come in place of question-mark (?) in the given number series ?

12. 5   388    387    383    359    ?

(A)  167

(B)  154

(C)  181

(D)  199

(E)  173

Answer: (A)

13. 350   410    466    513    544    ?

(A)  585

(B)  560

(C)  572

(D)  595

(E)  550

Answer: (E)

14. 2   3        8        27      ?        565

(A)  174

(B)  132

(C)  166

(D)  148

(E)  112

Answer: (E)

15. 23   11      10      5   25      ?

(A)  90

(B)  45

(C)  50

(D)  75

(E)  60

Answer: (E)

16. 86   86      79      105    42      ?

(A)  112

(B)  166

(C)  188

(D)  148

(E)  128

Answer: (B)

17. Ram and Ali were travelling on their motorbikes from point P to Q. Ram, travelling at a speed of 60 km/hr leaves from point P at 6 am. Ali travelling at a speed of 90 km/hr leaves point P at 10 am on the same day as Ram. At what distance from point P, will Ram and Ali meet?

(A)  400  km

(B)  720 km

(C)  420 km

(D)  640 km

(E)  480 km

Answer: (B)

18. A started a business with an investment of Rs 12,000. At the end of six months from the start of the business, A withdrew half of his initial investment and B and C invested in the ratio of 10 : 9 respectively. If A’s share in annual profit of Rs 14,000 was Rs 4,500, what was the investment made by B?

(A)  Rs 20,000

(B)  Rs 25,000

(C)  Rs 10,000

(D)  Rs 12,000

(E)  Rs 12,500

Answer: (A)

Directions – (Q. 19-24) What approximate value will come in place of question mark in the given question ?

(You are not expected to calculate the exact value).

19. 85 + 12.91 × 2.04 = 112.98 × ?

(A)  2

(B)  9

(C)  4

(D)  5

(E)  3

Answer: (D)

20. (419.89 + 59.99) = ? ×899

(A)  18

(B)  12

(C)  15

(D)  6

(E)  8

Answer: (B)

21. 

(A)  9

(B)  11

(C)  13

(D)  7

(E)  15

Answer: (B)

22. ? % of (144.98 ÷06) + 27.01 = 50.04.

(A)  80

(B)  30

(C)  65

(D)  90

(E)  75

Answer: (C)

23. √145 + ?2 ×98 = 44

(A)  2

(B)  1

(C)  8

(D)  4

(E)  6

Answer: (D)

24. 

(A)  1

(B)  6

(C)  3

(D)  2

(E)  4

Answer: (C)

Directions – (Q. 25-30) – Refer to the pie charts ad answer the given question.

25. Combining stores D and E, only 40% of bags (of domestic and international brands) sold was of brand ‘X’. Among those, only 50% was leather bags. What is the total number of leather bags of brand ‘X’ sold by stores D and E together?

(A)  249

(B)  236

(C)  247

(D)  243

(E)  239

Answer: (D)

26. Approximately, what per cent of bags sold by store D was of domestic brands?

(A)  40

(B)  52

(C)  58

(D)  45

(E)  35

Answer: (D)

27. Number of bags of international brands sold by stores B and E increased by 20% and 25% respectively from 2010 to 2011. In 2011, if the respective ratio between total number of bags of international brands sold by stores B and E together and that of domestic brands sold by same stores together was 13 : 21 what was the total number of bags of domestic brands sold by stores B and E together in 2011?

(A)  588

(B)  609

(C)  567

(D)  667

(E)  547

Answer: (C)

28. What is the difference between total number of bags of domestic brands sold by stores A and B together and that sold by stores D and E together?

(A)  275

(B)  279

(C)  283

(D)  287

(E)  297

Answer: (B)

29. What is the average number of bags (of domestic and international brands) sold by stores A, B and D?

(A)  528

(B)  522

(C)  518

(D)  520

(E)  532

Answer: (B)

30. What is the central angle corresponding to number of bags of international brands sold by store C? (in degrees)

(A)  78.2

(B)  86.4

(C)  93.2

(D)  84.8

(E)  85.4

Answer: (B)

31. Ria invests Rs P in scheme A which offers simple interest @ 12% p.a. and Rs P + 2,520 in scheme B which offers compound interest (compounded annually) @ 20% p.a. The respective ratio between the amount received from scheme A at the end of 5 years and that received from scheme B at the end of 2 years was 4 : 5. What is the value of P?

(A)  6,720

(B)  6,500

(C)  6,480

(D)  8,000

(E)  6,000

Answer: (C)

32. Cost price of article B is 20% more than that of article A. Article B was marked 20% above its cost price. The difference (in Rs)between the cost price and the marked price is equal that of article A. Both the articles were sold at a discount of 10%. If the selling price of article B is Rs 45 more than that of article A, what is the cost price of article A?

(A)  Rs 250

(B)  Rs 300

(C)  Rs 280

(D)  Rs 225

(E)  Rs 240

Answer: (A)

Directions- (Q. 33-38) Study the following information carefully to answer the question.

   A bakery sells three baked products-Macaroons, cupcakes and brownies. On January 1, 2017, the total number of customers who bought products from the bakery was ‘X’. The respective ratio between male customers and female customers was 4 : 9.

   Out of the total number of male customers, 1/5th bought only macaroons 3/6th bought only cupcakes and 30% bought only brownies. The number of customers who bought only macaroons and cupcakes both was equal to the number of customers who bought only cupcakes and brownies both. The number of customers who bought only macaroons and brownies was 6 less than the number of customers who bought only macaroons and cupcakes both. 8 customers bought all the three baked products and 48 customers bought only brownies.

   Out of the total number of female customers, 1/8th bought only macaroons, 3/8th bought only brownies, 10% bought only macaroons and cupcakes and 5% bought only cupcakes and brownies. The number of customers who bought only cupcake was twice the number of customers who bought only macaroons. 16 customers bought only macaroons and brownies and the remaining bought all three baked products.

33. What is t he respective ratio between the total number of male customers who purchased both cupcakes and brownies and the total number of female customers who purchased the same?

(A)  15 : 17

(B)  14 : 23

(C)  15 : 19

(D)  12 : 19

(E)  12 : 17

Answer: (D)

34. The total number of customers (both male and female) who bought only brownies was what per cent more than that who bought only cupcakes?

(A)  55.5

(B)  52.5

(C)  54

(D)  58

(E)  51.5

Answer: (B)

35. Out of the male customers who bought macaroons, 1/3rd bought 4 macaroons each and remaining bought 2 macaroons each. How many total macaroons were sold to male customers?

(A)  176

(B)  181

(C)  153

(D)  167

(E)  168

Answer: (A)

36. What is the difference between the number of males and females customers who bought only cupcakes?

(A)  30

(B)  50

(C)  60

(D)  40

(E)  70

Answer: (C)

37. On January 2, 2017 the total number of customers who purchased from the bakery decreased by 10% as compared to the previous day. If the respective ratio between male customers and female customers was 4 : 5, what was the number of male customers?

(A)  236

(B)  232

(C)  216

(D)  208

(E)  220

Answer: (D)

38. What is the difference between the total number of customers (both male and female) who bought all the three baked products and that who bought only two of the baked products?

(A)  84

(B)  74

(C)  53

(D)  58

(E)  48

Answer: (A)

39. A jar had 120 litre mixture of milk and water in the respective ratio of 5 : 1. 30 litres of this mixture is taken out and ‘X’ litre of each milk and water is added to the jar. The respective ratio between milk and water in the jar was 4 : 1 respectively, what was the total quantity of both milk and water added to jar ? (in litres)

(A)  12

(B)  10

(C)  16

(D)  20

(E)  18

Answer: (B)

Directions – (Q. 40-45) In the following questions two equations numbered I and II are given you have to solve both the equations and give answer, if-

(A) x > y       (B) x ≥ y

(C) x < y       (D) x ≤ y

(E) x = y or relationship cannot be established.

40. I. 4x2 + 12x + 5 = 0

II. y2 + 7y + 12 = 0

Answer: (A)

41. I. 3x2 – 10x + 8 = 0

II. 3y2 – 9y + 6 = 0

Answer: (E)

42. I. 2x2 + 3x + 1 = 0

II. 2y2 + 9y + 9 = 0

Answer: (A)

43. I. x2 – 8x + 15 = 0

II. y2 – 11y + 28 = 0

Answer: (E)

44. I. 2x2 + 13x + 21 = 0

II. 2y2 + 9y + 10 = 0

Answer: (C)

45. I. x2 = 64

II. y2 – 15y + 56 = 0

Answer: (E)

46. The present age of A’s father is equal to six times of A’s age two years hence. If A’s mother is eight years younger than A’s father and the respective ratio between present age of A’s mother and father is 7 : 9, what is A’s present age ? (in years)

(A)  12

(B)  6

(C)  5

(D)  4

(E)  8

Answer: (D)

47. If the circumference of a circle is 36 cm more than the perimeter of a square and the radius of t he circle is 3 cm less than the side of the square. What is the measure of the area of square? (in cm2)

(A)  567

(B)  676

(C)  476

(D)  576

(E)  675

Answer: (D)

48. A project can be completed by 20 men and 36 women together in  28 men can complete the same project in 15 days. How much work will be left unfinished if 40 women together work for 7 days?

(A)  1/3

(B)  1/5

(C)  2/5

(D)  2/3

(E) 

Answer: (E)

49. In village A and B, out of their respective population (male + female), 2/5th and 3/8th are females. The numbers of males in village B are 25% more than those in village A. The number of females in village A and B together is 2125, what is the total population (male + female) of village A?

(A)  3600

(B)  2500

(C)  3000

(D)  2000

(E)  3500

Answer: (B)

50. In the month of July, Ramu spent Rs 3,225 on paying electricity bill, Rs. 1,290 on paying water bill, Rs 5,160 on paying telephone bill and Rs 645 on paying cable bill. After paying these bills he is left with 84% of his monthly salary. Out of the remaining salary he spends 20% on buying groceries and the remaining he invests in mutual funds and VPF in the respective ratio of 5 : 3. How much does he invests in mutual funds every month?

(A)  Rs 29,400

(B)  Rs 28,500

(C)  Rs 30,650

(D)  Rs 26,550

(E)  Rs 27,090

Answer: (E)

IBPS PO-MT Preliminary Examination Held on October 22, 2016 Question Paper With Answer Key

IBPS PO-MT Preliminary Examination Held on October 22, 2016
IBPS PO-MT Preliminary Examination Held on October 22, 2016 Question Paper With Answer Key

IBPS PO/MT Preliminary Examination Held on October 22, 2016

Part A

English Language

 

Directions (Q. Nos. 1-10) Read the sentences to find out whether there is any grammatical error in it. The error, if any, will be in one part of the sentence. Mark that part with the error as your answer. If there is not error, mark ‘No error’ as your answer. (Ignore the errors of punctuation, if any)

1. A lot of research has been/conducted on the field of human/ resources for understanding what creates/ work culture in an organization.

(a)  A lot of research has been

(b)  conducted on the field of human

(c)  resources for understanding what creates

(d)  work culture in an organization

(e)  No error

Answer: (b)

2. During our visit to the hill station, we/came across signboards, which read that/ the area where we was/ under observation by the neighbouring country.

(a)  during our visit to the hill station, we

(b)  came across signboards which read that

(c)  the area where we was

(d)  under observation by the neighbouring country

(e)  No error

Answer: (c)

3. No matter what people opine about/the stern measures taken against/ traffic signal violators, taking such an/action have been pending since long.

(a)  No matter what people opine about

(b)  the stern measures taken against

(c)  traffic signal violators, taking such an

(d)  action have been pending since long

(e)  No error

Answer: (d)

4. Though these buildings have been given/clearance by first safety officials, any/layman can understand that hardly/ any fire safety norms have followed

(a)  Though these buildings have been given

(b)  clearance by fire safety officials, any

(c)  layman can understand that hardly

(d)  any fire safety norms have followed

(e)  No error

Answer: (d)

5. Hardly he had entered the building/when the security guard called and/ informed him that he had left his/ car door opening the parking lot.

(a)  Hardly he had entered the building

(b)  when the security guard called and

(c)  informed him that he had left his

(d)  car door open in the parking lot

(e)  No error

Answer: (a)

6. The new variety of genetically modified/ crops is being extremely successful in /curbing the usage of/ pesticides and increasing the per unit output.

(a)  The new variety of genetically modified

(b)  crops is being extremely successful in

(c)  curbing the usage of

(d)  pesticides and increasing the per unit output

(e)  No error

Answer: (e)

7. Air pollution in the city rises/ beyond the permissible limits every winter/ as the pollutants cannot escape from the/ atmosphere due to radial inversion.

(a)  Air pollution in the city rises

(b)  beyond the permissible limits every winter

(c)  as the pollutants cannot escape from the

(d)  atmosphere due to radial inversion

(e)  No error

Answer: (b)

8. Globally, the Indian market is the second/ largest user of mobile phones, with more than/ a billion people using mobile/ phones for calling and internet purposes.

(a)  Globally, the Indian market is the second

(b)  largest user of mobile phones, with more than

(c)  a billion people using mobile

(d)  phones for calling and internet purposes

(e)  No error

Answer: (e)

9. After having working for five/ years in a private firm, Karan/ got down to preparing for/ various bank entrance examinations.

(a)  After having working for five

(b)  years in a private firm, Karan

(c)  got down to preparing for

(d)  various bank entrance examinations

(e)  No error

Answer: (a)

10. Those who want to do good are/ neither selfish nor in a hurry because/ they know what it requires a long/ time to impregnate people with good.

(a)  Those who want to do good are neither

(b)  selfish nor in a hurry because

(c)  they know what it requires a long

(d)  time to impregnate people with good

(e)  No error

Answer: (c)

Directions (Q. Nos. 11-15)  Rearrange the given six sentences A, B, C, D, E and F in a proper sequence, so as to form a meaningful paragraph and then answer the given questions.

A. Moreover, the number of licence-holders has risen even faster, one in five Chinese now has a licence.

B. Apart from the fact that the country’s population is, so large, most of these accidents have to do with the fact that China is so new to the business of driving cars.

C. Accidents are a common sight on the roads of China for many reasons.

D. In 2015, it added more cars to its roads than were driving in the whole country in 1999.

E. Economic rise has played a large part in all these developments.

F. In the rich world, where this economic rise has already taken place, the number of licence-holder is flat or falling.

11. Which of the following should be the fifth sentence after the rearrangement?

(a)  E

(b)  D

(c)  A

(d)  F

(e)  C

Answer: ()

12. Which of the following should be the sixth sentence after the rearrangement?

(a)  E

(b)  D

(c)  A

(d)  B

(e)  F

Answer: ()

13. Which of the following should be the first sentence after the rearrangement?

(a)  A

(b)  C

(c)  B

(d)  F

(e)  E

Answer: (b)

14. Which of the following should be the second sentence after the rearrangement?

(a)  A

(b)  B

(c)  C

(d)  F

(e)  D

Answer: (a)

15. Which of the following should be the fourth sentence after the rearrangement?

(a)  A

(b)  B

(c)  C

(d)  F

(e)  D

Answer: ()

Directions (Q. Nos. 16-22) Read the following passage and answer the given questions.

   After the Second World War, the leaders of the Western world tried to build institutions to prevent the conflicts of the preceding decades from recurring. They wanted to foster both prosperity and inter dependence, to ‘make war not only unthinkable, but materially impossible’. Their work bore fruit, Expanded global trade was raised incomes around the world. While globalization is sometimes portrayed as a corporate plot against the workers; that was not how it was seen before 1914. British Trade Unions were in favour of free trade, which kept down food prices for their members and also opened up markets for the factories in which they worked. Yet, as the Brexit vote demonstrates globalization now seems to be receding. Most economists have been blindsided by the backslash. Free trade can be a hard sell politically. The political economy of trade is treacherous. Its benefits, though substantial, are dilute, but its costs are often concentrated. This gives those affected a strong incentive to pus h for protectionism. Globalization itself thus seems to create forces that erode political support for integration.

   Deeper economic integration required harmonization of laws and regulations across countries. Differences in rules on employment contracts or product safety requirements, for instance, act as barriers to trade. Trade agreements like the Trans-Pacific Partnership focus more on ‘non-tariff barriers than they do on tariff reduction. The net impact of this is likely to be that some individuals, consumers and businesses are not likely to be as benefitted as others and given rise to discontent. Thus the consequences of such trade agreements often run counter to popular preferences. Joseph Stiglitz, a Nobel Prize winner, has warned that companies influence over trade rules harms workers and erodes support for trade liberalization. Clumsy government efforts to compensate workers hurt by globalization contributed to the global financial crisis, by facilitating excessive household borrowing, among the other things. Researchers have also documented how the cost of America’s growing trade with China has fallen disproportionately on certain. American cities.

Such costs perpetuate a cycle of globalization. Periods of global integration and technological progress generate rising inequality, which inevitably triggers two countervailing forces, one beneficial and one harmful. On the one hand, governments tend to respond to rising inequality by increasing redistribution and investing in education, on the other, inequality leads to political unheaval and war. The first great era of globalization, which ended in 1914, gave way to a long period of declining inequality, in which harmful forces played a bigger rise than beneficial ones. History might repeat itself, he warns. Such warnings do not amount to arguments against globalization. As many economists are quick to note, the benefits of openness are massive. It is increasingly clear, however, that supporters of economic integration underestimated the risks both that big slices of society would feel left behind and that nationalism would continue to provide an alluring alternative. Either error alone might have undercut support for globalization and the relative peace and prosperity, it has brought in combination, they threaten to reverse it.

16. What can be concluded from the example of Britain cited in the passage?

(a)  Countries which previously supported globalization no longer do.

(b)  Trade unions are losing their influence.

(c)  Agriculture has suffered in most developed countries.

(d)  Britain has not recovered from the financial crisis.

(e)  Technological progress boosts economic growth tremendously.

Answer: (a)

17. Which of the following has/have been outcome(s) of global integration?

I. Laws have become fairer for all.

II. Trade unions have become more peaceful.

III. Trade has grown substantially.

(a)  Only III

(b)  Only I

(c)  I and II

(d)  I and III

(e)  All I, II and III

Answer: (a)

18. Which of the following is the author’s view of Trans-Pacific Partnership?

(a)  It is likely to face opposition.

(b)  It will provide beneficial to all workers.

(c)  It will reduce tariffs effectively.

(d)  Trade with China will suffer.

(e)  None of the given options

Answer: (a)

19. Which of the following is true in the context of the passage?

(a)  The first era of globalization resulted in a decline in inequality.

(b)  Governments are  making efforts to help workers hurt by globalization.

(c)  Standardizing policy regulations will boost economic integration.

(d)  Technology has exacerbated the ill-effects of globalization.

(e)  All of the given options are true in the context of the passage.

Answer: (a)

20. Which of the following best explains the phrase ‘Such warnings do not amount to arguments against globalization’ in the context of the passage?

(a)  Most economist are unnecessarily alarmist about globalization.

(b)  Globalization is beneficial to all.

(c)  Do not do away with globalization, but take concerns about globalization seriously.

(d)  Politicians warn against globalization during elections, but actually support it.

(e)  We cannot reverse globalization but we must stall it.

Answer: (c)

21. Which of the following can be said about America’s trade with China?

(a)  America’s discontent against globalization has fallen.

(b)  Worker’s wages have rise tenfold.

(c)  America has been badly hit by the slowdown in china.

(d)  It has been especially harmful for certain American cities.

(e)  None of the given options can be said.

Answer: ()

22. Which of the following is the central idea of the passage?

(a)  Protectionism is the only way for developed countries to retain stability.

(b)  Globalization is receding and its decline should be speeded up.

(c)  While politicians are in favour of globalization, economists are not.

(d)  While developed countries are on the decline emerging ones are rising.

(e)  The backlash against globalization is serious and must be handled carefully.

Answer: (e)

Directions (Q. Nos. 23-30) In the given passage, there are blanks, each of which has numbered. These numbers are printed below the passage and against each five words are suggested, one of which fits the blank appropriately. Find out the appropriate word in each case.

The sue of technology in education has been present throughout history. Over the last century, schools have modified their (23) to teaching as well as methods that are used to enhance student learning. Chalk and slate were at one time the newest technology. From there, technological changes have gone from film, radio and television to desk-top computers and now into interactive white boards like SMART Technology. The capabilities teachers have with new technology give them the (24) to differentiate lessons for (25) overall learning. Microsoft PowerPoint is one of the most popular technology tools used in any classroom. SMART technologies has integrated the SMART Board Software with PowerPoint, thus combining the newest technology with the most popular S1. This brief description shows that new technology is  being implemented in the classroom. Unfortunately, due to (26) costs, the more advanced the new equipment becomes; the less likely schools are willing to (27) it for their classrooms. (28) sufficient funds, it is difficult for schools to obtain technologically advanced classrooms. SMART Technology  is the most recent equipment to enter the classroom. In 2002, SMART Boards ranged from $ 999 to $ 1999 and that was for just the board. If a school wanted to better (29), its finance and purchase the rolling floor stand accessory, which makes the technology more accessible to all teachers. It would pay an additional amount of somewhere $ 425 and $ 499. Any school that desires new technology must have the capacity to (30) it. However, even with sufficient funds, a technological integration effort is only as strong as the administrative support behind it.

23.

(a)  pathway

(b)  departure

(c)  approach

(d)  pedagogy

(e)  syllabus

Answer: (c)

24.

(a)  faculty

(b)  lesson

(c)  limitation

(d)  abundance

(e)  ability

Answer: (e)

25.

(a)  bigger

(b)  better

(c)  inferior

(d)  benefit

(e)  alleviated

Answer: (b)

26.

(a)  much

(b)  overhaul

(c)  high

(d)  hover

(e)  humble

Answer: (c)

27.

(a)  achieve

(b)  acquire

(c)  archive

(d)  excess

(e)  disperse

Answer: (b)

28.

(a)  Without

(b)  Following

(c)  Despite

(d)  Bereft

(e)  Unless

Answer: (a)

29.

(a)  allotment

(b)  proportion

(c)  allocate

(d)  conform

(e)  pro rata

Answer: (c)

30.

(a)  earmark

(b)  patent

(c)  publish

(d)  sharp

(e)  purchase

Answer: (e)

PART B Reasoning

Directions (Q. Nos. 31-35) In these questions, relationship between different elements is shown in the statement. The statements are followed by two conclusions. Study the conclusions based on the given statement and select the appropriate answer.

Give answer

a. if only conclusion I follows

b. if only conclusion II follows

c. if either conclusion I or II follows

d. if neither conclusion I nor II follows

e. if both conclusions follow

31. Statements L < A = M < P;

A = C < T;

M ≥ O > R

Conclusions I. O < P

II. C > L

Answer: (e)

32. Statements D ≥ U = S ≥ T;

O = P ≤ S

Conclusions I. D > P

II. P = D

Answer: (c)

33. Statements L < A = M < P;

A ≤ C < T;

M ≥ O > R

Conclusions I. T < M       II. A > R

Answer: (b)

34. Statements T > I ≥ L > E;

I ≤ N < B;

N ≥ S > D

Conclusions I. N ≤ E       II. T < B

Answer: (a)

35. Statements T > I = L > E;

I = N < B;

N = S > D

Conclusions I. S = I                   II. L > D

Answer: (d)

Directions (Q. Nos. 36-41) Study the information carefully and answer the questions based on it.

Seven athletes M, N, O, P, Q, R and S line on seven different floors of a building but not necessarily in the same order. The lower ment floor of the building is numbered two and so on till the top most floor is numbered seven.

Each one of them runs for a different distance in a Marathon 750m, 1200 m, 2200 m, 2900 m, 3600 m, 4300 m and 5000 m, but not necessarily in same order.

The one who runs for 4300 m lives on floor number 4. only one person lines between M and the one who runs for 4300 m. Only two people live between M and S.

The one who runs for 1200 m lives on an even numbered floor above Q. Only two people lives between the one who runs for 1200 m and one who runs for 2900 m. N live on one of the floors above M.

N run for 2100 m more than one who lives on floor number 3. The number of people living between the one who runs for 5000 m and S is same as the number of people living between M and R. Only one person lives between R and O. The one who runs for shortest distance lives immediately below P. Only one person lives between P and the one who runs for 3600 m.

36. Which of the following lives between O and the one who runs for 2900 m?

(a)  S and R

(b)  R and P

(c)  P and Q

(d)  Q and S

(e)  None of the above

Answer: (a)

37. As per the given arrangement four of the following five are alike in a certain way and so form a group. Which one of the following does not below to the group?

(a)  M-4300 m

(b)  Floor number 7-S

(c)  Floor number 4-N

(d)  P-3600 m

(e)  Floor number 5-5000 m

Answer: (c)

38. How many people live between S and O?

(a)  Five

(b)  One

(c)  None

(d)  Four

(e)  Three

Answer: (c)

39. Who amongst the following runs for 2900 m?

(a)  S

(b)  M

(c)  N

(d)  P

(e)  Q

Answer: (d)

40. If the total distance covered by B and M is 4800 m, then how much did B runs alone?

(a)  4050 m

(b)  1900 m

(c)  2600 m

(d)  1200 m

(e)  3600 m

Answer: (a)

41. Which of the following statements is true with respect to the given arrangement?

(a)  None of the option is true.

(b)  Only two people line between P and R

(c)  M lives on floor number 2.

(d)  The one who runs 750 m lives immediately above S

(e)  Q runs for 750 m

Answer: (c)

Directions (Q. Nos. 42-44) Study the given information carefully to answer the given question

Point M is 15 m to the East of Point L. Point C is 3 m to the North of Point M. Point Q is 6 m to the East of Point C. Point P is 3 m to the South of Point Q. Point V is to the North of Point L. A person walks 9 m from Point V towards South, reaches Point R, takes a left turn and reaches Point C.

42. In which direction is Point V with respect to Point P?

(a)  South-West

(b)  West

(c)  South-East

(d)  North-West

(e)  North-East

Answer: (d)

43. If a person walks 15 m towards east from Point V, takes a right turn and walks 4m, for will he be from Point M?

(a)  5 m

(b)  9 m

(c)  12 m

(d)  8 m

(e)  6 m

Answer: (b)

44. What is the difference of the distance between Points V, L and Point R, Q?

(a)  14 m

(b)  9 m

(c)  8 m

(d)  3 m

(e)  7 m

Answer: (b)

Directions (Q. Nos. 45-49) Study the given information carefully and answer the questions based on it.

Eight different people viz. C, D, E, F, W, X, Y and Z are sitting around a circular table facing the centre but not necessarily in the same order. Each one of them is wearing a watch of a different brand viz. Titan, Rado, Casio, Tissot, Rolex, Swatch, Omega Casio, Tissot, Rolex, Swatch, Omega and Longines but not necessarily in the same order.

Only two people sit between the one wearing Radio and X. The one wearing Tissot sits second to the left of X. Only three people sit between the one wearing R ado and W. The one wearing Casio sits second to the right of the one wearing Swatch. Neither X nor W is wearing Swatch. The one wearing Swatch is not an immediate neighbour of the one wearing Tissot. Z is not wearing Tissot. The one wearing Titan sits to the immediate right of Z. C is an immediate neighbour of one wearing Titan. Only three people sit between F and the one wearing Omega. Neither F nor E is wearing Rado. Only one person sits between the ones wearing Omega and Rolex.

45. Who amongst the following sits to the immediate left of the one wearing Rado?

(a)  C

(b)  The one wearing Omega

(c)  D

(d)  The one wearing Swatch

(e)  Z

Answer: (d)

46. Who amongst the following is wearing Longines?

(a)  D

(b)  Y

(c)  X

(d)  E

(e)  Z

Answer: (c)

47. Four of the following five are alike in a certain way based on the given arrangement and thus form a group. Which is the one that does not belong to that group?

(a)  Z-Rado

(b)  E – Longines

(c)  X – Rolex

(d)  W – Swatch

(e)  C-Titan

Answer: (d)

48. Which of the following represents the brand of watch worn by E?

(a)  Casio

(b)  Omega

(c)  Longines

(d)  Rolex

(e)  Swatch

Answer: (b)

49. Who amongst he following sit exactly between X and the one wearing Rado when counted from the right of X?

(a)  The ones wearing Tissot and Titan

(b)  Z and the one wearing Longines

(c)  D and F

(d)  E and Z

(e)  E and the one wearing Swatch

Answer: (e)

Directions (Q. Nos. 50-54) Study the following information to answer the given question.

Eight friends – P, Q, R, S, T, U, V and W are seated in a straight line with equal distance between each other, but not necessarily in the same order. Some of them are facing North while some are facing South. V is an  immediate neighbour of the person sitting at an extreme end of the line. R sits second to the left of V. Only one person sits between R and T. As many people sits to the right of T as to the left of P. V and P face the same direction (i.e. if V faces North then P also faces North and vice-versa.)

Immediate neighbours of P face opposite directions (i.e., if one neighbour faces North then the other faces South and vice-versa.) Q sits fourth to the left of S. Q is not an immediate neighbour of V. Persons sitting at the extreme ends face opposite directions (i.e., if one persons faces North then the other person faces South and vice-versa.)

W faces South. W does not sit at an extreme end of the line. U sits to the immediate right of W. U and Q face the same direction (i.e. if U faces North then Q also faces North and vice-versa.)

50. As per the given arrangement, which of the following statements is not true with respect to U?

(a)  U sits at an extreme end of the line

(b)  Only three  persons sit between U and R

(c)  U sits second to the left of T

(d)  All the given statements are true

(e)  U is an immediate neighbour of Q.

Answer: (d)

51. What is the position of Q with respect to W?

(a)  Immediate left

(b)  Second to the right

(c)  Third to the left

(d)  Third to the right

(e)  Second to the left

Answer: (e)

52. Four of the following five are alike in a certain way based on the given arrangement and hence form a group. Which of them does not belong to that group?

(a)  WS

(b)  QT

(c)  WR

(d)  UP

(e)  RV

Answer: (c)

53. How many persons sit to the left of T?

(a)  Three

(b)  None

(c)  More than three

(d)  One

(e)  Two

Answer: (e)

54. Which of the following represents the immediate neighbours of P?

(a)  R, T

(b)  S, V

(c)  W, P

(d)  T, S

(e)  V, R

Answer: (e)

Directions (Q. Nos. 55-57) Read the following information and answer the given question.

T is the daughter of P. P is the father of L. L is the only son of A. B is the daughter-in-law of A. W is the son of B.

55. How is P related to B?

(a)  Father-in-law

(b)  Brother

(c)  Son-in-law

(d)  Father

(e)  Brother-in-law

Answer: (a)

56. If Y is the husband of T, how is L related to Y?

(a)  Brother-in-law

(b)  Nephew

(c)  Son-in-law

(d)  Son

(e)  Brother

Answer: (a)

57. How is A related to W?

(a)  Uncle

(b)  Grandfather

(c)  Grandson

(d)  Granddaughter

(e)  Grandmother

Answer: (e)

Directions (Q. Nos. 58-62) In the following questions, a statement is followed by two conclusion. You have to consider the statement to be true, even if it is seems to be at variance from commonly known facts. You have to decide, which of the given conclusion, if any following from the given statements.

Code

a. Both conclusion I and II follow

b. Neither conclusion I nor II follows

c. Only conclusion I follows

d. Only conclusion II follows

e. Neither conclusion I nor II follows

58. Statements

No  biscuit is a cookie.

All cookies are pastries.

Some pastries are sandwiches.

Conclusions

I. All pastries are cookies.

II. All biscuits being pastries is a possibility.

Answer: (e)

59. Statements

No biscuit is a cookie.

All cookies are pastries.

Some pastries are sandwiches.

Conclusions

I. At least some cookies are sandwiches.

II. Some sandwiches are biscuits.

Answer: (e)

60. Statements

Some keys are locks.

Some locks are drawers.

All drawers are tables.

Conclusions

I. No key is a drawer.

II. At least some keys are drawers.

Answer: (c)

61. Statements

Some keys are locks.

Some locks are drawers.

All drawers are tables

Conclusions

I. All key can never be tables.

II. At least some locks are tables.

Answer: (a)

62. Statements

All frames are pictures.

Some pictures are images.

Conclusions

I. Some frames are images.

II. All frames are images.

Answer: (c)

Directions (Q. Nos. 63-65) Study the given information carefully to answer the given question.

Six friends – A, B, C, D, E and F, each of different thickness, are kept on a table. F is thicker than B but thinner than C. A is thinner than both B and E, but not thinnest. E is thinner than F. The second thinnest book is 7 cms thick and the second thickest book is 13 cm thick. (Note The thickness of all the books is in whole numbers.)

63. If E is 12 cm thick, then which of the following is true about E?

(a)  F is 2 cms thicker than E

(b)  The total thickness of E and C together is 12 cms

(c)  E is thinner than D

(d)  All the given statements are true

(e)  E is the third thickest book of all

Answer: (e)

64. If A is 2 cm thicker than D, then how thick is D?

(a)  Cannot be determined

(b)  15 cm

(c)  5 cm

(d)  11 cm

(e)  3 cm

Answer: (c)

65. With respect to the thickness of given books, if B + A = 17, then F + B = ?

(a)  17

(b)  21

(c)  23

(d)  19

(e)  25

Answer: (c)

PART C

Quantitative Aptitude

Directions (Q. Nos. 66-70) In this question two equations numbered and II are given. You have to solve both the equations and mark the appropriate answer.

Give answer

a. if x > y b. if x ≥ y

b. if x < y d. if x ≤ y

e. if x = y or relationship can’t be established between x and y.

66. I. x2 – 9x + 18 = 0

II. 5y2 – 22y + 24 = 0

Answer: (a)

67. I. 6x2 + 11x + 5 = 0

II. 2y2 + 5y + 3 = 0

Answer: (b)

68. I. x2 + 10x + 24 = 0

II. y2 – √625 = 0

Answer: (e)

69. I. 10x2 + 11x + 1 = 0

II. 15y2 + 8y + 1 = 0

Answer: (e)

70. I. 15x2 – 11x + 2 = 0

II. 10y2 – 9y + 2 =0

Answer: (d)

71. The time taken by 24 children to complete a project is twice the time taken by 16 women to complete the same If 28 women complete the project. If 28 women complete the project in 8 days, how m any days will 28 women and 24 children together take to complete the project?

(a)  56/9

(b)  18

(c)  19

(d)  21

(e)  24

Answer: (a)

72. The circumference of the semi-circle is 108 cm. If the side of a square is 30% more the diameter of the semi-circle, what is the perimeter of the square?

(a)  212.2 cm

(b)  226.4 cm

(c)  224.8 cm

(d)  214.6 cm

(e)  218.4 cm

Answer: (e)

Directions (Q. Nos. 73-77) What approximate value will come in place of question mark in the given question? (You are not expected to calculate the exact value)

73. 

(a)  15

(b)  18

(c)  9

(d)  35

(e)  27

Answer: (c)

74. 02 + 241 ÷ 6 – 165.11 = ?

(a)  27

(b)  125

(c)  1000

(d)  64

(e)  216

Answer: (b)

75. 09% of 80.15 + 60.04% of 160.12 = ?

(a)  80

(b)  160

(c)  180

(d)  140

(e)  128

Answer: (e)

76. 082 – 3.01 × 104.11 ÷ 4.02 = ?

(a)  280

(b)  200

(c)  160

(d)  120

(e)  180

Answer: (d)

77. 

(a)  −1364

(b)  20

(c)  6

(d)  15

(e)  10

Answer: (a)

78. Ronnie invested Rs P in a scheme A offering simple interest at 12% per annum for two years. He invested the whole amount he received from scheme A, in another scheme B offering simple interest 15% pa for two years. If the difference between the interest earned from scheme A and B was Rs 264, what is the value of P?

(a)  2640

(b)  2500

(c)  2250

(d)  1800

(e)  2000

Answer: (e)

Directions (Q. Nos. 79-83) Study the table and answer the given question.

79. What is the respective ratio between the total number of females studying stream C in all the universities together and the total number of females studying in stream D in all the universities together?

(a)  20 : 13

(b)  15 : 14

(c)  5 : 4

(d)  15 : 13

(e)  4 : 5

Answer: (d)

80. Total number of males studying in stream A in all the universities together in 2010 is 1200 more than that in the year 2009. In 2010, what was the total number of students (male + female) studying in stream A in all the universities together, if the total number of male students in stream A in 2010, constituted 50% of the total number of students?

(a)  6400

(b)  6000

(c)  5000

(d)  3000

(e)  2000

Answer: (a)

81. Total number of students studying in stream A and B together in VK are what percent less than those studying in same streams together in St. James?

(a)  35%

(b)  38%

(c)  40%

(d)  52%

(e)  30%

Answer: (e)

82. What is the average number of male students studying in stream D in all the given universities?

(a)  240

(b)  210

(c)  290

(d)  310

(e)  340

Answer: (c)

83. Number of students studying in stream C in VK is what percent of that studying in stream B in DVA?

(a)  35%

(b)  38%

(c)  40%

(d) 

(e) 

Answer: (c)

84. The distance between two cities (M and N) is 350 km. A train starts from city M at 6 am and travel towards city N at the speed of 63 km/h. Another train starts from city N at 7 am and travel towards city M at the speed of 77 km/h. At what time will the trains meet?

(a)  5 : 00 am

(b)  10 : 00 am

(c)  9 : 05 am

(d)  8 : 24 am

(e)  8 : 30 am

Answer: (c)

85. A, B and C started a business with investments of Rs 1500, Rs 550 and Rs 2400 respectively. After 8 months from the start of the business, A and C left and B invested an additional amount of Rs 450. If different between the Share in annual profit received by B and the total annual profit was Rs 13000, what was the total annual profit received?

(a)  Rs 16500

(b)  Rs 18150

(c)  Rs 13200

(d)  Rs 19800

(e)  Rs 19000

Answer: (a)

Directions (Q. Nos. 86-90) What value will be come in place of question marks in the following number series?

86. 14   15      32      99      ?        2005

(a)  354

(b)  372

(c)  400

(d)  368

(e)  324

Answer: (c)

87. 8   5        4        7        13      ?

(a)  31

(b)  37

(c)  33.5

(d)  35.5

(e)  31.5

Answer: (c)

88. 4   9        20      43      84      ?

(a)  149

(b)  157

(c)  191

(d)  145

(e)  137

Answer: (a)

89. 13    15      20      37      102    ?

(a)  351

(b)  359

(c)  375

(d)  377

(e)  363

Answer: (b)

90. 69   74      67      78      65      ?

(a)  85

(b)  78

(c)  82

(d)  84

(e)  75

Answer: (c)

91. In Jar A, 140 litres milk was mixed with 40 litre water. Some of this mixture was taken out from Jar A and put in Jar B. If after 17 litre milk in Jar B, the resultant ratio between milk and water in Jar B was 19 : 3 respectively, what was the amount of mixture that was taken out from Jar A?

(a)  21 L

(b)  36 L

(c)  46 L

(d)  18 L

(e)  27 L

Answer: (e)

Directions (Q. Nos. 92-96) Refer to the pie-charts and answer the given questions.

92. Number of bags sold by store M decreased by what percent from 2004 to 2006?

(a) 

(b) 

(c) 

(d) 

(e) 

Answer: (c)

93. What is the respective ratio between total number of bags sold by stores M and N together in 2006 and that in 2008?

(a)  13 : 8

(b)  11: 9

(c)  6 : 5

(d)  9 : 8

(e)  11 : 8

Answer: (d)

94. In 2004, 30% of the bags sold by store M and 40% of the bags sold by store N were leather bags. What was the total number of leather bag sold by store M and N together in 2004?

(a)  168

(b)  172

(c)  184

(d)  164

(e)  176

Answer: (b)

95. If the average number of bags sold by store N in 2007, 2008 and 2009 was 305, what was the number of bags sold by the same store in 2009?

(a)  420

(b)  445

(c)  425

(d)  440

(e)  415

Answer: (c)

96. What is the difference between total number of bags sold by stores M and N together in 2005 and that in 2007?

(a)  250

(b)  240

(c)  210

(d)  260

(e)  290

Answer: (b)

97. Three years ago, Manini’s age at that time was thrice of Rinu’s age at that time. The respective ratio between Rinu’s age six years hence and Manini’s age eight years hence, will be 3 : 7. What will be Rinu’s age two years hence?

(a)  20 yr

(b)  26 yr

(c)  29 yr

(d)  32 yr

(e)  23 yr

Answer: (a)

98. The respective ratio between the monthly salary of Neil and that of Dipti is 5 : 6. Neil and Dipti, both save 40% and 25% out of their respective monthly salary. Neil invests 5/8th of his savings in LIC and Dipti invests 3/5th of her savings in LIC. If Neil invests Rs. 1750 more than Dipti in LIC, what is Neils’s monthly salary?

(a)  Rs 20000

(b)  Rs 25000

(c)  Rs 40000

(d)  Rs 15000

(e)  Rs 30000

Answer: (b)

99. Cost price of two beds are equal. One bed is sold at a profit of 25% and the other one for Rs 6596 less than the first one. If the overall profit earned after selling both the beds is 8%, what is the cost price of each bed?

(a)  Rs 20400

(b)  Rs 19400

(c)  Rs 18600

(d)  Rs 19400

(e)  Rs 16800

Answer: (b)

100. A bag contains 16 eggs out of which 5 are rotten. The remaining eggs are in good condition. If two eggs are drawn randomly, what is the probability that one of the eggs drawn is rotten?

(a)  11/24

(b)  13/24

(c)  5/12

(d)  17/24

(e)  7/12

Answer: (a)

IBPS CRP-V Specialist Officer (IT) Exam-2015 Online Held on February 14, 2016 Question Paper With Answer Key

IBPS CRP-V Specialist Officer (IT) Exam-2015 Online Held on February 14, 2016
IBPS CRP-V Specialist Officer (IT) Exam-2015 Online Held on February 14, 2016 Question Paper With Answer Key

IBPS CRP-V Specialist Officer (IT) Exam-2015 Online Held on February 14, 2016

Part I Reasoning

1. This question consists of an information and two statements numbered I and Ii given below it. You have to decide which of t he given statements weaken/s or strengthen/s the information, and decide the appropriate answer.

Information Since that past three years, more number of students in country G are opting for law courses as compared to the earlier years.

I. The number of applications for admission in law colleges of country G has been consistent since the past 5 years.

II. The number of students ready to pay heavy donations to get admission in the most sought after law college of country G are consistently increasing for past three years.

(a)  Both statement I and statement II are neutral statements.

(b)  Both statement I and statement II weakens the information.

(c)  Statement I strengthens the information, while statement II weakens the information.

(d)  Statement I weakens the information, while statement II strengthens the information.

(e)  Both statement I and statement II strengthen the information.

Answer: (e)

2. Read the given information and answer the question.

Patients of lung ailments were mostly prescribed Medicine X which contains only two constituents viz ‘agnet and ‘serovil’. However, there were reports that many patients developed severe addiction to it on long term u se. Therefore, doctors now prescribe Medicine A to deal with the same because it contains ‘agnet’ and ‘servoil’.

Which of the following can be concluded from the given statement?

(a)  A medicine containing only serovil would not aid in treatment of the mentioned lung disease at all.

(b)  Medicine A contains no other constituent other than agnet.

(c)  Short term usage of Medicine X does not develop its addiction even to a minor extent.

(d)  Medicine X is not prescribed to treat any aliment these days.

(e)  Serovil was the constituent responsible for causing addiction of Medicine X.

Answer: (d)

Directions (Q. Nos. 3-7) Study the following information to answer the given questions.

Twelve people are sitting in two parallel rows containing six people each, in such a way that there is an equal distance between adjacent persons. In row-I, G, H, I, J, K and L are seated (but not necessarily in the same order) and all of them are facing South. In row-2 S, T, U, V, W and X are seated (but not necessarily in the same order) and all of them are facing North. Therefore; in the given seating arrangement, each member seated in a row faces another member of the other row.

The one who faces H sits third to the left of V. V does not sit at any of the extreme ends of the line. S sits second to the left of V. The one facing T sits third to the right of G. T does not sit any of the extreme ends of the line. Only one person sits  between G and I. K is neither an immediate neighbour of I nor H. The one facing W sits second to the left L. L does not face X.

3. Who amongst the following sits second to the left of person who faces T?

(a)  H

(b)  I

(c)  J

(d)  K

(e)  L

Answer: (e)

4. Which of the following is true regarding S?

(a)  Both T and W are immediate neighbours of S.

(b)  None of the given  options is true.

(c)  Only one person sits between S and U.

(d)  S sits second to right of X.

(e)  K is an immediate neighbour of the person who faces S.

Answer: (d)

5. Who amongst the following faces L?

(a)  U

(b)  S

(c)  T

(d)  W

(e)  V

Answer: (a)

6. Which of the following groups of people represents the people sitting at extreme ends of both the rows?

(a)  H, I, S, U

(b)  L, J, X, U

(c)  J, K, X, W

(d)  I, L, U, X

(e)  H, G, ,S, W

Answer: (c)

7. Which of the following is true with respect to the given information?

(a)  I faces W.

(b)  None of the given options is true.

(c)  G sits exactly between J and I.

(d)  U is an immediate neighbour of W.

(e)  H faces one of the  immediate neighbours of X.

Answer: (e)

Directions (Q. Nos. 8-12) Study the following information carefully and answer the given questions.

When a word and number arrangement machine is given an input line of words and numbers, it arranges them following a particular rule. The following is an illustration of input and rearrangement.

(All the numbers are two-digit numbers.)

Input centre 24 actual 15 require impact 37 96 marine 49 unable 82

Step I 24 centre actual 15 require impact 37 96 marine  unable 82 49

Step II 82 24 centre actual 15 require impact 96 marine unable 49 37

Step III 96 82 24 centre actual require impact marine unable 49 37 15

Step IV centre 96 82 24 actual require impact marine 49 37 15 unable

Step V marine center 96 82 24 actual require 49 37 15 unable impact

Step VI require marine centre 96 82 24 49 37 15 unable impact actual

Step VI is the last step of the above arrangement as the intended arrangement is obtained.

As per the rules followed in the given steps, find out the appropriate steps for the given input.

Input embark 53 palace 16 65 salute obvious 42 achieve 71 heaven 98

8. Which element is fourth to the right of the one which is ninth from the right end in Step III of the given input?

(a)  16

(b)  achieve

(c)  53

(d)  salute

(e)  obvious

Answer: (b)

9. Which element comes exactly between ‘16’ and ‘salute’ in step V of the given input?

(a)  Both obvious and heaven

(b)  Only achieve

(c)  Both embark and palace

(d)  Only 65

(e)  Only 71

Answer: (e)

10. If in the Vth step, ‘16’ interchanges its position with ‘obvious’ and ‘heaven’ also interchanges its position with ‘65’, then which element will be fifth to the right of ‘42’?

(a)  16

(b)  65

(c)  obvious

(d)  71

(e)  heaven

Answer: (e)

11. Which of the following combinations represents the first two and last two elements in step VI of the given input?

(a)  salute, obvious, embark, palace

(b)  heaven, palace, achieve, obvious

(c)  palace, heaven, obvious, embark

(d)  salute,  palace, embark, achieve

(e)  embark, obvious, palace, achieve

Answer: (b)

12. In which step are the elements ‘heaven 98 achieve 71’ found in the same order?

(a)  Fourth

(b)  sixth

(c)  fifth

(d)  third

(e)  The given order of elements is not found in any step

Answer: (e)

Directions (Q. Nos. 13-17) Study the given information carefully to answer the given questions.

Eight people P, Q, R, S, W, X, Y and Z live on eight different floors of a building, but not necessarily in the same order. The lowermost floor of the building is numbered one, the one above that is  numbered two and so on till the topmost floor is numbered eight.

P lives on an even numbered floor above the floor numbered three. Y lives immediately below P. Only three people live between Y and R. Only two people live between R and W. There are as many people between W and P as are there between W and Q. Only two people live between Q and Z. S lives immediately above Z.

13. Who amongst the following live exactly between Y and W?

(a)  Only P

(b)  Only X

(c)  No one

(d)  Both P and X

(e)  Both X and Q

Answer: (c)

14. Who amongst the following lives on the floor numbered five?

(a)  X

(b)  Q

(c)  S

(d)  R

(e)  Y

Answer: (a)

15. As per the given arrangement, four of the following five are alike in a certain way and so form a group. Which one of the following does not belong to t he group?

(a)  Q-Two

(b)  V-Seven

(c)  R-Five

(d)  P-Six

(e)  Z-Three

Answer: (b)

16. On which of the following floor numbers does R live?

(a)  Seven

(b)  Six

(c)  Three

(d)  Eight

(e)  Two

Answer: (c)

17. If X and P interchange their places and so do Z and R, then who will live between P and Z as per the new arrangement?

(a)  Q

(b)  W

(c)  R

(d)  Y

(e)  Other than those given as options

Answer: (a)

Directions (Q. Nos. 18-23) Study the following information and answer the questions.

Seven friends, namely P, Q, R, S, T, U and V watch even different movies namely Spectre, Joy, Burnt, Concussion, Frozen, Everly and Legend, not necessarily in the same order, starting from Monday to Sunday (of the same week).

T watches a movie on Thursday. Only one person watches movie between T and the one who watches Frozen. P watches movie immediately after the one who watches Frozen. Only three people watch movie between P and the one who watches Burnt.

Only two people watch movie between the one who watches Burnt and U. The one who watches Legend watches  movie before U, but after Thursday. More than two people watch movie between the one who watches Legend and R. The one who watches Spectre watches movie immediately before the one who watches Everly. The one who watches Concussion watches movie  immediately before Q. V does not watch movie on Tuesday.

18. Who amongst the following watches Joy?

(a)  R

(b)  S

(c)  P

(d)  T

(e)  V

Answer: (a)

19. Four of the following five are alike in a certain way and so form a group. Which of the following does not belong to that group?

(a)  R-Tuesday

(b)  T-Thursday

(c)  Q-Sunday

(d)  P-Friday

(e)  U-Saturday

Answer: (a)

20. Which of the following is true about S?

(a)  S watches movie immediately after R.

(b)  S watches Burnt.

(c)  Only two people watch movie between S and R.

(d)  All the given statements are true.

(e)  S watches movie on Sunday.

Answer: (b)

21. Which of the following movies does T watch?

(a)  Burnt

(b)  Spectre

(c)  Everly

(d)  Concussion

(e)  Legend

Answer: (b)

22. As per the given arrangement Q is related to the one who watches Burnt in a certain way and R is related to the one who watches Everly in the same way. To which of the following is U related to in the same way?

(a)  The one who watches Joy.

(b)  The one who watches Concussion.

(c)  The one who watches Burnt.

(d)  The one who watches Spectre.

(e)  The one who watches Legend.

Answer: (b)

23. On which of the following days does V watch a movie?

(a)  Friday

(b)  Sunday

(c)  Wednesday

(d)  Saturday

(e)  Monday

Answer: (e)

24. Read the following information and answer the question.

The women of Village X are becoming financially independent after Shakti, an NGO, started assisted the women in mastering art of making organic colours thus helping them to earn a living.

Which of the following statements can be inferred from the given statement? (An inference is something by, which you can logically deduce something to be true based on known premises.)

(a)  In order to master the art of making organic colours, one must seek the assistance of Shakti only.

(b)  Organic colours made by the women of Village X have captured the interest of people to some extent.

(c)  The women of Village X have never tried their hand at any art except colour making.

(d)  Shakti provides assistance only in the field of organic colour making and specifically to women.

(e)  Women of village X have never been assisted by any other NGO in the past.

Answer: (c)

25. How many such pairs of letters are there in the word NEUTRAL each o f which has as many letters in the word in both forward and backward directions as there are between them in English alphabetical series?

(a)  Three

(b)  None

(c)  Two

(d)  More than three

(e)  One

Answer: (a)

26. Study the given information carefully to answer the given question.

The following are two findings of an internal survey of a catering company ‘Mini Menu’ based in City X.

A. Each year,, the profit earned from the orders having multi-cuisine menu is much higher than that earned from Indian menu.

B. This year, ‘Mini Menu’ had 45% more orders for multi-cuisine menu as compared to any of the previous years, though the other orders remained more or less same.

Which of the following can be inferred from the given information?

(An inference is something by which you can logically deduce something to be true based on known premises.)

(a)  The ingredients used to cook Indian foods are costlier as compared to those used in other cuisines.

(b)  This year, ‘Mini Menu’ earned the highest profit as compared to all the previous years of its service.

(c)  The numbers of multi-cuisine orders placed within ‘Mini Menu’ this year were double the number of all other orders.

(d)  Indian cuisines require much more effort than any other cuisine,. thus becoming the costliest menu for parties.

(e)  This year the numbers of parties/functions held in city X were much higher as compared to any other  year.

Answer: (d)

27. Read the given information and answer the question.

Although School Z has started providing a number of facilities like free lunch for students, independent class report of each student and yearly picnics, for some years now, parents still prefer to enroll their children in school M over school Z.

Which of the following may not be a reason for the parents’ preference for school M over school Z?

(a)  School M is the only school in the city which has student friendly class rooms so that even those sitting at the last benches can clearly  hear the teacher and see the board.

(b)  School M provides door to door bus service for students, while school Z provides bus service up to the nearest bus top which may be within 2 km of the  house of the child.

(c)  Unlike school Z, school M uses audio visual aids for teaching, which helps students to understand concepts better and to retain them.

(d)  School M allows the students to give specific feedback regarding the teachers, while school Z has surprise inspection every month to assess the effectiveness of the teacher.

(e)  School M, unlike school Z, provides locker facilities to students and gives notes and homework on pen drive, to ensure that students do not have to carry heavy bags to the school every day.

Answer: (e)

28. If ‘3’ is subtracted from each odd digit and ‘1’ is added to each even digit in the number 8547692, which of the following numbers will appear twice in the new number thus formed?

(a)  Only 2

(b)  Both 3 and 7

(c)  None

(d)  Both 3 and 5

(e)  Only 4

Answer: (c)

Directions (Q. Nos. 29-33) In each of the question, two/three statements followed by two conclusions numbered I and II have been given. You have to take the given statements to be true even if they seem to  be at variance from the commonly known facts and then decide which of the given conclusions logically follows from the given statements disregarding commonly known facts.

Give answer

a. if only conclusion I follows

b. if only conclusion II follows

c. if either conclusion I or II follows

d. if neither conclusion I nor II follows

e. if both conclusions follow

29. Statements Some ideas are clues.

Some clues are hints.

No hint is a notion.

Conclusions I. No idea is a hint.

II. At least some ideas are hints.

Answer: (b)

30. Statements All gardens are parks.

All parks are lawns.

Some lawns are orchards.

Conclusions I. At least some orchards are gardens.

II. All lawns are parks.

Answer: (d)

31. Statements All gardens are

All parks are lawns

Some laws are orchards.

Conclusions I. All gardens are lawns.

II. All orchards being parks is a possibility.

Answer: (a)

32. Statements No spice is a flavor.

No flavor is a colour.

Conclusions I. At leats some spices are colours.

II. All spices being colours is a possibility.

Answer: (d)

33. Statements Some ideas are clues.

Some clues are hints.

No hints is a notion.

Conclusions I. No notion is a clue.

II. At least some ideas are notions.

Answer: (a)

Directions (Q. Nos. 34-38) In these questions, relationship between different elements is shown in the statements. The statements are followed by conclusions. Study the conclusions based on the given statement and select the appropriate answer.

Give answer

a. if only conclusion I is true

b. if neither conclusion I nor II is true

c. if both conclusions are true

d. if only conclusion II is true

e. if either conclusion I or II is true

34. Statements U > Y ≥ W ≤ K; W = X ≥ Z

Conclusions I. U > K       II. Z ≤ K

Answer: (a)

35. Statements G ≥ H > J ≤ K; H > M; J > U

Conclusions I. H > U       II. M < G

Answer: (d)

36. Statements L ≤ K > J ≥ U; J ≤ T ≤ R

Conclusions I. T > L        II. U ≤ R

Answer: (c)

37. Statements P ≥ Q ≥ W = S ≥ L; Y ≥ S

Conclusions I. P > Y        II. Y = P

Answer: (e)

38. Statements G ≥ H > J ≤ K; H > M; J > U

Conclusions I. M < K      II. K > U

Answer: (c)

Directions (Q. Nos. 39-43) Study the following information carefully and answer the given questions.

Eight friends, J, K, L, M, N, O, P and Q are sitting around a square table in such way that four of them sit at four corners while four sit in the middle of each of the four sides. The ones who sit in middle of the sides face the centre while those who sit at the four corners face outside (i.e. opposite to the centre.)

L sits third to the right of M. M sits in middle of one of the sides of the table. Only three people sit between L and Q. Only one person sits between Q and P. N is one of the immediate neighbours of P. Only three people sit between N and K. J sits second to the right of K.

39. How many people sit between J and P when counted from the right of P?

(a)  Two

(b)  One

(c)  None

(d)  Three

(e)  Four

Answer: (c)

40. Four of the following five are alike in a certain way and so form a group. Which is the one that does not belong to that group?

(a)  Q

(b)  O

(c)  P

(d)  L

(e)  K

Answer: (e)

41. Which of the following is true regarding O?

(a)  O sits at  middle of the one of the sides.

(b)  Both M and K are immediate neighbours of O.

(c)  N sits second to left of O.

(d)  Only three people sit between O and J.

(e)  None of the given options is true

Answer: (b)

42. What is the position of L with respect to P?

(a)  Second to the right

(b)  Fourth to the right

(c)  Third to the left

(d)  Fourth to the left

(e)  Second to the left

Answer: (a)

43. Who sits second to the left of Q?

(a)  P

(b)  O

(c)  K

(d)  J

(e)  N

Answer: (b)

Directions (Q. Nos. 44-48) Study the given information carefully to answer the questions.

‘ban all animals products’ is written as ‘tp fm ax bz’

‘animal hunting is wrong’ is written as ‘ke hw tp u’

‘all are wrong answers’ is written as ‘dy nib z hw’

‘products are well known’ is written as ‘fm gr sl dy’

(All codes are two-letter codes only)

44. What does the code ‘sl’ stand for in the given code language?

(a)  either ‘well’ or ‘known’

(b)  products

(c)  either ‘hike’ or ‘people’

(d)  are

(e)  profit

Answer: (a)

45. What will be the possible code for ‘wrong products received’ in the given code language?

(a)  tp dy gr

(b)  fm zu hw

(c)  gr fm tp

(d)  fm hw ni

(e)  ni zu fm

Answer: (b)

46. In the given code language, if ‘busy’ is coded as ‘ot’, then how will ‘is busy hunting’ be coded as?

(a)  cu ot hw

(b)  tp ke ot

(c)  ot hw tp

(d)  ot ke hw

(e)  ot cu ke

Answer: (e)

47. What will be the code for ‘animal’ in the given code language?

(a)  cu

(b)  hw

(c)  tp

(d)  fm

(e)  Other than those given as options

Answer: (c)

48. What is the code for ‘all’ in the given code language?

(a)  hw

(b)  ax

(c)  bz

(d)  ni

(e)  dy

Answer: (c)

Directions (Q. Nos. 49-50) The following questions consist of a question and two statements numbered I and II given below it. You have decide whether the data given in the statements are sufficient to answer the questions. Read both the statements and

Give answer

a. if the data in statement I alone are sufficient to answer the question, while the data in statement II alone are not sufficient to answer the question

b. if the data in statement II alone are sufficient to answer the question, while the data in statement I alone are not sufficient to answer the question

c. if the data either in statement I alone or in statement II alone are sufficient to answer the question

d. if the data even in both statements together are not sufficient to answer the question

e. if the data in both statements together are necessary to answer the question

49. How far is point R from point T?

I. Point R is 5 m to the north of Point M. Point U is 4 m to the east of Point R. Point T is to the west of Point R such that points U, R and T form a straight line of 6 m.

II. Point Z is to be south of Point T. Point U is 6 m to the east of point T. Point M is 2 m to the East of point Z. Point R is 5 m to the North of point M. Point R lies on the line formed by joining points T and U.

Answer: (c)

50. How many people are standing in a straight line? (Note All are facing North)

I. R stands third from the left end of the line. Only one person stands between R and U. V stands second to the right of U. V stands at one of the extreme ends of the line.

II. M stands at exactly the centre of the line. Only two people stand between M and J. Only three people stand between J and U. Only one person stands between U and V.

Answer: (e)

Part II English Language

Directions (Q. Nos. 51-55) The question has two blanks, each blank indicating that something has been omitted. Choose the set of words for each blank that best fits the meaning of the sentence as a whole.

51. Couples possessing a specific gene were less bothered by the emotional …….. in their marriage as ……. to those who didn’t have that gene.

(a)  upheavals; compared

(b)  traumas; against

(c)  challenges; belonged

(d)  tribulations; regards

(e)  bonding; respect

Answer: (a)

52. Chhattisgarh was ………. the first few States in the country that ……… a ban on ‘Gutkha’.

(a)  among; imposed

(b)  between; set

(c)  part; restricted

(d)  one; lifted

(e)  along; force

Answer: (a)

53. Right from a young age, children should be taught about the harmful effects of smoking so that they ………. from ……… in it once they grow up.

(a)  avert; revelling

(b)  resolve; rejoicing

(c)  refrain; indulging

(d)  abstain; taking

(e)  withdraw; addicting

Answer: (c)

54. Childhood obesity is a major health ………. and a leading ………… of cancer.

(a)  hazard; result

(b)  issues; reason

(c)  matter; grounds

(d)  problem; cause

(e)  phenomenon; base

Answer: (d)

55. A police team was ………. to …………….. the case.

(a)  formed; following

(b)  constituted, investigate

(c)  located; adjudge

(d)  remanded; study

(e)  erected; probe

Answer: (b)

Directions (Q. Nos. 56-65) Read each sentence to find out whether there is any grammatical error in it. The error, if any, will be in one  part of the sentence. Mark the part with the error as your answer. If there is no error, mark ‘No error’ as your answer.

(Ignore the errors of punctuation, if any.)

56. Bad lipstick woes/ strike even/ the most diligent/ of makeup users.

(a)  Bad lipstick woes

(b)  strike even

(c)  the most diligent

(d)  of makeup users

(e)  No error

Answer: (d)

57. Our behaviour/ and interest in/ socializing is determined / by our genes.

(a)  Our behaviour

(b)  and interest in

(c)  socializing is determined

(d)  by our genes.

(e)  No error

Answer: (e)

58. With the problem of/ shortage of doctors/ plagues the entire State/ rural areas are the worse hit.

(a)  With the problem of

(b)  shortage of doctors

(c)  plagues the entire State

(d)  rural areas are the worse hit

(e)  No error

Answer: (a)

59. While the government continues to /promise better health care facilities / for all the residents of the State, / rural areas remain a major cause on concern.

(a)  While the government continues to

(b)  promise  better health care facilities

(c)  for all the residents of the State

(d)  rural areas remain a major cause on concern

(e)  No error

Answer: (d)

60. Human temperamental factors, / which motivate people to / form relationships and maintain them, / are prove to have a genetic base.

(a)  Human temperamental factors

(b)  which motivate people to

(c)  form relationships and maintain them

(d)  are prove to have a genetic base

(e)  No error

Answer: (d)

61. The man, who was alleged / stolen a sum of Rs 6 lakh, / was finally / arrested by the police.

(a)  The  man, who was allegedly

(b)  stolen a sum of Rs 6 lakh

(c)  was finally

(d)  arrested by the police

(e)  No error

Answer: (a)

62. The tobacco laced toothpaste, / popular in the rural areas of the State, / is considered to been the major cause behind / the rising member of oral cancer cases.

(a)  The tobacco laced toothpaste

(b)  popular in the rural areas of the State

(c)  is considered to been the major cause behind

(d)  the rising member of oral cancer cases

(e)  No error

Answer: (d)

63. It is important to note/ that childhood cancers can be prevented / to a greatest extent / if a healthy lifestyle is followed.

(a)  It is important to note

(b)  that childhood cancers can be prevented

(c)  to a greatest extent

(d)  if a healthy lifestyle is followed

(e)  No error

Answer: (c)

64. A watermelon / is one of the healthiest fruit / to eat, if you are / planning to go on a diet.

(a)  A watermelon

(b)  is one of the healthiest fruit

(c)  to eat, if you are

(d)  planning to go on a diet

(e)  No error

Answer: (b)

65. Patients can now scheduled / appointments while at home/ and get confirmation for the same/ via email.

(a)  Patients can now scheduled

(b)  appointments while at home

(c)  and get confirmation for the same

(d)  via email

(e)  No error

Answer: (a)

Directions (Q. Nos. 66-75) Read the following passage carefully and answer the questions. Certain words/phrases are given in bold to help locate them, while answering some of the questions.

In 2012, the Arctic was hot. But while still matters for environmental reasons, the surge of interest in its economy has ebbed. That surge was driven by three things. First, the Arctic contains vast amounts of energy, which would become accessible as the world warms and the ice retreats in summer.

The US Geological survey has said that about a quarter of the world’s undiscovered oil and gas lies in Arctic waters. Second, the melting ice allows cargo ships to sail round Russia’s northern coast for about two months in summer. This cuts the distance for ships travelling from Shanghai to Rotterdam by almost a quarter and the transmit time by about two weeks. It was once said the Northern.

Sea Route would one day, rival the Suez canal as the best way to ship goods from East Asia to Europe. Third, ,the Arctic seemed a model of international cooperation. The eight countries with territory inside Arctic circle settled through the Arctic Council, originally a scientific forum which in 2011-13 signed its first treaties, on search and rescue missions and cleaning up oil spills. Nothing exemplified its popularity better than the rush of tropical Asian countries to join. China, India and Singapore were granted observer status in 2013.

But since mid-2014, the Arctic’s allure has lessened. Its energy is pricey. Even at $100a barrel, many fields are marginal because the weather is so extreme. A Russian and Norwegian firm together developing one of the largest gas fields ever discovered, mothballed the project in 2012. With o il at $50 a  barrel, few Arctic fields would be economic. Energy exploration in the Arctic is in fact referred to by some as a license to lose money. With regard to the Northern Sea Route- In 2013, 71 ships traversed Russia’s Arctic, according to the Northern Sea Route Information Office : a large increase since 2010, when the number was just four. But 16000 ships passed through the Suez canal between Europe and Asia in 2013, so the northern route is not starting to compete. In 2014 traffic along the Northern Sea Route fell to 53 ships, only four of which sailed from Asia and docked in Europe (the rest went from one Russian port to another.)

The route does not yet link Europe and East Asia. The decline in 2014 was partly caused by the weather: less sea ice melted last summer than in 2013, so the route was more dangerous. But its limitations go beyond that. Cutting a week or two off transit time is not the benefit it may seem if the vessel arrives a day late. In shipping, just-in-time arrival matters, not only speed. The new-generation container ships are too cumbersome to use the Arctic so, as these become more common, the northern route becomes less attractive.

The Arctic Council continues to expand: it is setting up a new economic body to boost business. But however much its members cooperate, the council cannot offset hostilities between Russia and the West-hostilities, which affect the Arctic, too. Russia is stepping up its military operation there. This does not mean fighting is about to break out in the Arctic; nor are shipping and energy exploration about to end.

66. Which of the following can be said about the Suez canal?

(a)  Its shipping traffic has dramatically reduced with the opening up of the Northern Sea Route.

(b)  Russian shipping traffic through this route is increasing dramatically.

(c)  It is at present the best way to ship goods between Europe and East Asia.

(d)  New-generation cargo ships are facing challenges along this route.

(e)  Work to expand its capacity should be undertaken soon.

Answer: (c)

67. Which of the following is true in the context of the passage?

(a)  Climate change has resulted in a huge increase in Arctic temperatures.

(b)  Oil and natural gas reserves in the Arctic have more or less been depleted.

(c)  Coal and natural gas are the most effective energy sources today.

(d)  Asian countries have been awarded the major mining rights in the Arctic.

(e)  None of the given statements is true in the context of the passage.

Answer: (e)

68. What does the phrase’ as a license to lose money’ convey?

(a)  The penalties for polluting the Arctic are very heavy.

(b)  Exploring the energy in the Arctic is expensive and may not be profitable for companies.

(c)  It is very difficult to obtain requisite permissions to explore the Arctic explorers.

(d)  Mining for minerals in the Arctic poses risks to the health and lives of explorers.

(e)  As countries have deliberately lowered the price of oil, Russian oil companies are in financial trouble.

Answer: (b)

69. Choose the word which is most nearly the same in meaning to the word ‘Lies’ given in bold as used in the passage.

(a)  Rests

(b)  Reclines

(c)  Untruths

(d)  Reality

(e)  Lazy

Answer: (a)

70. Which of the following is the central idea of the passage?

(a)  The Arctic has a lot of resources and many countries are fighting over these at present.

(b)  Dialogue is the only way to achieve resolution of the conflict between Russia and the West.

(c)  The Arctic region has seen a major economic transformation thanks to its natural health.

(d)  The Arctic circle is in the midst of a tremendous environmental and economic crisis.

(e)  While the potential of the arctic was hyped in the past, today there are reservations about it.

Answer: (e)

71. What does the author suggest regarding the Arctic Council?

A. It should convince its members to reduce the price of oil.

B. It should not be exclusive and should admit other countries as members.

C. It should continue to remain purely a scientific body.

(a)  Only A

(b)  B and C

(c)  All of these

(d)  A and C

(e)  None of these

Answer: (e)

72. Choose the word which is opposite in meaning to the word ‘Extreme’ given in bold as used in the passage.

(a)  Nearest

(b)  Minimum

(c)  Moderate

(d)  Peaceful

(e)  Insignificant

Answer: (e)

73. What is the author’s view regarding Russia’s present actions?

(a)  It will shortly result in military conflict in the region.

(b)  It has caused closure of shipping through the Northern Sea Route.

(c)  Oil and gas projects with Norway will be suspended.

(d)  It will adversely affect the spirit of cooperation in the Arctic region.

(e)  Other than those given as options

Answer: (e)

74. Which of the following is/are (a) reason/s which impact energy exploration in the Arctic?

A. High cost of exploration projects.

B. Climatic conditions.

C. Excessive red tape and clearances to be obtained from the Arctic Council.

(a)  Only A

(b)  Only B

(c)  All of these

(d)  A and B

(e)  B and C

Answer: (a)

75. What do the statistics regarding the Northern Sea Route cited in the passage indicated?

(a)  It is  drastically reduced the burden on the Suez canal.

(b)  It has not been utilized to the degree anticipated.

(c)  It  has caused the Arctic to be severely polluted.

(d)  There have been a huge number of accidents along this route.

(e)  The burden on Russian ports as decreased substantially.

Answer: (b)

Directions (Q. Nos. 76-85) Which of the phrases given against the sentence should replace the word/phrase given in bold in the sentence to make it grammatically correct? If the sentence is correct as it is given an no correction is required, mark ‘No correction required’ as the answer.

76. While buying a home or renting an apartment, one of the most sought after spaces is that of parking.

(a)  few of the much

(b)  some of much

(c)  once the most

(d)  one for the most

(e)  No correction required

Answer: (e)

77. Some of the best experiments coming from outside the chain of command.

(a)  comes of

(b)  comes from

(c)  coming for

(d)  come from

(e)  No correction required

Answer: (d)

78. Besides the obvious tangible benefits, the new policy will have in immensely positively effect on the mindsets of working women.

(a)  immensely positively affect

(b)  effective positive

(c)  immense positive affect

(d)  immensely positive effect

(e)  No correction required

Answer: (d)

79. Banks will now take control of a company that owes its customers Rs 9500 crore.

(a)  take controlling of

(b)  takes control of

(c)  controlled

(d)  taking control for

(e)  No correction required

Answer: (e)

80. Labour markets around the world have not been able to keep pace on rapid shifts in the global economy.

(a)  kept pace with

(b)  keep pace with

(c)  keeping pace on

(d)  keeps pace in

(e)  No correction required

Answer: (b)

81. According to media, the CEO has energized the company and build strong foundations for its future.

(a)  built strong

(b)  strongly build

(c)  building strong

(d)  strongly building

(e)  No correction required

Answer: (a)

82. India now qualifies among the sixteen countries having the longer paid leave for new mothers.

(a)  having the long

(b)  having the longest

(c)  who has the longer

(d)  the longest

(e)  No correction required

Answer: (b)

83. APJ Abdul Kalam rose from humbly beginnings to become the country’s top missile scientist and its first technocrat President.

(a)  rose from humble

(b)  rising from humble

(c)  rise of humble

(d)  risen humbly of

(e)  No correction required

Answer: (a)

84. To secure privacy meaningfully, some limits have to be place on the government’s ability to gather information.

(a)  has to be placed on

(b)  having being placed in

(c)  have to be placed on

(d)  had placed on to

(e)  No correction required

Answer: (c)

85. As predicted by the IMO, monsoon is likely too stay strong for they next strong days.

(a)  lie to stay

(b)  likely for staying

(c)  likely to stay

(d)  liked for staying

(e)  No correction required

Answer: (c)

Directions (Q. Nos. 86-90) Rearrange the following six sentence A, B, C, D, E and F in a proper sequence to form a meaningful paragraph, then answer the given questions.

A. To this affect, the Prime Minister recently appealed to the farmers to grow pulses on a part of their land.

B. We would rather not produce this to be deposed of crop in the first place as production takes a toll not only one the natural resources like soil and water, but also impacts environment because of green house emissions.

C. Every year Indians spend crores of rupees from the public exchequer for preventing decay of specific foodgrains, but to no avail.

D. Alternatively, we could produce other crops like pulses, which we import very often.

E. But such appeals in all likelihood will go unheeded as long as distorted incentives to produce cereals-wheat and rice-continue.

F. We then spend a fortune again to dispose of the plied up waste.

86. Which of the following should be the third sentence after the rearrangement?

(a)  A

(b)  B

(c)  F

(d)  D

(e)  E

Answer: (c)

87. Which of the following should be the sixth (last) sentence after the rearrangement?

(a)  A

(b)  E

(c)  D

(d)  C

(e)  B

Answer: (b)

88. Which of the following should be the first sentence after the rearrangement?

(a)  A

(b)  B

(c)  C

(d)  D

(e)  E

Answer: (c)

89. Which of the following should be the fourth sentence after the rearrangement?

(a)  A

(b)  B

(c)  C

(d)  E

(e)  D

Answer: (e)

90. Which of the following should be the second sentence after the rearrangement?

(a)  A

(b)  B

(c)  C

(d)  D

(e)  F

Answer: (b)

Directions (Q. Nos. 91-100) In the given passage, there are blanks, each of which has been numbered. Against each, five words are suggested, one of which fits the blank appropriately. Find the appropriate word in each case.

(91) it innovation on steroids, innovation at warp speed or just the innovation of rapid innovation, the essential point remains. Technology is (92) innovation at its core, allowing companies to test new ideas at speeds and prices that were (93) even a decade ago. They can stick features, on websites and tell within hours how customers respond. They can see results from in-store promotions, or efforts to boost process productivity, almost as quickly. The (94)? Innovation initiatives that used to take months and megabucks to coordinate and launch can now be started in seconds, for peanuts.

And that makes innovation, the lifeblood of growth, more (95) and cheap. Companies are able to get a much better idea of how their customers behave and what they want. this gives new offerings and marketing efforts a better shot at success.

As a result, companies will also be (96) to try out new things because the price of failure is so much lower. That will bring bit changes in corporate culture making it easier to challenge accepted wisdom, for instance and forcing managers to give more employees a (97) in the innovation process.

There will be even better payoffs for customers, their likes and dislike will have much more (98) on companies’ decisions. In globally competitive markets, they will ultimately end up getting products and services better (99) to their needs. Already, this powerful new capability is (100) the way some of the biggest companies in the world do business, inspiring new strategies and revolutionizing the research-and-development process.

91.

(a)  Call

(b)  Says

(c)  Yell

(d)  Greet

(e)  Welcome

Answer: (a)

92.

(a)  curbing

(b)  transforming

(c)  letting

(d)  remaining

(e)  acting

Answer: (b)

93.

(a)  aware

(b)  fathom

(c)  unimaginable

(d)  their

(e)  curious

Answer: (c)

94.

(a)  pitfall

(b)  issue

(c)  cause

(d)  result

(e)  next

Answer: (d)

95.

(a)  fragile

(b)  dynamics

(c)  efficient

(d)  skilled

(e)  inept

Answer: (c)

96.

(a)  prone

(b)  willing

(c)  responsibly

(d)  foreword

(e)  volunteer

Answer: (b)

97.

(a)  talk

(b)  assert

(c)  say

(d)  faith

(e)  sound

Answer: (c)

98.

(a)  affect

(b)  crush

(c)  impress

(d)  attitude

(e)  impact

Answer: (e)

99.

(a)  specifically

(b)  tailored

(c)  comfortable

(d)  order

(e)  design

Answer: (b)

100.

(a)  changing

(b)  certain

(c)  paving

(d)  working

(e)  qualifying

Answer: (a)

Part II Quantitative Aptitude

Directions (Q. Nos. 101-105) What approximate value will come in place of question mark in the given question?

(You are not expected to calculate the exact value.)

101. 012 × 651/3 × 25.992 ÷ (211 × 12.972) = 2?

(a)  4

(b)  5

(c)  2

(d)  3

(e)  6

Answer: (b)

102. 

(a)  100

(b)  10

(c)  3

(d)  30

(e)  60

Answer: (d)

103. 

(a)  3

(b)  9

(c)  30

(d)  90

(e)  80

Answer: (a)

104. (24.99% of 399.995) ÷? = (125% of 4.111)2

(a)  80

(b)  4

(c)  60

(d)  16

(e)  40

Answer: (b)

105. √? = (1248.28 + 51.7) ÷9 ÷ 7.98

(a)  49

(b)  81

(c)  64

(d)  16

(e)  25

Answer: (e)

106. The distance between two places A and B is 110 km. 1st car departs from place A to B, at a speed of 40 km/h at 11 a, 2nd car departs from place B to A at a speed of 50 km/h at 1 pm. At what time will both the cars meet each other?

(a)  1 : 50 pm

(b)  1 : 20 pm

(c)  2 : 00 pm

(d)  2 : 30 pm

(e)  2 : 15 pm

Answer: (b)

Directions (Q. Nos. 107-112) Refer to the  pie-chart and answer the given questions.

107. In 2014, the number of female employees in department C was 5/13 of the total number of employees in same department. If the number of female employees in department F was 4 less than that in department C, what is the number of male employees department F?

(a)  41

(b)  42

(c)  58

(d)  54

(e)  48

Answer: (c)

108. In 2014, there were 25% post graduate employees department B. In 2015, 22 employees of the same department were shifted to Branch ‘PQR’. If in 2015, the percentage of post graduate employees in Department B became 28%, how many post graduate employees were shifted to branch ‘PQR’?

(a)  8

(b)  12

(c)  6

(d)  4

(e)  14

Answer: (d)

109. What is the average number of employees in departments A, D and F?

(a)  65

(b)  70

(c)  75

(d)  72

(e)  69

Answer: (e)

110. In department E, the respective ratio between the number of female employees and male employees was 5 : 4. There were equal number of unmarried males and unmarried females in department E. If the respective ratio between married females and married males was 3 : 2, what is the number of unmarried females?

(a)  6

(b)  15

(c)  12

(d)  4

(e)  8

Answer: (c)

111. What is the central angle corresponding to the number of employees in department E?

(a)  43.2°

(b)  46.5°

(c)  41.6°

(d)  42.8°

(e)  45.9°

Answer: (a)

112. The number of employees in department E is what percent less than the number of employees in departments A, B and C together?

(a)  72%

(b)  60%

(c)  65%

(d)  70%

(e)  68%

Answer: (d)

113. In the year 2013, the population of a village A was 20% more than the population of village B. The population of village A in 2014 increased by 10% as compared to the previous year. If the population of village A in 2014 was 5610, what was the population of village B in 2013?

(a)  4650

(b)  5550

(c)  4250

(d)  5800

(e)  4500

Answer: (c)

114. Five years ago, the respective ratio between the age of Opi and that of Mini was 5 : 3. Nikki is 5 years younger to Opi. Nikki is five years older to Mini. What is the Nikki’s present age?

(a)  35 years

(b)  25 years

(c)  20 years

(d)  10 years

(e)  30 years

Answer: (b)

115. Mohan gave 25% of a certain amount of money to Ram. From the money Ram received, he spent 20% on buying books and 35% on buying a watch. After the mentioned expenses, Ram has Rs 2700 remaining. How much did Mohan have initially?

(a)  Rs 16000

(b)  Rs 15000

(c)  Rs 24000

(d)  Rs 27000

(e)  Rs 20000

Answer: (c)

Directions (Q. Nos. 116-121) What will come in place of question mark in the given number series?

116. 849    282   93      30      9        ?

(a)  1

(b)  3

(c)  4

(d)  6

(e)  2

Answer: (e)

117. 18    20   14      26      6        ?

(a)  18

(b)  26

(c)  32

(d)  28

(e)  36

Answer: (c)

118. 12    13   20      39      82      ?

(a)  259

(b)  232

(c)  210

(d)  198

(e)  173

Answer: (e)

119. 18   8   20.4   23.6   30      ?

(a)  44.4

(b)  43.5

(c)  49.2

(d)  49.6

(e)  42.8

Answer: (e)

120. 17    9   10      5   35      ?

(a)  85

(b)  70

(c)  92.5

(d)  90

(e)  84.5

Answer: (d)

121. 6   4   5        11      ?        189

(a)  82

(b)  39

(c)  44

(d)  65

(e)  96

Answer: (b)

122. 16 men and 10 women together can complete a project in 10 days. If 12 women can complete the project in 25 days, in how many days 10 men complete the same project?

(a)  28 days

(b)  24 days

(c)  18 days

(d)  26 days

(e)  10 days

Answer: (c)

123. A and B started a business with an investment of Rs 2800 and Rs 5400 respectively. After 4 months, C joined with Rs 4800. If the difference between C’s share and A’s share in the annual profit was Rs 400, what was the total annual profit?

(a)  Rs 13110

(b)  Rs 12540

(c)  Rs 17100

(d)  Rs 11400

(e)  Rs 14250

Answer: (d)

124. The interest earned when Rs ‘P’ is invested for five years in a scheme offering 12% p.a. simple interest is more than the interest earned when the same sum (Rs P) is invested for two years in another scheme offering 8% p.a. simple interest, by Rs 1100. What is the value of P?

(a)  Rs 2500

(b)  Rs 2000

(c)  Rs 4000

(d)  Rs 3500

(e)  Rs 3000

Answer: (a)

Directions (Q. Nos. 125-130) Refer to the graph and answer the given questions.

125. What is the average number of tourists in city B in February and March?

(a)  18000

(b)  23000

(c)  19000

(d)  21000

(e)  17500

Answer: (c)

126. Number of tourists in city B in April are what percent more than that ini city A in March?

(a)  20%

(b)  18%

(c)  12%

(d)  25%

(e)  8%

Answer: (c)

127. What is the respective ratio between the total number of tourists in city A in January and February together and that in city B in the same months together?

(a)  25 : 11

(b)  16 : 9

(c)  30 : 17

(d)  16 : 11

(e)  31 : 18

Answer: (e)

128. In the month of June that year, the total number of tourists in both the cities together reduced by 40% from the previous month. What was the number of tourists in both the cities together in June?

(a)  32000

(b)  40000

(c)  20000

(d)  42000

(e)  30000

Answer: (e)

129. What is the difference between the total number of tourists in cities A and B together in May and that in March?

(a)  10000

(b)  9000

(c)  7000

(d)  8000

(e)  9500

Answer: (b)

130. The number of tourists in city A in April is what percent less than that in the same city in January?

(a) 

(b)  20%

(c) 

(d)  30%

(e)  25%

Answer: (e)

Directions (Q. Nos. 131-136) Study the table and answer the given questions.

Data regarding number of books sold in either hard bounds or paperback editions and also the categories of books sold in Fiction and Non-Fiction category, by four different shops, in a particular month (January, 2014)

Total books sold = Number of hard bounds sold + Number of paperbacks sold

131. What is the respective ratio between the number of non-fictions sold by shop C and number of non-fictions sold by shop B?

(a)  10 : 2

(b)  5 : 2

(c)  5 : 3

(d)  7 : 3

(e)  7 : 4

Answer: (c)

132. In February, 2014, the number of paperback editions sold by shop D was 5% more than the same sold by the same shop in the previous month. The number of paperback editions sold in February, 2014 by shop D constituted 75% of the total number of books sold by shop D in February 2014. What was the total number of books sold in February 2014 by shop D?

(a)  1840

(b)  2040

(c)  1960

(d)  2080

(e)  2400

Answer: (c)

133. Number of non-fictions sold by shop C is what percent of number of non-fictions sold by shop D?

(a)  38%

(b) 

(c) 

(d)  60%

(e)  31%

Answer: (b)

134. Total number of books sold by shop is what percent more than sold by shop C?

(a)  50%

(b) 

(c) 

(d)  30%

(e)  20%

Answer: (e)

135. What is the average number of fictions sold by shops A and B?

(a)  475

(b)  470

(c)  495

(d)  480

(e)  490

Answer: (c)

136. Number of hardbound editions sold by shop B is what percent less than that sold by shop D?

(a)  30%

(b)  36.75%

(c)  25%

(d)  32.5%

(e)  40.25%

Answer: (d)

Directions (Q. Nos. 137-141) Each question consist of a question and two statements numbered I and II given below it. You have to decide whether the data given in statements are sufficient to answer the questions. Read both the statements and

Give answer

a. if the data in statement I alone are sufficient to answer the question, while the data in statement II alone are not sufficient to answer the question

b. if the data in statement II alone are sufficient to answer the question, while the data in statement I alone are not sufficient to answer the question

c. if the data either in statement I alone are sufficient or in statement II alone are sufficient to answer the question

d. if the data even in both statements together are not sufficient to answer the question

e. if the data in both statements together are necessary to answer the question

137. What is the area of the circular field?

I. Area o f the largest square that can be inscribed in the given circular field is 392 cm2.

II. Area of the smallest square in which the given circular fields can be inscribed is 784 cm2.

Answer: (a)

138. What was the initial quantity of mixture of juice and water?

I. Juice and water were in the ratio of 6 : 1 respectively in the mixture initially.

II. When 7 litres of mixture is taken out and 5 litres of water is added the ratio between juice and water becomes 8 : 3 respectively.

Answer: (e)

139. What is the curved surface area of the right circular cylinder?

I. Area of the base of the cylinder is 616 cm2.

II. Volume of the cylinder is 9240 cm2.

Answer: (e)

140. In how many days can ‘B’ alone complete the work?

I. A, B and C together can complete the work in days.

II. A and B together can complete the work in  B and C together can complete the work together in and  A and (together can complete the work in 

Answer: (b)

141. How much money did Ms. Malini receive retirement funds?

I. Out of the total money received Ms. Malini gave 25% to her husband and 10% to her daughter. Out of the remaining she invested 30% in Mutual Funds, 60% in Pension Fund Scheme and remaining Rs 260000 she spent on miscellaneous items.

II. Out of the total money received Ms. Malini invested 58.5% In various schemes, gave 35% of the total money received to her husband and daughter and remaining money she spent on miscellaneous items.

Answer: (a)

142. The speed of a boat in still water is 16 km/h and the speed of the current is 2 km/h. The distance travelled by the boat from point A to point B downstream is 12 km more than the distance covered by the same boat from point S to point upstream in the same time. How much time will the boat take to travel from C to B downstream?

(a)  3 h

(b)  2 h 30 min

(c)  3 h 20 min

(d)  2 h 20 min

(e)  2 h

Answer: (d)

Directions (Q. Nos. 143-148) In the following questions, two equations numbered I and II are given solve  both the equations and

Given answer

a. if x > y b. if x ≥ y

c. if x < y d. if x ≤ y

e. if x = y or the relationship cannot be established

143. I. x2 – 6x + 9 = 0 II. 4y2 – 15y + 14 = 0

Answer: (a)

144. I. 2x2 – 9x + 4 = 0 II. y2 – 14y + 40 = 0

Answer: (d)

145. I. 4x2 – 10x + 25 = 0 II. y2 – 9y + 20 = 0

Answer: (b)

146. I. 4x2 – 13x + 10 = 0 II. .2y2 – 15y+ 22 = 0

Answer: (d)

147. I. 30x2 + 17x + 2 = 0 II. 3y2 + 8y + 4 = 0

Answer: (c)

148. I. 6x2 + 11x + 3 = 0 20y2 + 9y + 1 = 0

Answer: (a)

149. A jar has 40 L milk. From the jar, 8 L of milk was taken out and replaced by an equal quantity of water. If 8 L of the newly formed mixture is taken out of the jar, what is the final quantity of milk left in the jar?

(a)  32.5 L

(b)  30 L

(c)  25.6 L

(d)  24.2 L

(e)  24 L

Answer: (c)

150. Two mobile phones were purchased at the same price. One was sold at a profit of 20% and second was sold at a price, which was Rs 1520 less than the price at which the first was sold. If the overall profit earned by selling both the mobile phones was 1%, what was the cost price of one mobile phone?

(a)  Rs 6000

(b)  Rs 5200

(c)  Rs 4800

(d)  Rs 4000

(e)  Rs 5000

Answer: (d)

Part IV Professional Knowledge

151. What are code generators?

(a)  CASE tools that enable the automatic generation of program and database definition code directly from the design documents diagrams, forms and reports stored in the repository.

(b)  CASE tools that support the circulation of graphical representations of various system elements such as process flow, data relationships, and program structures.

(c)  CASE tools that enable the easy production of user documentation in standard formats.

(d)  CASE tools that enable the easy production of technical documentation in standard formats.

(e)  CASE tools that support the production of systems forms and reports in order to prototype how systems will ‘look and feel’ to users.

Answer: (a)

152. The Web Layout View is available in, which Office 2013 application?

(a)  Excel

(b)  Word

(c)  Access

(d)  PowerPoint

(e)  Webmaker

Answer: (b)

153. ………….. produces printout of a data in user-defined manner

(a)  Query language

(b)  DML

(c)  Report generator

(d)  DCL

(e)  Metadata

Answer: (c)

154. Which security model is used in a peer-to-peer network?

(a)  Password-protected Shares

(b)  Access Control Lists

(c)  Share-level Security

(d)  User-level Security

(e)  Access Control Entries (ACEs)

Answer: (c)

155. A hierarchical data model combines records and fields that are

(a)  cross structure and relational structure

(b)  tree structure

(c)  logical manner

(d)  cross structure

(e)  relational structure

Answer: (b)

156. Which protocol is used for transferring data and information from one network to the other network?

(a)  Internet Protocol

(b)  Transmission Control Protocol

(c)  File Transfer Protocol

(d)  Hyper-Text Transfer Protocol

(e)  Ethernet Protocol

Answer: (c)

157. What does ios ate mean as an argument in of stream in C++?

(a)  Open file for read access only

(b)  Open file, create

(c)  Open file, but do not create

(d)  Open file, set the position to the end

(e)  Open file, for write access only

Answer: (d)

158. An object-oriented database does not depend upon ………. for interactions.

(a)  data dictionary

(b)  Oracle

(c)  SQL

(d)  index

(e)  DBMS

Answer: (c)

159. What is a brownout in an electrical supply system?

(a)  A slight elevated voltage lasting from seconds to minute.

(b)  Alternating power out, power on lasting a few minutes.

(c)  Has nothing to do with electricity.

(d)  Complete power out lasting a few minutes.

(e)  A slightly decreased voltage lasting from seconds to minutes or more.

Answer: (e)

160. Which common backstage view task creates a PDF file from an existing file?

(a)  Export

(b)  Account

(c)  Create

(d)  Options

(e)  Info

Answer: (a)

161. Which device operates at the internet layer of the TCP/IP model?

(a)  Switch and Hub

(b)  Router

(c)  Switch

(d)  PBX

(e)  Firewall

Answer: (b)

162. A coding scheme recognized by system software for representing organizational data best defines a

(a)  data type

(b)  data size

(c)  tuple

(d)  hyperlink

(e)  hashing algorithms

Answer: (a)

163. Which phase of the SDLC involves converting to the new system?

(a)  Systems design

(b)  Systems implementation

(c)  Systems analysis

(d)  Systems development

(e)  Other than those mentioned as options

Answer: (b)

164. What is the purpose of ARP?

(a)  To resolve know IP addresses to unknown physical addresses.

(b)  To resolve domain names to unknown IP addresses.

(c)  To resolve MAC addresses and NetBIOS names to IP addresses.

(d)  To resolve known MAC addresses to unknown IP addresses.

(e)  To resolve NetBIOS names to IP addresses.

Answer: (a)

165. In Java, communication between objects is done by

(a)  calling a method

(b)  creating a class hierarchy

(c)  defining a Java class

(d)  creating a class

(e)  running a program

Answer: (a)

166. Which of the following CASE tools generate reports that help you identify possible inconsistencies, redundancies and omissions in diagrams,, forms and reports?

(a)  Form generators

(b)  Report generators

(c)  Diagramming tools

(d)  Analysis tools

(e)  Documentation generators

Answer: (b)

167. An attribute whose value is unique across all occurrences

(a)  primary key

(b)  recursive key

(c)  join attribute

(d)  data marker

(e)  single-valued key

Answer: (a)

168. An IP protocol field of 0 x 06 indicates that IP is carrying what as its payload?

(a)  UDP or IGRP

(b)  UDP

(c)  ICMP

(d)  TCP

(e)  IGRP

Answer: (c)

169. A named set of table rows stored in a contiguous section of secondary memory best describes a

(a)  entity

(b)  collection

(c)  physical file

(d)  pointer

(e)  relation

Answer: (c)

170. Which of the following attributes makes TCP reliable?

(a)  Connectionless establishment

(b)  Low overhead and Null sessions

(c)  Null sessions

(d)  Connection establishment

(e)  Low overhead

Answer: (d)

171. Which shortcut will create a new folder in a Windows folder?

(a)  Ctrl + C

(b)  Ctrl + Shift + W

(c)  Ctrl + V

(d)  Ctrl + Alt

(e)  Ctrl + O

Answer: (b)

172. Which of the following is not a benefit of networks?

(a)  Increased data integrity

(b)  Sharing of peripheral devices

(c)  Sharing of programs and data

(d)  Access to databases

(e)  Better communications

Answer: (a)

173. Which type of software is best used with specific questions to verify hypotheses?

(a)  Data fusion tools

(b)  Intelligent agents

(c)  Data cleansing tools

(d)  Query and reporting tools

(e)  Multidimensional-analysis tools

Answer: (d)

174. Which method of database recovery involves undoing unwanted changes to the database, such as undoing a partially completed transaction interrupted by a power failure?

(a)  Rollforward

(b)  Mirroring

(c)  Reprocessing

(d)  Rollback

(e)  Other than those given as options

Answer: (d)

175. What is the default maximum number of processes that can exist in Linux (with a 32-bit platform)

(a)  1024

(b)  32768

(c)  Unlimited

(d)  4096

(e)  99999

Answer: (b)

176. The network database model uses

(a)  rows and fields

(b)  data elements

(c)  keys

(d)  records and sets

(e)  data dictionary

Answer: (d)

177. …………. is a communication medium where data are transformed into light pulses.

(a)  Twisted-pair cable

(b)  Optical fibre

(c)  Coaxial cable

(d)  Copper cable

(e)  Satellite

Answer: (b)

178. ……… is a computer with a web server that serves the pages for one or more websites.

(a)  Portal

(b)  Backbone

(c)  Business Service Provider

(d)  Host

(e)  Gateway

Answer: (d)

179. The database management software creates and reads the data dictionary to ascription what ………….. exist and checks to see if specific users have the proper access rights to view them.

(a)  data element definitions

(b)  schema objects

(c)  meta data

(d)  reference keys

(e)  metadata and data element definitions

Answer: (b)

180. The foreign key is not necessarily the ………. in its current table.

(a)  report generator

(b)  foreign key

(c)  primary key

(d)  data dictionary

(e)  schema objects

Answer: (c)

181. Hierarchical database do not use …………. as relational databases do or searching procedures.

(a)  indexes

(b)  primary key

(c)  database dictionary

(d)  rows and columns

(e)  foreign key

Answer: (a)

182. A/an ………. can shield electronic equipment from power spikes.

(a)  UPS

(b)  surge protector

(c)  firewall

(d)  encryption program

(e)  antivirus

Answer: (b)

183. If there are no functional dependencies between two (or more) non-primary key attributes, this describes

(a)  third normal form

(b)  fifth normal form

(c)  second normal form

(d)  first normal form

(e)  fourth normal form

Answer: (c)

184. …………… is a special field value, distinct from 0, blank or any other value, that indicates that the value for the field is missing or otherwise unknown.

(a)  BLOB value

(b)  Designated value

(c)  Null value

(d)  Hashed value

(e)  Non specific value

Answer: (c)

185. The concept of ‘zero administration’ is associated with

(a)  Desktop computers

(b)  Mini computers

(c)  PDAs and organizers

(d)  Portable computers

(e)  Network computers

Answer: (e)

186. Collecting personal information and effectively posing as another individual is a crime known as

(a)  Spoofing

(b)  Identify theft

(c)  Hacking

(d)  Personally theft

(e)  Spooling

Answer: (b)

187. A relational database model uses …………. to contain and organize information.

(a)  records and fields

(b)  image, audio, documents

(c)  columns and rows

(d)  foreign key

(e)  data dictionary

Answer: (c)

188. Which of the following forms is used specifically to identify the entity created when encapsulating data inside data l ink layer headers and trailers?

(a)  Packet

(b)  Segment

(c)  Data

(d)  Chunk

(e)  Frame

Answer: (e)

189. Which of the following is a valid class declaration in C++?

(a)  class B {}

(b)  pubic class A {}

(c)  Class B {} and public class A {}

(d)  Class A {int x:};

(e)  Object A {int x.};

Answer: (d)

190. …………. is a device that has been customized to perform few specialized computing tasks well with minimal effort.

(a)  Dedicate appliance

(b)  Information appliance

(c)  Shopping bot

(d)  Server appliance

(e)  Substandard device

Answer: (b)

191. A good example of an operating system that makes your computer function and controls the working environment is

(a)  Microsoft Account

(b)  File Explorer

(c)  Microsoft Office 2013

(d)  Windows 8

(e)  Task Manager

Answer: (d)

192. ………… are used to refers to Java methods or variables that belongs to other classes.

(a)  Simple names

(b)  Keywords

(c)  External names

(d)  Qualified names

(e)  Method calls

Answer: (b)

193. Which connectionless protocol is used for its low overhead and speed?

(a)  ARP

(b)  TCP & ARP

(c)  ICMP

(d)  UDP

(e)  TCP

Answer: (d)

194. Which spread spectrum technology does the BO2. 11b standard define for operation?

(a)  IR, FHSS and DSSS

(b)  DSSS

(c)  DSSS and FHSS

(d)  FHSS

(e)  IR

Answer: (b)

195. Which of the following ensures that only authorized users can use or apply specific software applications?

(a)  Servicelevel agreement

(b)  Access log

(c)  Dongle

(d)  Private key

(e)  Authorized program analysis report

Answer: (c)

196. …………….. is an attempt to make a machine or network resource unavailable to its intended users, such as to temporarily or indefinitely interrupt or suspend services of a host connected to the internet due to bombardment of fake traffic.

(a)  Cracking

(b)  A Trojan horse

(c)  A virus

(d)  A denial of service attack

(e)  A worm

Answer: (d)

197. Which of the following best supports communication availability, acting as a counter measure to the vulnerability of component failure?

(a)  Integrated corrective network controls

(b)  Simple component redundancy

(c)  High network throughput rate

(d)  Careful network monitoring with a dynamic real-time alerting system

(e)  Other than those given as options

Answer: (b)

198. Data transmission technologies vary. Which one does Ethernet use?

(a)  CSMA/ CS

(b)  CSMA/ CA or CSMA/ CS

(c)  CSMA/CD

(d)  CSMA/CD

(e)  CSMA/CA

Answer: (c)

199. Applications communicate with kernel by using

(a)  unit calls

(b)  shell script

(c)  system calls

(d)  shell

(e)  C programs

Answer: (c)

200. In which of the following office applications, can you name a file at the same time you create it?

(a)  Access

(b)  PowerPoint

(c)  Paint

(d)  Word

(e)  Excel

Answer: (a)

IBPS Agriculture Officers Examination-2016 Held on 29-1-2017 Reasoning Question Paper With Answer Key

IBPS Agriculture Officers Examination-2016 Held on 29-1-2017 Reasoning
IBPS Agriculture Officers Examination-2016 Held on 29-1-2017 Reasoning Question Paper With Answer Key

IBPS Agriculture Officers Examination-2016 Held on 29-1-2017

Reasoning

1. ‘People travelling in city X have to face heavy traffic during the peak hours which causes them delay and other such difficulties.’ – A survey.

Which of the following may be a reason for poor traffic conditions in City X?

1. The number of vehicles on the road of city X has increased sustainability with a growth in the purchasing power of the people of city X over a passage of time.

2. The scope of finding open spaces for parking has reduced remarkably over the last few years due to increased population of the city and this has led to unauthorized parking by the road sides.

3. The State Government has announced flexi timings policy for all government offices of the state.

(A)  Both 1 and 2

(B)  Both 2 and 3

(C)  Only 1

(D)  Both 1 and 3

(E)  All of these

Answer: (A)

Directions – (Q. 2 to 7) In this question, three statements followed by two conclusions numbered I and II have been given. You have to take the given statements to be true even if they seem to be at variance from the commonly known facts and then decide which of the given conclusions logically follows from the given statements disregarding commonly known facts.

2. Statements : All bedsheets are cupboards. Some cupboards are buckets. No bucket is a tap.

Conclusion :

I. All taps are cupboards.

II. No bedsheet is a bucket.

(A)  Neither conclusion I nor II is true

(B)  Only conclusion II is true

(C)  Either conclusion I or II is true

(D)  Both conclusion I and II are true

(E)  Only conclusion I is true

Answer: (A)

3. Statements : Some penguins are kangaroos. Some kangaroos are dolphins. All dolphins are elephants.

Conclusion :

I. At least some dolphins are penguins.

II. All kangaroos being penguins is a possibility.

(A)  Neither conclusion I nor II is true

(B)  Only conclusion II is true

(C)  Either conclusion I or II is true

(D)  Both conclusion I and II are true

(E)  Only conclusion I is true

Answer: (B)

4. Statements : Some plants are animals. Some animals are flowers. Some flowers are insects.

Conclusion :

I. All insect being plants is a possibility

II. Atleast some animals are insects.

(A)  Neither conclusion I nor II is true

(B)  Only conclusion II is true

(C)  Either conclusion I or II is true

(D)  Both conclusion I and II are true

(E)  Only conclusion I is true

Answer: (E)

5. Statements : All metals are gases. All gases are wood. All wood is fabric.

Conclusion :

I. Atleast some fabrics are metals.

II. No wood is a metal.

(A)  Neither conclusion I nor II is true

(B)  Only conclusion II is true

(C)  Either conclusion I or II is true

(D)  Both conclusion I and II are true

(E)  Only conclusion I is true

Answer: (E)

6. Statements : Some circles are squares. All squares are triangles. All triangles are hexagons.

Conclusion :

I. Atleast some circles are hexagons.

II. All hexagons being circles is a possibility.

(A)  Neither conclusion I nor II is true

(B)  Only conclusion II is true

(C)  Either conclusion I or II is true

(D)  Both conclusion I and II are true

(E)  Only conclusion I is true

Answer: (D)

7. Statements : All plants are moons. .Some moons are stars. All stars are sun.

Conclusion :

I. Some plants are definitely not stars.

II. All suns being moons is a possibility.

(A)  Neither conclusion I nor II is true

(B)  Only conclusion II is true

(C)  Either conclusion I or II is true

(D)  Both conclusion I and II are true

(E)  Only conclusion I is true

Answer: (E)

8. The question consists of a statement and two conclusions numbered I and II given below it. A conclusion is something which can be directly deduced from t he given information in light of all the given facts. You have to decide which of the given conclusion/s logically follows from the given statement and select the appropriate answer.

Statement : “ We have noticed that private hospitals charge huge amounts for maternity wards irrespective of whether operation procedure is required or not. As a result, we are proposing a cap on the amount that can be charged in such hospitals which will be win-win situation for both the patients as well as government hospitals, what are overcrowded most of the times.” Statement by Health minister of country Z.

I. With a cap on prices on private hospitals the number of people of country Z opting for private hospitals instead of government hospitals is expected to increase.

II. The maternity wards of government hospitals of country Z are likely to undergo atleast some cut in revenue with a cap on prices of private hospitals.

(A)  Neither I nor II can be concluded from the given statement.

(B)  Both I and II can be concluded from the given statement.

(C)  Either I or II can be concluded from the given statement.

(D)  Only II can be concluded from the given statement.

(E)  Only I can be concluded from the given statement.

Answer: (B)

9. Study the given information carefully and answer the given question.

Following are the observations of an experiment on ‘sleep and weight-gain’ conducted on 18 healthy adults (Age 35 to 45) and 18 healthy adolescents (Ages 7 to 17).

(a) Recurrent sleep deprivation in men increased the  preferences for high-calorie foods and their overall calorie intake.

(b) Women who sleep less than 6 hours a night or more than 10 hours were more likely to gain 11 pounds (5 kilograms) compared with women who slept 7-9 hours a night.

(c) Less than 8 hours sleep in adolescents results in fatigue and less physical activity and hence leading to weight gain.

Which of the following cannot be concluded from the given findings of the research?

I. Hyperactivity in adolescents is a result of more than 7-8 hours’ sleep a night.

II. The hormone that increases the desire for high-calorie foods is more active in men who don’t get adequate sleep.

III. Sleeping more than 8 hours a night can help a person reduce weight drastically.

IV. Inadequate sleep affects the body weight in both adolescents and adults.

(A)  All the given statements can be concluded from the given findings of the research.

(B)  Both II and IV

(C)  Both I and III

(D)  Only IV

(E)  Both I and IV

Answer: (A)

Directions – (Q. 10 to 14) Study the following information carefully to answer the given question.

    Ten persons from different companies viz. Samsung, Bata, Microsoft, Google, Apple, HCL, ITC, Reliance, Airtel and Vodafone are sitting in two parallel rows containing five  people each, in such a way that there is an equal distance between adjacent persons. In row 1-B, C, ,D, E and F are seated and all of them are facing south. In row-2 R, S, T, U and V are seated and all of them are facing north, therefore, in the given seating arrangement, each member seated in a row faces another member of the other row. (All the information given above does not the order of seating as in the final arrangement.)

• These people sit between R and the person from Apple. The person from Reliance is an immediate neighbour of the one who faces the person from Apple. V sits to the immediate left of the one who faces the person from Reliance.

• Only one person sits between V and T. The person from Bata sits second to the right of the one who faces T. F sits second to the left of the person from Google. The person from Google does not sit at an extreme end of the line.

• Only two people sit between F and D. The person from Samsung faces an immediate neighbour of D. U is an immediate neighbour of the person from Microsoft. V is not from Microsoft. B sits second to the left of C.

• The person from ITC is an immediate neighbour of the person from Vodafone. Neither V nor F is from ITC. The person from ITC faces the person from HCL.

10. F is related to ITC in the same ways as T related to HCL, based on the given arrangement. To who amongst the following is D related to following the same pattern ?

(A)  Microsoft

(B)  Samsung

(C)  Apple

(D)  Bata

(E)  Reliance

Answer: (D)

11. Which of the following is true regarding E ?

(A)  E is from ITC

(B)  E is an immediate neighbour of the person from Samsung

(C)  E sits at an extreme end of the line

(D)  The person from Airtel faces E

(E)  None of the given options is true

Answer: (A)

12. Who amongst the following sit at extreme end of the rows ?

(A)  The person from Apple and F

(B)  V, E

(C)  The person from Samsung and C

(D)  The person from HCL and Bata

(E)  R and the person from Reliance

Answer: (A)

13. Four of the following five are alike in a certain way based on the given arrangement and so form a group. Which is the one that does not belong to that group ?

(A)  R

(B)  V

(C)  C

(D)  F

(E)  B

Answer: (E)

14. Who amongst the following faces the person from Airtel ?

(A)  The person from Google

(B)  B

(C)  The person from reliance

(D)  E

(E)  The person from Bata

Answer: (C)

15. In this question are given two statements (I) and (II). These statements may be either independent causes or may be effects of independent causes or a common cause. One of those statements may be the effect of the other statement. ‘Read both the statements and decide which of the given answer choice correctly depicts the relationship between these two statements.

Statements :

I. Company ABC, a leading automobile company in country G has decided to merge all its subsidiary companies into the parent company last week.

II. Company XYZ, a subsidiary of automobile company ABC, has opened five new branches in country F in the previous financial year.

(A)  Both the statement I and II are effects of some common cause.

(B)  Both the statements I and (I) are independent causes.

(C)  Statement II is the cause and statement I is its effect.

(D)  Statement I is the cause and Statement II is its effect.

(E)  Both the statements I and II are effect independent causes.

Answer: (C)

Directions – (Q. 16 to 20) Study the given information carefully and answer the given question.

    When a word and number arrangement machine is given an input line of words and numbers, it arranges them following a particular rule. The following is an illustration of input and rearrangement (All the numbers are two digit numbers,.)

Input : 87 46 part 68 30  burn heat 93 got mend.

Step I 87 46 68 30 burn heat goat mend put 91.

Step II 46 68 30 burn heat goat put 91 mud 85.

Step III 46 30 burn great put 91 mud 85 hut 66.

Step IV 30 burn put 91 mud 85 hut 66 gut 44.

Step V put 81 mud 85 hut 66 gut 44 bun 28.

    Step V is the last step of the above arrangement as the intended output of arrangement is obtained. As per the rules followed in the given steps, find the-

Input : 61 neon 17 skin 28 four cart 86 42 here

16. What is the difference between the fifth element from the left end in step II and the fifth element from the right end in step IV?

(A)  2

(B)  5

(C)  31

(D)  17

(E)  9

Answer: (D)

17. In step IV, how many elements are there between ‘84’ and the third element from the right end?

(A)  One

(B)  None

(C)  Two

(D)  More than three

(E)  Three

Answer: (E)

18. Which of the following is the third to the left of the seventh element from the left end of step II ?

(A)  sun

(B)  cart

(C)  28

(D)  84

(E)  42

Answer: (B)

19. Which of the following represents the element that is fifth to the left of ‘nun’ in step III ?

(A)  35

(B)  here

(C)  28

(D)  cart

(E)  hue

Answer: (C)

20. ‘sun’ is related to ‘cart’ in step I in the same way as ‘hue’ is related to ‘84’ in step III. Following the same pattern to which element is ‘59’ related to in step V ?

(A)  40

(B)  35

(C)  cut

(D)  fur

(E)  sun

Answer: (E)

Directions – (Q. 21 to 24) In this question, relationship between different elements is shown in the statements. The statements are followed by conclusions. Study the conclusions based on the given statement and select the appropriate answer.

21. Statements : P ≤ L ≤ A = N > D ≥ I; K ≤ A ≤ Q

Conclusions : I. K ≤ P

II. I < Q

(A)  Neither conclusion I nor II is true

(B)  Only conclusion II is true

(C)  Either conclusion I or II is true

(D)  Both conclusion I and II are true

(E)  Only conclusion I is true

Answer: (B)

22. Statements : K < A ≤ U ≥ M > J; A > Y; R ≥ U

Conclusions : I. R > Y

II. J < R

(A)  Neither conclusion I nor II is true

(B)  Only conclusion II is true

(C)  Either conclusion I or II is true

(D)  Both conclusion I and II are true

(E)  Only conclusion I is true

Answer: (D)

23. Statements : K < A ≤ U ≥ M > J; A > Y; R ≥ U

Conclusions : I. M < Y

II. Y > K

(A)  Neither conclusion I nor II is true

(B)  Only conclusion II is true

(C)  Either conclusion I or II is true

(D)  Both conclusion I and II are true

(E)  Only conclusion I is true

Answer: (A)

24. Statements : P ≤ L ≤ A = N > D ≥ I; K ≤ A ≤ Q

Conclusions : I. Q  ≥ P

II. K ≤ D

(A)  Neither conclusion I nor II is true

(B)  Only conclusion II is true

(C)  Either conclusion I or II is true

(D)  Both conclusion I and II are true

(E)  Only conclusion I is true

Answer: (E)

Directions – (Q. 25 to 30) Study the given information carefully to answer the given question.

   Seven boxes – P, Q, R, S, T, U and V are kept one above the other, but not necessarily in the same order. Each box has a different number viz. 2, 5, 6, 7, 10, 11 and 15, but not necessarily in the same order.

Only two boxes are kept between R and S. Only four boxes are kept between R and box number 10. Box number 2 is kept immediately below V. V is kept at one of the positions below S. R’s number is not 2. Only three boxes are kept between Q and box number 11. Q is neither the topmost nor the lowermost box. The difference between Q and box immediately below it is more than nine. Q’s box number is not 5. Only two boxes are kept between box number 5 and U. The sum of box numbers of U and T is 8.

25. Four of the following five are alike in a certain way and hence form a group. Which of the following does not belong to the group ?

(A)  V-15

(B)  P-5

(C)  Q-6

(D)  R-7

(E)  S-10

Answer: (B)

26. What is the position of T in the given stack of boxes?

(A)  Fifth from the bottom

(B)  Fourth from the top

(C)  First from the top

(D)  Third from the bottom

(E)  Second from the bottom

Answer: (E)

27. Which of the following boxes is numbered 6 ?

(A)  R

(B)  S

(C)  U

(D)  Q

(E)  V

Answer: (C)

28. Which of the following boxes is kept immediately below T ?

(A)  Box number 11

(B)  Box number 7

(C)  S

(D)  Q

(E)  Box number 6

Answer: (A)

29. What is the number of box V ?

(A)  11

(B)  5

(C)  6

(D)  15

(E)  7

Answer: (E)

30. How many boxes are kept between P and box number 15 ?

(A)  Two

(B)  One

(C)  More than three

(D)  None

(E)  Three

Answer: (D)

31. This consists of information and two statements numbered I and II given below it. You have to decide which of the given statements weaken/s or strengthen/s the information and decide the appropriate answer.

Statement : India has not recorded a win ever in overseas to country A in the last 25 years. Within win in every other overseas tour, India’s record in country A has been particularly dismal.

I. Country Z which has a much stronger team then India, has won most of their matches in the overseas tour to Country A.

II. Due to diplomatic issues, India has toured country A only twice in the past 25 years as against eight an average to other countries.

(A)  Statement I is a neutral statement while statement II weakens the information.

(B)  Statement I weakens the information while statement II strengthens the information.

(C)  Both statement I and statement II weaken the information.

(D)  Both statement I and statement II strengthen the information.

(E)  Statement I strengthen the information while statement II weakens the information.

Answer: (E)

32. Statement : In country X, government has made the provision of day-care facility mandatory in all such organizations where total number of female employees is 250 or more. FL enterprise, therefore, did not have to provide a day-care as only 21% of its staff constitutes female employees.

Which of the following can  be inferred from the given statement ?

1. No male employees is allowed to avail the day-care facilities as per the mandate.

2. Number of male employees in FL enterprise is definitely more than 950.

3. Had there been 25 less male employees in FL enterprise, they would have had to provide for the day-care facility.

4. The total number of employees in FL enterprise is definitely less than 1250.

(A)  Only 3

(B)  Only 1

(C)  Only 2

(D)  Only 4

(E)  None of these

Answer: (B)

33. This questions consist of a statement followed by two courses of action numbered I and II given below it. A course of action is an administrative decision to be taken for improvement, follow up or further action in regard to the problem, policy etc. You have to assume everything in the statement to be true and then decide which of the suggested courses of action/s logically follow/s from the given statement.

Statement : In a competitive exam held across country ‘Banimia’, more than 8 lakhs candidates appeared. But, about 4,000 candidates were found to be using unfair means, 75% of which were from state K of the country.

Course of Action I : Government of ‘Banimia’ should improve inspection and invigilation in all test centres of State K for all competitive exams.

Course of Action II : From now on for any competitive exams in Banimia, any application from state K should not be acceptable.

(A)  Only II follows

(B)  Only I follows

(C)  Either I or II follows

(D)  Neither I nor II follows

(E)  Both I and II follows

Answer: (B)

Directions – (Q. 34 to 38) Study the following information to answer the question-

In a certain code language,

‘prepare to know economics’ is written as ‘rt mj sp ai’

‘economics is important today’ is written as ‘qv ai nl dm’

‘to know study hard’ is written as ‘zu r toy sp’

‘study is prepare medium’ is written as ‘mj oy tk nl’

(All does are two letter codes only)

34. What does the code ‘mj’ stand for in the given code language?

(A)  to

(B)  prepare

(C)  study

(D)  economics

(E)  today

Answer: (B)

35. What is the code for ‘medium economics’ in the given code language?

(A)  tk rd

(B)  ai tk

(C)  tk oy

(D)  ai nl

(E)  nl oy

Answer: (B)

36. Which of the following additional statements is required to definitely find the code of ‘today’ in the given code language ?

(A)  ‘and more imported work’ is written as ‘fg hd lh dm’

(B)  ‘important work complete today’ is written as ‘pe qv fg dm’

(C)  ‘prepare important study today’ is written as ‘qv oy mj dm’

(D)  ‘discuss well to know’ is written as ‘wx sp xm rt’

(E)  No additional statement is required to find the code

Answer: (A)

37. What may be the code for ‘hard study material’ in the given code language ?

(A)  gm rc st

(B)  zu oy qv

(C)  oy sp zu

(D)  ye st el

(E)  zu cr oy

Answer: (E)

38. If the code for ‘is distance to’ is ‘sp nl ba’ then what may code for ‘know distance only’ in the given code language?

(A)  rt ba sp

(B)  rt ez ba

(C)  ba ez sp

(D)  ba ye dm

(E)  rt ye sp

Answer: (B)

Directions – (Q. 39 to 43) Study the given information carefully to answer the given question-

     Seven people M, N, O, P, Q, R and S have an interview on seven different days of the same week, starting from Monday and ending on Sunday, but not necessarily in the same order. Each one of them also likes different subjects namely-Geography, Zoology, Sociology, Mathematics, Hindi, History and Accountancy, but not necessarily in the same order.

Only three  people have their interview between N and the one who likes Zoology. N has his interview on one of the days after Tuesday but before Saturday. Only one person has an interview between P and the on e who likes Zoology. Only two people have their interview between P and the one who likes Geography. The one who likes Sociology has an interview immediately before the one who likes Geography. The number of people having interview between P and the one who likes Sociology is the same as that of the number of people between Q and N. The one who likes Hindi has an interview after P but before Q. Only two people have their interview between S and the one who likes Hindi. The one who likes Accountancy has an interview immediately after M but not on Wednesday. R has an interview on one of the days before who likes Mathematics.

39. Four of the following five are alike in a certain way based on the given arrangement and hence form a group. Which of the following does not belong to the group ?

(A)  Mathematics-Saturday

(B)  Geography-Wednesday

(C)  Sociology-History

(D)  O-Hindi

(E)  Thursday-S

Answer: (D)

40. How many people have their interviews between S and P ?

(A)  None

(B)  Two

(C)  Three

(D)  More than three

(E)  One

Answer: (E)

41. Who has an interview immediately before M ?

(A)  The one who likes Geography

(B)  O

(C)  Q

(D)  R

(E)  The one who likes Sociology

Answer: (E)

42. How many people have their interviews after the one who likes Geography ?

(A)  Two

(B)  None

(C)  Three

(D)  One

(E)  More than three

Answer: (D)

43. Which of the following statements is NOT TRUE as per the given arrangement?

(A)  All the given statements are true

(B)  Only one person has an interview between O and R

(C)  M has an interview on Saturday

(D)  P likes Accountancy

(E)  The one who likes History has an interview on one of the days before N

Answer: (D)

44. T is the father of M and P. P is the only daughter of V. M is married to N. A and B are children of M. How is V related to B?

(A)  Grand-mother

(B)  Uncle

(C)  Aunt

(D)  Sister

(E)  Grand-father

Answer: (A)

Directions- (Q. 45 to 49) These questions consist of a question and two statements I and II given below it. You have to decide whether the data provided in the statements are sufficient to answer the question. Read both the statements and choose the appropriate option.

(A) The data in statement I alone are sufficient to answer the question while the data in statement II alone are not sufficient to answer the question.

(B) The data in statement II alone are sufficient to answer the question, while the data in statement I alone are not sufficient to answer the question.

(C) The data either in statement I alone or statement II alone are sufficient to answer the question.

(D) The data in both statement I and II together necessary to answer the question.

(E) The data even in both statements I and II together are not sufficient to answer the question.

45. How many people are standing in a straight line (Note : All are standing in a straight line facing north.)?

I. D stands fifth from the left end of the line. Only one person stands between D and X. S stands fourth from the right end of the line. X stands exactly in the centre of the line.

II. Y stands second from the left end of the line. R stands second to the right of Y. D is an immediate neighbour of R. Only four people stand between D and S. As many people stand to the right of S as to the left of R.

Answer: (D)

46. Among six people – M, N, O, P, Q and R standing around a circle, while some facing the centre, some facing outside (i.e., outside the centre) but not necessarily in the same order, what is the position of O with respect to R?

I. Only two people stand between P and Q. Q is an immediate neighbour of both N and O. Q faces the centre. R stands second to the left of Q. R is not an immediate neighbour of N.

II. O stands second to the right of P. P faces the centre. N stands second to the left of O. R is an immediate neighbour of P. N is an immediate neighbour of both Q and M. R and O face opposite, directions (i.e., if R faces the centre then O faces outside and vice-versa.)

Answer: (B)

47. In the building the ground floor is building one; first floor is numbered two and so on till the topmost floor is numbered five. Amongst five people-A, B, C, D and E, each living on a different floor but not necessarily in the same order. On which floor does E live ?

I. B lives on an even numbered floor. D lives immediately above B. Only one person lives between D and E. E does not live on the topmost floor.

II. C lives on an odd numbered floor. A lives either immediately above or immediately below C. Only one person lives between A and B. E lives on one of the floors below B.

Answer: (E)

48. Among people S, T, U, V, W and X, each having a different weight, who is the second heaviest ?

I. V is heavier than only one person. T is heavier than S but lighter than W. S is heavier than both U and X. U is heavier than V.

II. S is heavier than only three people. V is heavier than X but lighter than U. U is lighter than both W and T. T is not the heaviest.

Answer: (C)

49. How far and in which direction is point D from point H ?

I. Point V is the midpoint of the 10 m horizontal straight line formed by joining points U and T. Point U is to the west of point V. Point L is 8 m to the north of point U. Point D forms the midpoint of the straight line formed by joining Points L and U. Point H is 4 m to the north of point T.

II. Point K is 5 m to the east of point L. Point M forms the midpoint of the 8 m vertical straight line formed by joining points K and V. Point V is to the south of point K. Point D is 4 m to the south of point L. Point H is 5 m away from point M.

Answer: (A)

50. Which of the following expressions will be definitely false if the given expression ‘U > B = N ≥ T < C ≤ R ≤ E’ is the definitely true ?

(A)  T < E

(B)  T > U

(C)  E ≤ C

(D)  N < C

(E)  R > T

Answer: (B)

IBPS PO Management Trainee Main Examination Held on 25-10-2015 General Awareness Question Paper With Answer Key

IBPS PO Management Trainee Main Examination Held on 25-10-2015
IBPS PO Management Trainee Main Examination Held on 25-10-2015 General Awareness Question Paper With Answer Key

IBPS PO Management Trainee Main Examination

Held on 25-10-2015

General Awareness

1. Which of the following services is not provided by the post offices in India ?

(A)  Sale of stamps

(B)  Issuance of demand drafts

(C)  Life insurance cover

(D)  All the given options are services provided by post offices in India

(E)  Savings bank scheme

Answer: (B)

2. In a surprise move Central Bank of ……….. cuts its benchmark interest rate to (−) 0.1 per cent.

(A)  Japan

(B)  South Africa

(C)  China

(D)  USA

(E)  None of these

Answer: (A)

3. The name ‘Punjab’ is derived from Indo-Iranian words-‘Punj’ means ‘five’ and ‘aab’ means ‘waters’. Which of the following is not the one among those five is not the one among those five rivers?

(A)  Sutlej

(B)  Beas

(C)  Yamuna

(D)  Chenab

(E)  Jhelum

Answer: (C)

4. Maximum loan amount to individuals against security of shares can be-

(A)  20.00 lacs

(B)  15.00 lacs

(C)  25.00 lacs

(D)  50.00 lacs

(E)  Other than those given as options

Answer: (A)

5. Abhinav Bindra is a famous-

(A)  social worker

(B)  film star

(C)  journalist

(D)  sports personality

(E)  politician

Answer: (D)

6. ‘Renimmibhi’ is the currency of the-

(A)  People’s republic of China

(B)  People’s republic of Bulgaria

(C)  People’s republic of Brazil

(D)  People’s republic of Ethiopia

(E)  People’s republic of Bangladesh

Answer: (A)

7. Which of the following forms of securing a loan is the safest one ?

(A)  Lien

(B)  Assignment

(C)  Pledge

(D)  Hypothecation

(E)  Mortgage

Answer: (C)

8. Which of the following is not acceptable as address proof for opening a bank account by an individual ?

(A)  Letter from any recognized public authority

(B)  Domicile certificate

(C)  Electricity Bill

(D)  Other than those given as options

(E)  A next agreement indicating the address of the customer duly registered with State Government

Answer: (D)

9. Who amongst the following is the author of the book ‘The Heart of India’?

(A)  Mark Tully

(B)  Other than those given as options

(C)  Ruskin Bond

(D)  Meera Syal

(E)  Gita Mehra

Answer: (A)

10. Which of the following is the correct full form of ECS often used in banking ?

(A)  Electronic Clash System

(B)  Electronic Cash Service

(C)  Electronic Clearing Service

(D)  Electronic Charging System

(E)  None of these

Answer: (C)

11. A demand draft issued by a bank is valid for-

(A)  3 years

(B)  There is no time limit

(C)  3 months

(D)  12 months

(E)  6 months

Answer: (C)

12. Dronacharya award is an award presented by the Government of India for excellence in-

(A)  Cinema

(B)  Corporate Governance

(C)  Human Resource Development

(D)  Sports Coaching

(E)  Employee Weffare Activities

Answer: (D)

13. NRI Day is observed on which of the following days ?

(A)  9th February

(B)  9th March

(C)  9th January

(D)  19th February

(E)  19th March

Answer: (C)

14. In which of the following types of banking, there is a direct execution of transaction between a bank and its consumers ?

(A)  Retain Banking

(B)  Universal Banking

(C)  Virtual Banking

(D)  Unit Banking

(E)  Other than those given as options

Answer: (A)

15. Recently India had exported Metro Coaches to-

(A)  South Africa

(B)  Egypt

(C)  Iran

(D)  Indonesia

(E)  Australia

Answer: (E)

16. A new programme called ‘Resilience’ has been introduced in certain schools in Bihar. Which of the following statement is incorrect about it-

(A)  Resilience was conceptualized in the USA by Steve Levanthal of Corstone

(B)  It works with adolescents at psychological and social level

(C)  Pilot study regarding this programme was first conducted in Tamil Nadu

(D)  The new programme is called Girls First and Youth first

(E)  None of these

Answer: (C)

17. The abbreviation ‘EPOS’ stand for-

(A)  Electronic Payment of Sale

(B)  Electronic Point of Sale

(C)  Electronic Purchase of sale

(D)  Electronic Price of Sale

(E)  Electronic Platform of Sale

Answer: (B)

18. Which of the following bodies regulates the Regional Rural Banks ?

(A)  RBI

(B)  NABARD

(C)  Department of Rural Development

(D)  State Government

(E)  SIDBI

Answer: (B)

19. Stratospheric Observatory for Infrared Astronomy (SOFIA) the largest airborne observatory in the word which was operational in 2004 is operated by US and ………… space agencies.

(A)  German

(B)  French

(C)  Chinese

(D)  Russian

(E)  Indian

Answer: (A)

20. Deposits under Foreign Currency Non-Resident [FCNR(B)] scheme can be accepted for a minimum of-

(A)  15 days

(B)  3 months

(C)  6 months

(D)  1 year

(E)  7 days

Answer: (D)

21. The Head of the Reserve Bank of India is officially known as-

(A)  Managing Director (MD) of RBI

(B)  Chief Executive of RBI

(C)  President of RBI

(D)  Governor of RBI

(E)  Executive Director of RBI

Answer: (D)

22. India has different categories of commercial banks. Which of the following is not a commercial bank ?

(A)  Foreign Banks

(B)  Commodity Banks

(C)  Nationalized Banks

(D)  Private Banks

(E)  Cooperative Banks

Answer: (E)

23. In case failed ATM transaction if the amount is not credited to a customer’s account within 7 working days from the date of receipt of the complaint. Banks have to pay compensation at the rate of Rs ……….. per day.

(A)  150

(B)  200

(C)  100

(D)  10

(E)  50

Answer: (C)

24. The green revolution in India was the outcome of the efforts of who amongst the following ?

(A)  Dr. Ramesh Mohan

(B)  Dr. C. Rangarajan

(C)  Mr. K. V. Kamath

(D)  Dr. M. S. Swaminathan

(E)  Other than those given as options

Answer: (D)

25. Fiscal Policy is concerned with which of the following ?

(A)  Public Revenue and Expenditure

(B)  Issue of currency

(C)  Education for all

(D)  Population control

(E)  Export and Import

Answer: (A)

26. Accelerator is most closely related to-

(A)  Interest rate

(B)  Inventories

(C)  Idle capacity

(D)  Induced investment

(E)  None of these

Answer: (D)

27. Which one of the following is known as ‘Demat’ account ?

(A)  Account in which shares are held in electronic form

(B)  Other than those given as options

(C)  Account allowed to be operated by guardian of minor

(D)  Account operated by business correspondents in rural centres

(E)  Account opened with zero balance

Answer: (A)

28. The maximum amount that can be remitted through RTGS s-

(A)  Rs 1 crore

(B)  Rs 50,00 lacs

(C)  Rs 2.00 lac

(D)  Rs 10.00 lacs

(E)  No upper ceiling

Answer: (E)

29. Lionel Messi who was in the news recently for his contribution to football, plays from which country ?

(A)  Argentina

(B)  Germany

(C)  Spain

(D)  Brazil

(E)  Other than those given as options

Answer: (A)

30. Which of the following is the full form of PURA ?

(A)  Provision of Urban Amenities in Rural Areas

(B)  Programme and Urban Rural Association

(C)  Preferential and Uniform Re-enforcement Act

(D)  Perfect Urban Rural Association

(E)  Other than those given as options

Answer: (A)

31. Who explained the importance of time element in the price determination ?

(A)  Chamberlin

(B)  Stigler

(C)  Mr. Jaun Robinson

(D)  Marshall

(E)  None of these

Answer: (D)

32. The foreign exchange of India is kept with-

(A)  SBI

(B)  ECGC

(C)  RBI

(D)  NABARD

(E)  Other than those given as options

Answer: (C)

33. Balance in a current account is classified is-

(A)  Hybrid deposit

(B)  Term deposit

(C)  Demand deposit

(D)  Flexi deposit

(E)  Other than those given as options

Answer: (C)

34. ‘Neoguri’, which means ‘raccoon dog’ in Korean, is the name of-

(A)  a Korean Motorbike brand

(B)  a domestic animals from south-east Asia

(C)  a South Korean port

(D)  a wild animal found in North Korea

(E)  a Powerful Typhoon

Answer: (E)

35. Which is the largest state in India in terms of area ?

(A)  Uttar Pradesh

(B)  Rajasthan

(C)  Jammu & Kashmir

(D)  Madhya Pradesh

(E)  Pre-divided Andhra Pradesh

Answer: (B)

36. IFSC Code has how many digits ?

(A)  7

(B)  15

(C)  11

(D)  9

(E)  10

Answer: (C)

37. Which one of the following is not a monetary tool of RBI ?

(A)  Reserve Repo Rate

(B)  SLR

(C)  Bank Rate

(D)  Repo Rate

(E)  Other than those given as options

Answer: (E)

38. Banks borrow money from RBI at which of the following rates ?

(A)  Bank rate

(B)  SLR      

(C)  CRR

(D)  Repo Rate

(E)  Reverse Repo Rate

Answer: (D)

39. Banks in India are required to benchmark their lending rates with reference to which of the following rates ?

(A)  Reverse Repo Rate

(B)  Base Rate

(C)  Govt. of India 10 year bond rate

(D)  Repo Rate

(E)  Other than those given as options

Answer: (B)

40. The rate of which domestic currency can be converted into foreign currency and vice-versa is known as-

(A)  LIBOR

(B)  Base Rate

(C)  Repo Rate

(D)  Exchange Rate

(E)  Inter Bank Call Money Rate

Answer: (D)

IBPS PO Management Trainee Main Examination Held on 25-10-2015 Reasoning Question Paper With Answer Key

IBPS PO Management Trainee Main Examination Held on 25-10-2015
IBPS PO Management Trainee Main Examination Held on 25-10-2015 Reasoning Question Paper With Answer Key

IBPS PO Management Trainee Main Examination

Held on 25-10-2015

Reasoning

Directions – (Q. 1-3) In these questions, relationships between different elements is shown in the statement. These statements are followed by two conclusions. Read both the statement and select the appropriate answer.

(A) If only conclusion I is true

(B) If only conclusion II is true

(C) If either conclusion I or II is true

(D) If neither conclusion I nor II is true

(E) If both conclusions I and II are true

1. Statements :

V ≥ M = T > X; R < T ≥ S

Conclusions : I. R < V

II. S ≤ X

Answer: (A)

2. Statements :

P < E ≤ R > F; E ≥ M; R < T

Conclusions : I. T > M

II. F < M

Answer: (A)

3. Statements :

R = Q ≤ I ≥ M = E; I < Z

Conclusions : I. Q ≤E

II. M > Z

Answer: (D)

4. Statement : The ministry of aviation has ordered to procure 35 new aircrafts to be added to the existing fleet this year.

Which of the following most appropriately proves that the decision taken by the aviation minister is unrealistic and not based on scientific projections?

(A)  A recent survey showed that the existing aircrafts in the fleet fail to match the international standards and quality norms

(B)  These new aircrafts have better technology and greater number of seats as compared to ones already existing in the fleet

(C)  The neighbouring country which also procured aircrafts lacked enough number of trained pilots

(D)  As the number of passengers has declined significantly this year, the existing fleet is not being used to its full potential

(E)  As many as 12 pilots are not on flying duties for two straight weeks due to lack of sufficient aircrafts to fly on the routes

Answer: (A)

5. In a certain code, PARTICLE is written as USBQFMDJ and GENERATE is written as FOFHFUBS, how is DOCUMENT written in that code ?

(A)  VDEPUONE

(B)  VDPENFUQ

(C)  VDPEUOFN

(D)  VDMENOFQ

(E)  VDPUFNOC

Answer: (C)

Directions – (Q. 6) Study the information carefully and answer the given questions-

   L, M, N, O and P are five different poles, each of different length. O is not the third shortest pole. N is bigger than only P. L is shorter than only one pole. The size of the shortest poles is 7 ft and that of the second tallest pole is 13 ft.

6. According to the given arrangement, which of the following combinations of pole and length is correct ?

(A)  N – 14 ft

(B)  P – 5 ft

(C)  O – 12 ft

(D)  L – 13 ft

(E)  Other than those given as options

Answer: (D)

7. R is sister of M. M is brother of H. D is mother of K. K is brother of M. How is R related to D?

(A)  Daughter

(B)  Mother

(C)  Other than those given as options

(D)  Sister

(E)  Data Inadequate

Answer: (A)

8. Pointing to a woman, Mr. Suresh said, she is the daughter of my grandfather’s only daughter. How is Suresh related to the woman ?

(A)  Cousin

(B)  Brother

(C)  Other than those given as options

(D)  Uncle

(E)  Cannot be determined

Answer: (B)

Directions- (Q. 9 and 10) The questions consists of a question and two statements numbered I and II given  below it. You have to decide whether the data provided in the statements are sufficient to answer the question. Read both the statements and mark the appropriate answer.

(A) Only statement I is sufficient

(B) Only statement II is sufficient

(C) Both statements are sufficient

(D) Neither I nor II statement is sufficient

(E) None of the above

9. P, Q, R, S and T are sitting around a circle facing towards the centre of the circle. Who is to the immediate right of T ?

I. R, Q and S are in the same respective sequence to the immediate left of T.

II. P is between S and T.

Answer: (A)

10. How is ‘smoking’ written in a code language ?

I. ‘Thanks for not smoking’ is written as ‘be je we no’ in that code language.

II. ‘No smoking area’ is written as ‘no se do’ in that code language.

Answer: (C)

Directions- (Q. 11-15) Study the information and answer the given question-

   In a certain code language,

    ‘capital cities are crowded’ is written as ‘ju sh pie be’

     ‘crowded cities create chaos’ is written as sh be nt ro’

     ‘huge industries create capital’ is written as ‘db ju nt ka’

    ‘industries are huge chaos’ is written as ‘ka pi ro db’

 (All the codes are two letter codes)

11. What is the code for ‘create’ in the given code language ?

(A)  sh

(B)  db

(C)  pi

(D)  nt

(E)  ro

Answer: (D)

12. What does ‘ro’ stand for in the given code language ?

(A)  chaos

(B)  capital

(C)  huge

(D)  create

(E)  are

Answer: (A)

13. What does ‘ka’ stand for in the given code language ?

(A)  either ‘cities’ or ‘crowded’

(B)  cities

(C)  create

(D)  either ‘huge’ or ‘industries’

(E)  chaos

Answer: (D)

14. Which of the following possibly means ‘crowded metro cities’ in the given code language ?

(A)  sh be ju

(B)  sh be ka

(C)  sh ka nt

(D)  ka nt  pi

(E)  sh un be

Answer: (E)

15. What is the code for ‘capital’ in the given code language ?

(A)  pi

(B)  ju

(C)  sh

(D)  be

(E)  db

Answer: (B)

Directions – (Q. 16-19) Study the following information to answer the given questions-

(i) In a family of 6 persons, there are two couples.

(ii) The lawyer is the head of the family and has only two sons-Mukesh and Rakesh-both teachers.

(iii) Mrs. Reena and her mother-in-law both are lawyers.

(iv) Mukesh’s wife is a doctor and they have a son, Ajay.

16. Which of the following is definitely a couple ?

(A)  Lawyer-Teacher

(B)  Doctor-Lawyer

(C)  Teacher-Teacher

(D)  Cannot be determined

(E)  None of the above

Answer: (A)

17. What is the profession of Rakesh’s wife ?

(A)  Teacher

(B)  Doctor

(C)  Lawyer

(D)  Cannot be determined

(E)  None of the above

Answer: (C)

18. How many male members are there in the family ?

(A)  Two

(B)  Three

(C)  Four

(D)  Cannot be determined

(E)  None of the above

Answer: (B)

19. What is the profession of Ajay ?

(A)  Teacher

(B)  Lawyer

(C)  Doctor

(D)  Cannot be determined

(E)  None of the above

Answer: (D)

20. Statement : Company A has approached the government with a proposal to build road and other infrastructure for providing transport facilities in area X.

Which of the following could possibly lead the government to turn down the proposal forwarded by Company A ?

(A)  Residents of Area X and its adjoining areas have to walk for several kilometers to seek transport facilities

(B)  Company B which had undertaken similar projects earlier had failed to build good quality road in the area.

(C)  Although, Area X is in dire need of road and infrastructure facilities, any company which underwork it would not make huge profits in the long run

(D)  Area X is a land with very low economic productivity and negligible residents

(E)  Area X is the unofficial hub for transportation of agriculture goods from State A to State B.

Answer: (B)

21. How many such pairs of letters are there in the word REFRESHING each of which has as many letters between them (in both forward and backward directions) in the word as they have in the English alphabet ?

(A)  Three

(B)  One

(C)  Two

(D)  None

(E)  More than three

Answer: (A)

22. If in the word ISOLATE, all the consonants are replaced by the previous letter in the alphabet and all the vowels are replaced by the next letter and then all the letters are arranged alphabetically, which letter will be third from the right end ?

(A)  Q

(B)  P

(C)  Other than those given as options

(D)  N

(E)  B

Answer: (B)

Directions – (Q. 23-26) A word and number arrangement machine when given an input line of words and numbers rearranges them following a particular rule in each step. The following is an illustration of input of rearrangement. (All the numbers are two digit numbers).

Input : talk 47 12 rise at 99 75 32 wise joke high 28 56 be

Step I : 12 talk 47 rise 99 75 32 wise joke high 28 56 be at

Step II : 28 12 talk 47 rise 99 75 32 wise joke high 56 at be

Step III : 32 28 12 talk 47 rise 99 75 wise joke 56 at be high

Step IV : 47 32 28 12 talk rise 99 75 wise 56 at be high joke

Step V : 56 47 32 28 12 talk 99 75 wise at be high joke rise

Step VI : 75 56 47 32 28 12 99 wise at be high joke rise talk

Step VII : 99 75 56 47 32 28 12 at be high joke rise talk wise

   Step VII is last step of the above rearrangement as the desired arrangement is obtained.

   Input : 83 why sat 14 32 no be ink feet 50 27 vain 67 92

23. Which step number is the following output?

32 27 14 83 why sat no 50 vain 6792 be feet ink

(A)  Step V

(B)  Step VI

(C)  Step IV

(D)  Step I

(E)  Other than those given as options

Answer: (E)

24. Which word/number would be at fifth position from the right is Step V?

(A)  14

(B)  92

(C)  feet

(D)  be

(E)  sat

Answer: (D)

25. How many elements (words or numbers) are there between ‘feet’ and ‘32’ as they appear in the last step of the output?

(A)  One

(B)  Three

(C)  Four

(D)  Five

(E)  Two

Answer: (B)

26. Which of the following represents the position of ‘why’ in the fourth step ?

(A)  Eighth from the left

(B)  Fifth from the right

(C)  Sixth from the left

(D)  Fifth from the left

(E)  None of the above

Answer: (C)

27. If Neena says, “Anita’s father Raman is the only son of my father-in-law Mahipal,” then how is Bindu, who is the sister of Anita, related to Mahipal

(A)  Niece

(B)  Daughter

(C)  Daughter-in-law

(D)  Cousin

(E)  None of the above

Answer: (E)

28. If in the number 38564927, first all the even digits are arranged in ascending order and then all the odd digits are arranged in ascending order which digit will be fourth from the right end?

(A)  3

(B)  5

(C)  4

(D)  Other than those given options

(E)  6

Answer: (A)

Directions – (Q. 29-33) Study the following information carefully and answer the questions-

   M, T, D, F, H, R and W are seven students studying in three different colleges I, II and III with at least two in each college. Each of them has a favourite subject from-English, History, Geography, Mathematics, Physics, Chemistry and Biology, not necessarily in the same order.

   D’s favourite subject is Physics and studies in College II with only M. H study in college I and he likes English. F studies in College III and does not like Mathematics. Those who like Geography and Chemistry study in the same college. W likes Biology and does not study in college I. R does not study with H. R does not like Chemistry. M does not like History.

29. What is M’s favourite subject ?

(A)  Mathematics

(B)  Chemistry

(C)  Data Inadequate

(D)  Geography

(E)  Other than those given as options

Answer: (A)

30. In which college T is studying ?

(A)  I

(B)  II or III

(C)  II

(D)  Data inadequate

(E)  III

Answer: (A)

31. Which of the following groups of students studies in college III ?

(A)  FTR

(B)  FWR

(C)  FM

(D)  Data Inadequate

(E)  Other than those given as options

Answer: (B)

32. What is T’s favourite subject ?

(A)  Chemistry

(B)  Biology

(C)  Mathematics

(D)  Data inadequate

(E)  Other than those given as options

Answer: (E)

33. Which of the following groups of students study in college I?

(A)  TR

(B)  HR

(C)  HT

(D)  HF

(E)  Other than those given as options

Answer: (C)

34. In a row of children facing north, Neeta is fifteenth from the left end of the row. If she is shifted towards the rightend of the row by four places, she becomes eighth from the right end. How many children are there in the row ?

(A)  24

(B)  28

(C)  Other than those given as options

(D)  27

(E)  26

Answer: (E)

35. Complete the series by replacing ?

AA40, CD51, GH64 ? US100

(A)  KK81

(B)  MM80

(C)  MM81

(D)  KK80

(E)  None of these

Answer: (C)

Directions-(Q. 36-40) Read the following information carefully and answer the given question-

    Seven people P, Q, R, S, T, U and V live on separate floors of a 7-floor building, but not necessarily in the same order. Ground floor is numbered 1; first floor is numbered 2 and so on till the topmost floor is numbered 7.

   P lives on floor number four. Only two people live between P and R. Q lives on the floor immediately below V. Q does not live on any of the floors below P. Only two people live between Q and S. Neither T nor S lives on the floor numbered two.

36. How many people live between the floors on which Q and U live ?

(A)  None

(B)  Four

(C)  Two

(D)  More than three

(E)  Three

Answer: (E)

37. Who lives on the floor immediately above P ?

(A)  S

(B)  Q

(C)  R

(D)  V

(E)  T

Answer: (E)

38. Who among the following lives on the ground floor ?

(A)  S

(B)  U

(C)  R

(D)  V

(E)  T

Answer: (C)

39. Which of the following options represents the people living immediately above and immediately below S respectively?

(A)  Q, R

(B)  U, T

(C)  T, P

(D)  P, R

(E)  P, U

Answer: (E)

40. If Q and S interchange their places, and so do P and R, then how many people would be there between S and P as per the new arrangement

(A)  More than four

(B)  Two

(C)  None

(D)  One

(E)  Four      

Answer: (E)

Directions – (Q. 41-44) Study the following information carefully and answer the given questions-

     In an international meet, representatives, A, B, C, D, E, F, G and H from eight different countries viz. Thailands, France, Holland, Austria, US, Spain, India and Germany (not necessarily in the same order) sit around a circular table facing the centre. A who represents Germany sits third to the left of E. F who is from India sits to the immediate right of A. D who is from Holland sits immediate neighbour of E. C who is from Spain sits exactly in the middle of people representing US and India. G is the representative from France sits second to the left of H who is from Thailand.

41. How many people sit between A and representative from Austria when counted in clockwise.

(A)  None

(B)  One

(C)  Two

(D)  Three

(E)  Four

Answer: (B)

42. Which of the following is true regarding representative F?

(A)  F is representative of Austria

(B)  F sits second to the left of the representative from Thailand

(C)  Representative from Germany is not an immediate neighbour of F

(D)  E sits third to the right of F

(E)  None is true

Answer: (E)

43. Which of the following pairs represents the immediate neighbours of the representative from Holland ?

(A)  CG

(B)  BE

(C)  AH

(D)  HB

(E)  GE

Answer: (E)

44. Representative from which of the following countries is seated second to the left of Indian representative ?

(A)  Thailand

(B)  US

(C)  France

(D)  Spain

(E)  None of these

Answer: (A)

Directions – (Q. 45-48) In each question below are two statements followed by two conclusions numbered I and II. You have to take the given statements to be true even if they seem to be at variance commonly known facts and then decide which of the given conclusions logically follows from the given statements disregarding commonly known facts. Give answer-

(A) If only conclusion I follows

(B) If only conclusion II follows

(C) If either conclusion I or conclusion II follows

(D) If neither conclusion I nor conclusion II follows

(E) If both conclusions I and conclusion II follows

45. Statements : No meeting is an argument. All debates are arguments. Some debates are fights.

Conclusions :

I. No fight is a meetings.

II. Some fights are meeting.

Answer: (D)

46. Statements : All hands are limbs. All limbs are fingers. Some fingers are thumbs.

Conclusions :

I. Some thumbs being limbs is a possibility

II. All hands are fingers.

Answer: (B)

47. Statements : All teams are participants. All members are teams. No member is a captain.

Conclusions :

I. Atleast some participants are members.

II. All teams being captains is a possibility.

Answer: (A)

48. Statements : Some slopes are mountains. No mountain is a river. Some rivers are ponds.

Conclusions :

I. All ponds being mountains is a possibility.

II. All slopes being rivers is a possibility.

Answer: (D)

Directions (Q. 49 and 50) J is the father of T. P is the brother of J. L is the mother of V. V is the brother of T. T is mother of S. T is the daughter-in-law-of W.

49. How is J related to S ?

(A)  Uncle

(B)  Brother

(C)  Grand-father

(D)  Cousin

(E)  Father

Answer: (C)

50. How is W related to P?

(A)  Son

(B)  Cannot be determined

(C)  Grandson

(D)  Aunt

(E)  Uncle

Answer: (B)

IBPS CWE-VI PO-MT Online Pre. Examination Held on October 23, 2016 Question Paper With Answer Key

IBPS CWE-VI PO-MT Online Pre. Examination Held on October 23, 2016
IBPS CWE-VI PO-MT Online Pre. Examination Held on October 23, 2016 Question Paper With Answer Key

IBPS CWE-VI PO/MT Online Pre. Examination

Held on October 23, 2016

Part I Reasoning

Directions (Q. Nos. 1-5) In the following questions, relationships between different elements is shown in the statements. The statements are followed by conclusions. Study the conclusions based on the given statements and select the appropriate answer.

Give answer

(a) if only conclusion II is true   (b) if only conclusion I is true

(c) if both conclusions are true   (c) if either conclusion I or II is true

(e) if neither conclusion I nor II is true

1. Statements S ≤ L ≤ I = P ≥ E > R; L > Q

Conclusions I. P ≥ S        II. I > R

Answer: (c)

2. Statements G > R ≤ E = A ≤ T ≤ S; D ≤ A ≤ J

Conclusions I. T ≥ D        II. R > S

Answer: (b)

3. Statements A ≥ B > C ≤ D ≤ E < F

Conclusions I. A ≥ E        II. C < F

Answer: (a)

4. Statements G > R ≥ E = A ≤ T ≤ S; D ≤ A ≤ J

Conclusions I. J > G        II. J = G

Answer: (e)

5. Statements S ≤ L ≤ I = P ≥ E > R: L > Q

Conclusions I. L < R        II. E ≥ Q

Answer: (e)

Directions (Q. Nos. 6-10) Study the following information carefully to answer questions based on it.

A, B, C, D, W, X, Y and Z are sitting is a circle. (But not necessary in the same order) there faces are on the centre. W is sitting third to the left of Y. The persons, who is from Dwarka is to the immediate right of W and W is not form Okhla. B is sitting fourth to the right of Z. Z is not the neighbour of Y. Neither B nor Z is an immediate neighbour of W. X is from Chanakyapuri and is sitting third to the right of the person from Dwarka. The person from Mehrauli is sitting second to the left of person from Chanakyapuri. The person from Rohini is sitting second to the left of W. A, who is from Lajpat Nagar, is sitting exactly between X and Z. The person from Saket is sitting second to the right of the person from Lajpat Nagar. C is sitting third to the left of X.

6. Who amongst the following persons belongs to Okhla ?

(a)   Y

(b)   D

(c)   C

(d)   B

(e)   Z

Answer: (a)

7. What is A’s position with respect to B ?

(a)   Third to the right

(b)   Second to the right

(c)   Third to the left

(d)   Second to the left

(e)   Fourth to the right

Answer: (a)

8. How many people are sitting between Z and C when counted in an anticlockwise direction From C?

(a)   One

(b)   Two

(c)   Three

(d)   Four

(e)   None

Answer: (d)

9. Four of the following five pairs are alike in a certain way based on their positions in the given arrangement and, so form a group. Which of the following does not belong to the group?

(a)   B-Rohini

(b)   Z-Mehrauli

(c)   D-Okhla

(d)   Y-Saket

(e)   X-Dwarka

Answer: (e)

10. Which of the following statements is false according to the mentioned arrangement?

(a)   C is to the immediate right of the person from Karol Bagh.

(b)   The person form Lajpat Nagar is third to the right of the person from Mehrauli.

(c)   The person from Dwarka is sitting exactly between the person from Karol Bagh and the person from Saket.      

(d)   D is neither from Chanakyapuri nor from Karol Bagh.

(e)   There are only three people between A and C.

Answer: (c)

(Q. Nos. 11-15) In each question below are given two/three statements followed two conclusions I and II. You have to take the given statements to be true even if they seem to be at variance with commonly known facts. Read all the conclusions and then decide, which of the given conclusions logically follows from the given statements, disregarding commonly known facts.

Give answer

(a) if only conclusion I follows

(b) if only conclusion II follows

(c) if either conclusion I or II follows

(d) if n either conclusion I nor II follows

(e) if both conclusions I and II follow

11. Statements All circles are triangles.

Some triangles are rectangles.

All rectangles are squares.

Conclusions I. All rectangles being triangles is a possibility.

II. All circles being square is a possibility.

Answer: (d)

12. Statements Some chairs are tables.

Some bed are tables.

No furniture is bed.

Conclusions I. All chairs being furniture is a  possibility.

II. Some tables are not bed is a possibility.

Answer: (b)

13. Statements All circles are triangles.

Some triangles are rectangles.

All rectangles are squares.

Conclusions I. Some triangles are not rectangles.

II. No square is a circle.

Answer: (e)

14. Statements All arts are theatre.

Some arts are drama.

Conclusions I. All drama being theatre is a possibility.

II. Some dramas are theatre.

Answer: (b)

15. Statements Some chairs are table.

Some bed are tables.

No furniture are bed.

Conclusions I. Some tables are not furniture.

II. All tables being furniture is a possibility.

Answer: (a)

(Q. Nos. 16-18) Read the given information carefully and answer the given questions.

Point B is 25 m South of point A. Point C is 10 m East of point B. Point D is 30 m North of point C. Point E is 7 m East of point D. Point X is 18 m South of point E. Point M is 12 m South of point X. Point C is 7 m West of point M.

16. B is in which direction from point D?

(a)   South

(b)   South-West

(c)   North-East

(d)   South-East

(e)   North

Answer: (b)

17. If point W is 3 m to the North of point A, then what is the distance between point B and point W?

(a)   28 m

(b)   15 m

(c)   22 m

(d)   24 m

(e)   17 m

Answer: (a)

18. What is distance between point B and point M?

(a)   17 m

(b)   15 m

(c)   21 m

(d)   19 m

(e)   13 m

Answer: (a)

(Q. Nos. 19-23) Study the following information to answer the given questions.

S, T, U, V, W, X, Y and Z are sitting in a straight line equidistant from each other (but not necessarily in the same order). Some of them are facing South while some are facing North.

S faces North. Only two people sit to the right of S. T sits third to the left of S. Only one person sits between T and X. X sits to t he immediate right of W. Only one persons sits between W and Z. Both the immediate neighbours of T face the same direction. U sits fourth to the left of X. T faces the opposite direction as S. Y does not sit at any of the extremes ends of the line. V faces the same direction as W. Both Y and U face the opposite direction of Z.

19. How many persons in the given arrangement are facing North?

(a)   More than four

(b)   Four

(c)   One

(d)   Three

(e)   Two

Answer: (d)

20. Four of the following five are alike in a certain way, and so form a group. Which of the following does not belong to the group?

(a)   W, X

(b)   Z, Y

(c)   T, S

(d)   T, Y

(e)   V, U

Answer: (a)

21. What is the position of X with respect to Z?

(a)   Second to the left

(b)   Third to the right

(c)   Third to the left

(d)   Fifth to the right

(e)   Second to the right

Answer: (b)

22. Who amongst the following sits exactly between Z and W?

(a)   T

(b)   Y

(c)   X

(d)   W

(e)   U

Answer: (a)

23. Who is sitting 2nd to the right of T?

(a)   Z

(b)   V

(c)   X

(d)   W

(e)   None of these

Answer: (c)

(Q. Nos. 24-26) Study the following information and answer the given questions.

D is daughter of N. E is wife of N. G is sister of D. C is married to G. N has no son. K is mother of E. Q is only daughter of C.

24. How Q is related to D?

(a)   Daughter

(b)   Cousin

(c)   Niece

(d)   Sister in law

(e)   Cannot be determined

Answer: (c)

25. How N is related to K?

(a)   Brother-in-law

(b)   Cousin

(c)   Son-in-law

(d)   Sister

(e)   Brother

Answer: (c)

26. How many daughters N have?

(a)   One

(b)   Three

(c)   Two

(d)   Cannot be determined

(e)   None of these

Answer: (c)

(Q. Nos. 27-29) Study the following information and answer the given question.

There are six wires in an extension box A, B, C, D, E and F they have different length, but not necessarily in the same order. E is greater than C, but less than D and B. A is greater than D and B. A is not longest wire. F is 13 cm long and E is 4 cm long.

27. If D is 5 cm less than F what would be the length of D?

(a)   7 cm

(b)   8 cm

(c)   9 cm

(d)   Can’t be determined

(e)   None of these

Answer: (b)

28. Which wire has least length?

(a)   B

(b)   A

(c)   C

(d)   E

(e)   None of these

Answer: (c)

29. If A is 10 cm length and B carry 5 cm length, then what would be the length of C?

(a)   6 cm

(b)   2 cm

(c)   7 cm

(d)   9 cm

(e)   None of these

Answer: (b)

(Q. Nos. 30-35) Study the given information carefully to answer the given questions.

M, N, O, P, Q, R and S are seven people live on seven different floors of a building but not necessarily in the same order. The lower most floor of the building is numbered 1, the one above that is numbered 2 and so on till t he topmost floor is numbered 7. Each one of them have different income i.e. Rs 3500, Rs 15000, Rs 7500, Rs 9000, Rs 11000, Rs 13500 and Rs 5000. (But not necessarily in t he same order.)M lives on an odd numbered floor, but not on the floor numbered 3.

The one, who has income of Rs 11000 lives immediately above M. Only two people live between M and the one who has income of 7500. The one. who has income of Rs 15000, lives on one of the odd numbered floors above P. Only three people live between O and the one who has income of  Rs 3500. The one who has income of 7500 lives immediately above O. The one who has income of Rs 3500 lives immediately above the one who has income of Rs 5000. S lives on an odd numbered floor. Only one person lives between N and Q. N lives on one of the floors above Q. Neither O nor M has income of Rs 9000. Q does not has income of Rs 7500.

30. How much income M has?

(a)   Rs 13500

(b)   Rs 5000

(c)   Rs 7500

(d)   Rs 15000

(e)   Rs 3500

Answer: (e)

31. Which of the following combinations is true with respect to the given arrangement?

(a)   Rs 3500-O

(b)   Rs 15000-R

(c)   Rs 5000-S

(d)   Rs 11000-P

(e)   Rs 9000-N

Answer: (b)

32. If all the people are made to sit in alphabetical order from top to bottom, the positions of how many people will remain unchanged?

(a)   Four

(b)   None

(c)   Two

(d)   One

(e)   Three

Answer: (d)

33. Which of the following statements is false with respect to the given arrangement?

(a)   The one who has income of 5000 lives immediately below M.

(b)   R has income of 15000.

(c)   None of the given options is true.

(d)   Only four people live between P and S.

(e)   S lives immediately below Q.

Answer: (e)

34. Who amongst the following lies on the floor numbered 2?

(a)   N

(b)   The one who has income of Rs 3500.

(c)   The one who has income Rs 5000.

(d)   P

(e)   R

Answer: (d)

35. How much income R has?

(a)   Rs 13500

(b)   Rs 5000

(c)   Rs 7500

(d)   Rs 15000

(e)   Rs 3500

Answer: (d)

Part II Quantitative Aptitude

(Q. Nos. 36-39) Study the table carefully and answer the given questions.

36. If the male population above poverty line for State R is 1.9 million, then, what is the total population of State R?

(a)   4.5 million

(b)   4.85 million

(c)   5.35 million

(d)   6.25 million

(e)   6 million

Answer: (d)

37. What will be the number of females above poverty line in the State S if it is known that the population of State S is 7 million?

(a)   3 million

(b)   2.13 million

(c)   1.33 million

(d)   5.7 million

(e)   4 million

Answer: (b)

38. What is the male population above poverty line for State P, if the female population below poverty line for State P is 2.1 million?

(a)   2.1 million

(b)   2.7 million

(c)   3.3 million

(d)   2.3 million

(e)   3 million

Answer: (c)

39. If the population of males below poverty line for State Q is 2.4 million and that for State T is 6 million, then what is the respective ratio of the total population of State Q and T?

(a)   1 : 3

(b)   2 : 5

(c)   3 : 7

(d)   4 : 9

(e)   3 : 9

Answer: (b)

40. Average runs of a cricketer in 10 innings is 60. How many runs should he make in 11th innings that his average runs would be 62?

(a)   80

(b)   81

(c)   82

(d)   83

(e)   84

Answer: (c)

41. Find the probability that a number from 1 to 300 is divisible by 3 or 7?

(a)   37/75

(b)   32/75

(c)   36/75

(d)   28/75

(e)   26/75

Answer: (b)

42. 14 men can do a work in 18 days, 15 women can do a work in 24 days. If 14 men work for first 3 days and 10 women work after that for 3 days, find the part of work left after that?

(a)   3/4

(b)   1/4

(c)   1/2

(d)   1/6

(e)   1/5

Answer: (a)

43. Perimeter of a rectangle is x and circumference of a circle is 8 more than the perimeter of the rectangle. Ratio of radius of circle and length of the rectangle is 1 : 2 and ratio of length and breadth of rectangle is 7 : 3. Find the length of the rectangle?

(a)   14

(b)   21

(c)   28

(d)   35

(e)   7

Answer: (c)

44. A invest on some scheme at 5% and B at 3% for two year. If the total sum invested by A and B is Rs 4000 and the simple interest received by both is same then find the amount invested by A?

(a)   Rs 1300

(b)   Rs 1500

(c)   Rs 2500

(d)   Rs 2700

(e)   Rs 2100

Answer: (b)

45. Two trains crosses each other in 14 seconds when they are moving in opposite direction, and when they are moving in same direction, they crosses each other in 3 minute 2 seconds. Find the speed of the faster train by what percent more than the speed of the slower train?

(a)   16.67%

(b)   17.33%

(c)   16.33%

(d)   17.67%

(e)   18.33%

Answer: (a)

(Q. Nos. 46-50) What will come in place of the question marks in the following number series?

46. 11  20  38  74  ?

(a)   146

(b)   154

(c)   128

(d)   132

(e)   136

Answer: (a)

47. 15  21  38  65  101  ?

(a)   124

(b)   145

(c)   136

(d)   158

(e)   162

Answer: (b)

48. 24  28  19  35  10  ?

(a)   26

(b)   36

(c)   16

(d)   46

(e)   15

Answer: (d)

49. 7  16  45  184  915  ?

(a)   2092

(b)   5496

(c)   1048

(d)   4038

(e)   3268

Answer: (b)

50. 12  19  35  59  90  ?

(a)   134

(b)   127

(c)   132

(d)   98

(e)   114

Answer: (b)

(Q. Nos. 51-55) Study the following graph and answer the questions that follow.

51. The incomes of company X and Y in 2010 were in the ratio of 3 : 4 respectively. What was the respective ratio of their expenditures in 2010?

(a)   7 : 22

(b)   14 : 19

(c)   15 : 22

(d)   27 : 35

(e)   35 : 49

Answer: (c)

52. If the expenditure of company Y in 2007 was Rs 220 crore, what was its income in 2007?

(a)   Rs 312 crore

(b)   Rs 297 crore

(c)   Rs 283 crore

(d)   Rs 275 crore

(e)   Rs 250 crore

Answer: (b)

53. If the expenditures of company X and Y in 2006 were equal and the total income of the total companies in 2006 was Rs 342 crore, what was the total profit of two companies together in 2006?

(Profit = Income – Expenditure)

(a)   Rs 240 crore

(b)   Rs 171 crore

(c)   Rs 120 crore

(d)   Rs 102 crore

(e)   Rs 150 crore

Answer: (d)

54. The expenditure of company X in the year 2008 was Rs 200 crore and the income of company X in 2008 was the same as its expenditure in 2011. What is the income of company X in 2011?

(a)   Rs 385 crore

(b)   Rs 465 crore

(c)   Rs 335 crore

(d)   Rs 295 crore

(e)   Rs 250 crore

Answer: (b)

55. If the incomes of two companies were equal in 2009, then what was the ratio of expenditure of company X to that of company Y in 2009?

(a)   6 : 5

(b)   5 : 6

(c)   11 : 6

(d)   16 : 15

(e)   20 : 11

Answer: (d)

(Q. Nos. 56-60) In each of these questions, two equations I and II are given. You have to solve both the equations and give answer

(a) if x > y     (b) if x ≥ y  (c) if x < y   (d) if x ≤ y

(e) if x = y or relationship between x and y cannot be established

56. I. 3x2 – 22x + 7 = 0 II. y2 – 15y + 56 = 0

Answer: (d)

57. I. 2x2 – 17x + 36 = 0 II. 2y2 – 19y +44 = 0

Answer: (d)

58. I. x – √169 = 0 II. y2 – 169 = 0

Answer: (e)

59. I. 3x2 + 20x + 25 = 0 II. 3y2 +14y +8 = 0

Answer: (c)

60. I. 3x2 + 5x + 2 = 0 II. 3y2 + 18y + 24 = 0

Answer: (a)

61. A seller mark the price 50% above the cost price and give 10% discount on an item. While selling, he cheats customer by giving 20% less in weight. Find his overall profit percent (approximate)?

(a)   26%

(b)   35%

(c)   68%

(d)   72%

(e)   76%

Answer: (b)

62. There are 81 L pure milk in a container. One-third of milk is replaced by water in the container. Again one-third of mixture is extracted and equal amount of water is added. What is the ratio of milk to water in the new mixture?

(a)   1 : 2

(b)   1 : 1

(c)   2 : 1

(d)   4 : 5

(e)   None of these

Answer: (d)

63. A is 2 years older than B while B is 3 year younger than C. The ratio of age of A,, 6 years hence and B, 2 years ago is 5 : 3. What was age of C, 6 years ago?

(a)   12 yr

(b)   19 yr

(c)   15 yr

(d)   14 yr

(e)   21 yr

Answer: (c)

64. A, B and C started a business with their investments in the ratio 1 : 2 : 4. After 6 month A invested the half amount more as before and B invested same the amount as before while C withdrew 1/4th of his investment. Find the ratio of their profits at the end of the year.

(a)   5 : 12 : 13

(b)   5 : 11 : 14

(c)   5 : 12 : 14

(d)   5 : 12 : 10

(e)   None of these

Answer: (e)

65. The average marks in Science subject of a class of 20 students is 68. If the marks of two students were misread as 48 and 65 of the actual marks 72 and 61 respectively, then what would be the correct average?

(a)   68.5

(b)   69

(c)   69.5

(d)   70

(e)   66

Answer: (b)

(Q. Nos. 66-70) What should come in the place of questions mark in the given questions?

66. 40% of 265 + 35% of 180 = 50% of ? + ?% of 80

(a)   80

(b)   95.5

(c)   130

(d)   125.5

(e)   115

Answer: (c)

67. 

(a)   31.4

(b)   28.6

(c)   27.6

(d)   24.2

(e)   22.4

Answer: (a)

68. √? + 416 = (60% of 920) – 110

(a)   576

(b)   676

(c)   784

(d)   1024

(e)   1156

Answer: (b)

69. (682% of 682) ÷ 856 = ?

(a)   4.50

(b)   10.65

(c)   2.55

(d)   8.75

(e)   6.25

Answer: (e)

70. 5% of 850 + 24.8% of 650 = ?

(a)   295

(b)   330

(c)   270

(d)   375

(e)   220

Answer: (a)

Part III English Language

(Q. Nos. 71-77) Read the following passage carefully and answer the questions given below it.

Governments looking for easy popularity have frequently been tempted into announcing give-a-ways of all sorts; free electricity, virtually free water, subsidized food, cloth at half price, and so on. The subsidy culture has gone to extremes. The richest farmers in the country get subsidized fertilizers, University education, typically accessed by the wealthier sections, is charged at fraction of cost. Postal services are subsidized, and so are railway services.

Bus fares cannot be raised to economical levels because there will be violent protest, so bus travel is subsidized too. In the past, price control on a variety of items, from steel to cement, meant that industrial consumer of these items got them at less than actual cost, while the losses of the public sector companies that produced them were borne by the taxpayer! A study done a few years ago, came to the conclusion that subsidizes in the Indian economy total as much as 14.5% of gross domestic product. At today’s level, that would work out to about Rs 150000 crore.

And who p ay the bill? The theory-and the political fiction on the basis of which it is sold to unsuspecting voters-is that subsidies go the poor, and are paid for by the rich. The fact is t hat most subsidies go the ‘rich’ (defined in the Indian context as those who are above the poverty line), and much of the tab goes indirectly to the poor. Because the hefty subsidy bill results in fiscal deficits, which in turn push up rates of inflation-which, as everyone knows, hits the poor the hardest of all. That is why taxmen call inflation the most regressive form of taxation.

The entire subsidy system is built on the thesis that people cannot help themselves, therefore governments must do so. That people cannot afford to pay for variety of goods and services, and therefore the government must step in. This thesis has been applied not just in the poor countries, but in the rich ones as well; hence the birth of the welfare State in the West, and an almost Utopian social security system; free medical care, food aid, old age security, et.al. But with the passage of time, most of the wealthy nations have discovered that their economies cannot sustain this social safety net, which in fact reduces the desire among people to pay their own way, and takes away some of the incentive to work, in short, the bill was unaffordable, and their societies were simply not willing to pay. To the regret of many, but because of the laws of economies are harsh, most Western societies have been busy pruning the welfare bill.

In India, the lessons of this experience over several decades, and in many countries-do not seem to have been learnt or they are simply ignored in the pursuit of immediate votes. People who are promised cheap food or clothing do not in most cases look beyond the gift horses-to the question of who picks up the tab. The uproar over higher petrol, diesel and cooking gas prices ignored this basic question; if the user of cooking gas does not want to pay for its cost, who should p ay? Diesel in the country is subsidized, and if the user of cooking gas does not want to pay for its full cost, who does he or she think should pay the balance of the cost” It is a simple question, nevertheless if remains unasked.

The Deva Gowda government has shown some courage in biting the bullet when it comes to the price of petroleum products. But it has been bitten by much bigger subsidy bug. It wants to offer food at half its cost to everyone below the poverty line, supposedly estimated at some 380 million people.

What will be the cost? And of course, who will pick up the tab? The Andhra Pradesh government has been bankrupted by selling rice as 2 per kg. Should the Central government be bankrupted too, before facing up to the question of what is affordable and what is not?

Already, India is perennially short of power because the subsidy on electricity has bankrupted most electricity boards, and made private investment wary unless it gets all manner of State guarantees. Delhi’s subsidized bus fares have bankrupted the Delhi Transport Corporation, whose buses have slowly disappeared from the capital’s streets.

It is easy to be soft and sentimental, by looking at  programmes that will be popular. After all, who does’ not like a free lunch? But the evidence is surely mounting that the lunch isn’t free at all. Somebody is paying the bill. And if you want to know who, take at the country’s poor economic performance over the years.

71. Which of the following should not be subsidized over the years?

(a)   University education

(b)   Postal services

(c)   Steel

(d)   Other than those given as options

(e)   All of the above options

Answer: (d)

72. The statement that ‘subsidies are paid by the rich and go to the poor’ is

(a)   fiction

(b)   fact

(c)   fact, according to the author

(d)   fiction, according to the author

(e)   Other than those given as options

Answer: (d)

73. Why do you think that the author calls the western social security system Utopian?

(a)   The countries belief in the efficacy of the system was bound to turn out to be false.

(b)   The system followed by these countries is the best available in the present context.

(c)   Everything under this system was supposed to be free, but people were changing money for them.

(d)   The theory of system followed by these countries was devised  by Dr. Utopia.

(e)   All the options are responsible.

Answer: (c)

74. It can be inferred from the passage that the author

(a)   believes that people can help themselves and do not need the government

(b)   believes that the theory of helping with subsidy is very destructive

(c)   believes in democracy and free speech.

(d)   is not a successful politician.

(e)   believes that subsidies are the best way to help poor.

Answer: (b)

75. Which of the following is not a victim of extreme subsidies?

(a)   The poor

(b)   The Delhi-Transport Corporation

(c)   The Andhra Pradesh government

(d)   Other than those given as options

(e)   The rich

Answer: (e)

76. Which of the following is not true in the context of the passage ?

(a)   Where subsides are concerned, the poor  ultimately pay the tab.

(b)   Inflation is caused by too much subsidies.

(c)   Experts call subsidies the most regressive form of taxation.

(d)   Fiscal deficits are caused due to heavy subsidy bills.

(e)   None of the following is true in the context of the passage.

Answer: (a)

77. A suitable title to the passage would be:

(a)   There’s No Such Things as a Free Lunch

(b)   The Economic Overview

(c)   Deva Gowda’s Government and its Follies

(d)   I takes Two to Tango

(e)   The Rich and The Poor: Extreme Partiality

Answer: (a)

(Q. Nos. 78-82) Rearrange the following six sentences A, B, C, D, E and F in the proper sequence to form a meaningful  paragraph and then answer the questions given below.

(A) It is the only country in the world that is carbon negative, which means, it produces more oxygen that it consumes.

(B) Bhutan, sandwiched between the two most populous nations on Earth, suffers for their sins.

(C) So far, so good. But then, two things happened.

(D) Carbon sinks, 70% forest cover, powered almost entirely by mountain streams-Bhutan is a poster child for green living.

(E) Glaciers are beginning to melt, flash floods and heavy rains-and even droughts-are common, and temperatures are climbing.

(F) One, India and China got richer.

78. Which of the following should be the FIRST sentence of the given paragraph?

(a)   E

(b)   D

(c)   C

(d)   B

(e)   A

Answer: (b)

79. Which of the following should be the THIRD sentence of the given paragraph?

(a)   A

(b)   B

(c)   C

(d)   D

(e)   E

Answer: (c)

80. Which of the following should be the LAST sentence of the given paragraph?

(a)   A

(b)   D

(c)   C

(d)   B

(e)   E

Answer: (e)

81. Which of the following should be the FOURTH sentence of the given paragraph?

(a)   F

(b)   C

(c)   B

(d)   E

(e)   D

Answer: (a)

82. Which of the following should be the SECOND sentence of the given paragraph?

(a)   B

(b)   D

(c)   A

(d)   C

(e)   E

Answer: (c)

(Q. Nos. 83-90) In the following passage, you have a brief passage. IN the following passage, some of the words have been left out. First read the passage over and try to understand what it is about. Then fill in the blanks with the help of the alternatives given.

Big ideas come from tackling (83) problems. When one is confronted with an overwhelming task, it’s pieces. Business jargon is full of phrases about that, like “pilot projects” and “low-hanging fruit.” They have their place, but in the repertory of management (84), they should share their place with bold approaches to bit challenges. Much of today’s most valuable management knowledge came from wrestling with such issues.

The most complicated workplace in the middle of the last century was the automobile assembly plant. Drawn to its complexity where Peter F. Drucker, W.

Edwards Deming, and Taiichi Ohno, among others, the work they and their disciples did,, applied in industry after industry, is the basis of the best that we know about operations, managing people, innovation, organizational design, and much more. The most complex workplaces are tertiary care hospitals. These vast (85) employ tens of thousands of people who, under one roof, do everything from neurosurgery to laundry. Each patient – that is to say, each “job” – calls on a different set of people with a different constellation of (86); even when the two patients have the same diagnosis, success may be (87) differently.

This is complexity of an order of magnitude greater than automobile assembly, and anyone, who (88) hospitalized knows that management has thus far been unequal to the scope of task. The workers, managers, consultants, and scholars (89) crack this nut will reshape industries and institutions just as (90) as Drucker, Deming, and Ohno did.

83.

(a)   small

(b)   big

(c)   irrelevant

(d)   buildings

(e)   minor

Answer: (b)

84.

(a)   weakness

(b)   strength

(c)   power

(d)   practice

(e)   symptom

Answer: (b)

85.

(a)   houses

(b)   institute

(c)   demagogue

(d)   forts

(e)   enterprises

Answer: (e)

86.

(a)   barbarity

(b)   talent

(c)   skills

(d)   unskilled

(e)   barbaric

Answer: (c)

87.

(a)   managed

(b)   officious

(c)   delivered

(d)   measured

(e)   postponed

Answer: (d)

88.

(a)   are been

(b)   have being

(c)   have been

(d)   has been

(e)   is be

Answer: (d)

89.

(a)   who

(b)   whom

(c)   whose

(d)   which

(e)   whomsoever

Answer: (a)

90.

(a)   profoundly

(b)   gradually

(c)   superficially

(d)   speciously

(e)   earnest

Answer: ()

(Q. Nos. 91-100) Identify the error in the sentences given below, if there is no error, choose option (e).

91.

(a)   The need to set-up

(b)   a good library in the locality

(c)   has been in the minds of people

(d)   for some time now

(e)   No error

Answer: (a)

92.

(a)   Most people would  have

(b)   attended the union meeting

(c)   if they had

(d)   had longer notice of it

(e)   No error

Answer: (d)

93.

(a)   He took to

(b)   reading times

(c)   for better knowledge

(d)   of the facts

(e)   No error

Answer: (c)

94.

(a)   When children have difficulty understanding

(b)   a certain mathematical process, it is often because

(c)   their teachers to do not understand it conceptually

(d)   themselves and do not present it in a way that children can understand

(e)   No error

Answer: (e)

95.

(a)   Studies show that the lives of millions of mothers

(b)   and their children could be saved if countries would

(c)   invest in programmes that ensures a healthy pregnancy.

(d)   and safe childbirth

(e)   No error

Answer: (b)

96.

(a)   Film viewers claim that

(b)   the number of scenes depicting alcohol consumption

(c)   have increased dramatically over

(d)   the last decade

(e)   No error

Answer: (c)

97.

(a)   Forty percent of the people alive today have

(b)   never made a phone call, but

(c)   thirty percent still have no electricity connections

(d)   to their homes

(e)   No error

Answer: (b)

98.

(a)   Workers with less

(b)   personal problems are

(c)   likely to be

(d)   more productive in their work

(e)   No error

Answer: (a)

99.

(a)   Everyone who visits Singapore

(b)   is impressed by its cleanliness

(c)   which is mainly a result of rigorous implementation

(d)   of their strict laws

(e)   No error

Answer: (d)

100.

(a)   The bridal dress was

(b)   most unique: the prince

(c)   designed it and his

(d)   mother provided the lace fabric

(e)   No error

Answer: (b)

IBPS CWE-VI Bank Clerk Main Online Examination Held on January 1, 2017 Question Paper with Answer Key

IBPS CWE-VI Bank Clerk Main Online Examination 2017 Held On January 1, 2017
IBPS CWE-VI Bank Clerk Main Online Examination Held on January 1, 2017 Question Paper with Answer Key

IBPS CWE-VI Bank Clerk Main Online Examination 2017

Held On January 1, 2017

 

1. MMID is a seven-digit number issued by banks for transfer of money. What does ‘I’ in MMID stands for?

(a)  Input

(b)  Internet

(c)  Identifier

(d)  Ink

(e)  Immediate

Answer: (c)

2. Which of the following is an electronic transfer system that operates in hourly batches?

(a)  NEFT

(b)  RTGS

(c)  Paytm

(d)  MICR

(e)  UPI

Answer: (a)

3. In which of the following payments banks, the stakes are wholly owned by government of India?

(a)  Paytm

(b)  NSDL

(c)  Bandhan Bank

(d)  IPPB

(e)  FINI pay tech

Answer: (d)

4. Rabobank is a multinational banking and financial services company. It is headquartered in

(a)  Germany

(b)  France

(c)  Sweden

(d)  Switzerland

(e)  Netherlands

Answer: (e)

5. EFTPOS is an electronic payment system involving electronic fund transfers based on the use of payment cards, such as debit or credit cards, at payment terminals located points of sale. What does ‘P’ stands for in EFTPOS?

(a)  Payment

(b)  Partial

(c)  Panel

(d)  Point

(e)  Public

Answer: (a)

6. Irom Chanu Sharmila is a political right activist and recently launched People’s Resurgence Justice Alliance in which o the following States?

(a)  Meghalaya

(b)  Manipur

(c)  Mizoram

(d)  Nagaland

(e)  Arunachal Pradesh

Answer: (b)

7. As per the draft guidelines for ‘on tap’ licensing of Universal Banks, released by RBI, maximum limit for foreign investment in India is

(a)  49%

(b)  71%

(c)  57%

(d)  74%

(e)  100%

Answer: (d)

8. The Global Citizen Fest was recently hosted in which of the following cities?

(a)  Delhi

(b)  Jaipur

(c)  Mumbai

(d)  Bengaluru

(e)  Gurugram

Answer: (c)

9. India has signed a deal of Rs. 5000 crore to buy M777 Howitzer artilleries from which of the following countries?

(a)  China

(b)  Japan

(c)  Russia

(d)  The USA

(e)  France

Answer: (d)

10. The rate at which RBI lends to the commercial banks, is called

(a)  Base rate

(b)  Repo Rate

(c)  Reverse Repo Rate

(d)  SLR

(e)  CRR

Answer: (b)

11. NFS is the largest network of shared Automatic Teller Machines (ATMs) in India and is run by which of the following organization?

(a)  Stock Holding Corporation of India Limited

(b)  National Stock Exchange

(c)  National Securities Depository Limited

(d)  National Payments Corporation of India

(e)  Reserve Bank of India

Answer: (d)

12. Who was the UAE-based Indian teenager that had recently won the International Peace Price Award 2016?

(a)  Meghna Pant

(b)  Vaidehi Reddy

(c)  Abhisekh Saharia

(d)  Chhavi Kohli

(e)  Kehkashan Basu

Answer: (e)

13. Which of the following is theme for the United Nations’s International Day of persons with Disabilities?

(a)  Sustainable Development : The Promise of Technology

(b)  Break Barriers, Open Door : For an Inclusive Society and Development for All

(c)  Achieving 17 Goals for the Future we want

(d)  Inclusion Matters : Access and Empowerment for People of All Abilities

(e)  Together for a Better World for All : Including Persons with Disabilities in Development

Answer: (c)

14. The RBI original share capital was divided into shares of 100 each fully paid, which were initially owned entirely by which of the following entities?

(a)  Government of India

(b)  British Government

(c)  East India Company

(d)  Public Shareholders

(e)  Private Shareholders

Answer: (e)

15. The International Criminal Court is an intergovernmental organization and international tribunal that is headquartered in which of the following countries?

(a)  Netherlands

(b)  Sweden

(c)  Switzerland

(d)  Denmark

(e)  Germany

Answer: (a)

16. Local Area Banks were set-up by the government of India with jurisdiction over two or three contiguous

(a)  localities

(b)  cities

(c)  states

(d)  districts

(e)  municipalities

Answer: (d)

17. India’s first banking robot Lakshmi to be launched by City Union Bank in which of the following cities?

(a)  Bengaluru

(b)  Chennai

(c)  Mumbai

(d)  Hyderabad

(e)  Delhi

Answer: (b)

18. Which of the following is not included in market risk?

(a)  Interest rate risk

(b)  Equity risk

(c)  Currency risk

(d)  Commodity risk

(e)  Specific risk

Answer: (e)

19. The author of the trilogy ‘The Immortals of Meluha’ is

(a)  Kavita Kane

(b)  Amish Tripathi

(c)  Amitav Ghosh

(d)  Vikram Seth

(e)  Gregory Roberts

Answer: (b)

20. The Kumarakom Bird Sanctuary is located in the Indian State of

(a)  Tamil Nadu

(b)  Jammu and Kashmir

(c)  Karnataka

(d)  Manipur

(e)  Kerala

Answer: (e)

21. The Heart of Asia summit to discuss regional issues, particularly encouraging security, political and economical cooperation among Afghanistan and its neighbours was held in which of the following cities?

(a)  Istanbul

(b)  Kabul

(c)  Amritsar

(d)  Delhi

(e)  Hyderabad

Answer: (a)

22. The saving bank account, which does not operate for more than 24 months, then the accounts is known as

(a)  inactive account

(b)  dormant account

(c)  inaccessible account

(d)  demat account

(e)  inoperative account

Answer: (b)

23. Who was awarded the best actor award at the Asia Pacific Screen Awards 2016?

(a)  Irrfan Khan

(b)  Manor Bajpayee

(c)  Nawazuddin Siddique

(d)  Ahmed Al-Rokh

(e)  David Puttnam

Answer: (b)

24. Which of the following is a web-based solution to facilitate interbank, high volume, electronic transactions which are repetitive and periodic in nature?

(a)  SEBI

(b)  NEFT

(c)  Paytm

(d)  NPCI

(e)  NACH

Answer: (e)

25. What is the maximum period up to which NBFCs can accept deposit from the date of receipt of such deposit?

(a)  12 months

(b)  24 months

(c)  48 months

(d)  84 months

(e)  96 months

Answer: (*)

26. Recently, RBI relaxed norms for 2-factor authentication for payments up to which of the following?

(a)  Rs. 2000

(b)  Rs. 5000

(c)  Rs. 1000

(d)  Rs. 4000

(e)  Rs. 10000

Answer: (a)

27. M. Balamurali Krishna, recently passed away at the age of 82, was a well known

(a)  Director

(b)  Carnatic singer

(c)  Cardiologist

(d)  Playwright

(e)  Politician

Answer: (b)

28. Gobind Sagar Dam is a man-made reservoir situated in Bilaspur district. It is situated in

(a)  Uttarakhand

(b)  Uttar Pradesh

(c)  Punjab

(d)  Himachal Pradesh

(e)  Jammu and Kashmir

Answer: (d)

29. Commercial Paper (CP) is an unsecured money market instrument issued in the form of which of the following?

(a)  Treasury bills

(b)  Certificate of deposit

(c)  Bills of exchange

(d)  Promissory note

(e)  Bank note

Answer: (d)

30. Asia’s first 207-km long cycling highway is inaugurated in the Indian State of

(a)  Kerala

(b)  Madhya Pradesh

(c)  Uttar Pradesh

(d)  Uttarakhand

(e)  Haryana

Answer: (c)

31. Joint training Exercise ‘Hand-in-Hand’ is held between India and

(a)  Russia

(b)  USA

(c)  China

(d)  Japan

(e)  France

Answer: (c)

32. The world’s first Bollywood theme park has recently been opened in

(a)  Mumbai

(b)  New York

(c)  Dubai

(d)  Hong Kong

(e)  Paris

Answer: (c)

33. If a person provides proof of having applied for an officially valid document, then the validity of the account is extended for how many months?

(a)  Six

(b)  Three

(c)  Eight

(d)  Twelve

(e)  Four

Answer: (d)

34. Loans of very small amounts given to low income groups or poor households, is called

(a)  Micro credit

(b)  Cash credit

(c)  Simple overdraft

(d)  Rural credit

(e)  No frills loan

Answer: (a)

35. Which of the following companies has recently started a service to deliver cash to doorsteps through micro ATMs powered by the bank?

(a)  Flipkart

(b)  Snapdeal

(c)  Amazon

(d)  Uber

(e)  Big Basket

Answer: (b)

36. Which of the following microfinance institution is a Universal Bank?

(a)  SKS Microfinance Private Limited

(b)  Disha Microfinance Private Limited

(c)  Arohan Financial Services Private Limited

(d)  Bandhan Financial Services Private Limited

(e)  Annupurna Private Limited

Answer: (b)

37. The small finance Banks also known as Payments Bank are mandated to open how much percent of their branches in rural unbanked areas?

(a)  28%

(b)  25%

(c)  30%

(d)  32%

(e)  44%

Answer: (b)

38. Which of the following is the Executive Director of the RBI Working Group on Fin Tech and Digital Banking?

(a)  AK Pandey

(b)  PG Apte

(c)  B. Mahapatra

(d)  Sudarshan Sen

(e)  DK Mohanty

Answer: (d)

39. Which of the following is the only Central Record Keeping Agency appointed by the government of India?

(a)  CIBIL

(b)  SEBI

(c)  NABARD

(d)  Stock Holding Corporation of India Limited (SHCIL)

(e)  National Securities Depository Limited (NSDL)

Answer: (e)

40. The National Financial Switch, which facilitates routing of ATM transactions, is developed by which of the following organizations?

(a)  RBI

(b)  SEBI

(c)  NPCI

(d)  IDRBT

(e)  SIDBI

Answer: (d)

Part II Reasoning

 

Directions (Q. Nos. 41-44) Study the following information carefully to answer the given questions.

In certain code language

‘work for earning money’ is coded as ‘Go3 None X Xor4 Farnin7’

‘like six years passed’ is coded as ‘Ti3 Qasse6 Zear5 Mik4’

‘hence good amount received’ is coded as ‘Seceive8 lenc5 Hoo4 Bmoun6’

41. What is the code for last earning was money?

(a)  Xa3 Mas4 None5 Farnin7

(b)  None5 Xa3 Mas4 Darnin7

(c)  None5 Mas4 Farnin7 Xa3

(d)  Mas4 one5 Farnin7 Xa3

(e)  None of these

Answer: (c)

42. If ‘money makes man perfect’ is coded as ‘Nake5 Qerfec7 Na3 None5’, then what is the code for good people always perfect?

(a)  Qeopl6 Hoo4 Blway6

(b)  Hoo4 Qerfec7 Blway6 Qeopl6

(c)  Qeople6 Hoo4 Qerfec7 Blway6

(d)  Qerfec7 Blway6 Qeopl5 Hoo5

(e)  None of the above

Answer: (b)

43. What is the code of the ‘hence always wrong hance’?

(a)  Blway6 lance5 Xron5 lenc5

(b)  Lanc5 Xron5 Blway5 Lenc5

(c)  Lanc5 Xron5 Blway5 Lenc5

(d)  Blway6 Lenc5 Lanc5 Xron5

(e)  Lanc6 Xron6 Blway5 Lenc5

Answer: (d)

44. ‘Framing’ is coded as

(a)  Garmin7

(b)  Gramin8

(c)  Garing8

(d)  Earin7

(e)  Earnin8

Answer: (a)

Directions (Q. Nos. 45-50) Read the given information carefully and answer the given questions.

There are five u nits i.e., 1, 2, 3, 4 and 5. Each unit has a different height. Also each unit contains books and boxes. Unit 2 is above Unit 1 and Unit 3 is above Unit 2 and so on. Every unit belongs to different country i.e., Beijing, Paris, London, Sydney and Zurich. The total h eight of all five u nits is equal to 252″ ft.

  • Total height of unit is equal to the total height of books plus total height of books plus total height of boxes in each unit.
  • Height of books is not equal to the height of boxes, unless specified.

The books belongs to London is an even unit. The total height of Unit 1 is 75ft. Sydney is not a Unit 1. The total height of unit, which belongs to Sydney, is 55 ft. There is only one gap between London and Paris. The height of books and height of boxes in Unit 3 are equal. The height of books in Unit 2 is not less than 30 ft. The books Unit 4 are four m ore than books, which i s in Unit 3.

The total height of London is not 37ft. The total height of unit, which contains 37 ft is not taking the place, which is immediately above a unit, which contains 20 ft more than the unit, which belongs to Sydney. The height of boxes in Unit 2 is 23 ft. Unit belongs to Zurich does not contain the equal height of books and boxes. The total height of Unit 2 is an odd number and h eight is more than 50 ft and less than 55ft. Unit 1 contains 23ft height of boxes more than Unit 4 contains height of boxes and Unit 5 contains 7 ft. height of books less than Unit 1 contains.

45. What is the total height in Unit 3?

(a)  37 ft

(b)  32 ft

(c)  53 ft

(d)  75 ft

(e)  80 ft

Answer: (b)

46. Unit 3 belongs to which country?

(a)  Paris

(b)  Zurich

(c)  Sydney

(d)  Beijing

(e)  London

Answer: (d)

47. If ‘Sydney’ is related to 37 ft in the same way as Beijing is related to 53 ft, which of the following is ‘Paris’ related to, following the same pattern?

(a)  53 ft

(b)  37 ft

(c)  75 ft

(d)  32 ft

(e)  30 ft

Answer: (c)

48. Four of the following five are a like in a certain way and hence they form a group. Which one of the following does not belong to that group?

(a)  Sydney

(b)  32 ft

(c)  75 ft

(d)  Paris

(e)  37 ft

Answer: (b)

49. What is the height of box in Unit 4?

(a)  23 ft

(b)  17 ft

(c)  27 ft

(d)  40 ft

(e)  20 ft

Answer: (b)

50. Which unit contains 30 ft book?

(a)  Unit 2

(b)  Unit 3

(c)  Unit 1

(d)  Unit 5

(e)  Unit 6

Answer: (a)

Directions (Q. Nos. 51-55) Read the given information carefully and answer the given questions.

Two buses A and B starts their journey from same depot but not necessary in the same order. Bus A starts moving 6 km towards North direction to point P, which is in the 8 km of East from dept and then take a right turn and move 18 km to reach Point R, then take a left turn and move 12 km to reach Point S. Point P, Q and R lies in same line.

Distance between P and Q is one third of the distance between Q and R. Bus B starts to move towards West 2 km more than Point P, Q and R from depot and reach Point W, then take a right turn and walk 18 km to reach Point U. U, V and W lies in the same lines as distance of V and W is double of the distance between U and V. From Point U Bus B take a right turn and reach Point X. Point T lies between U and X in exact midway.

51. What is the total distance between U and S?

(a)  29 km

(b)  32 km

(c)  36 km

(d)  30 km

(e)  28 km

Answer: (b)

52. In which direction is Point T with respect to Point W?

(a)  North

(b)  South

(c)  North-West

(d)  North-East

(e)  South-West

Answer: (e)

53. A boy want to go to depot and starts walking towards South from Point W. After reaching point V he suddenly realize that he is going to wrong direction. So What is the shortest route from Point V to bus depot?

(a)  Back to Point W then go to depot from there.

(b)  From Point V he should go to East direction 14 km then move toward. South and reach depot.

(c)  From Point V going towards North to reach Point U from there,  he should reach Point S, then Point R, then Point P, then depot.

(d)  Both A and B are shortest route

(e)  Back to Point V, he should go to West direction 14km then move towards North and Reach depot.

Answer: (a)

54. What is the distance between T and X?

(a)  14 km

(b)  12 km

(c)  16 km

(d)  18 km

(e)  6 km

Answer: (a)

55. Bus A is moving in which direction currently?

(a)  West

(b)  North-East

(c)  South

(d)  North

(e)  East

Answer: (d)

Directions (Q. Nos. 56-60) In each of these questions, two statements marked as I and II are provided. These may have a cause and effect relationship or may have independent causes or be the effects of independent causes. Read both the statements and

(a) if statement I is cause and statement II is effect

(b) if statement II is cause and statement I is effect

(c) if statement I and II both are independent causes

(d) if statement I and II both are the effect of independent causes.

(e) if both statements are effect of simple cause.

56. Statement I. It is the bounden duty of each member of the civil society to contain the air pollution by contributing their best in this endeavour to safeguard the health of their countrymen.

Statement II. The alarming air pollution in our country is causing asthma cases to constantly multiply.

Answer: (a)

57. Statement I. The prices of petrol and diesel in the domestic market have remained unchanged for the past few months.

Statement II. The crude oil prices in the international market have gone up substantially in the last few months.

Answer: (c)

58. Statement I. There has been an increase in the underground water level column at all places in Delhi due to the last year’s monsoon rains.

Statement II. Many train shad to be cancelled last year due to water-logging on the railway tracks.

Answer: (b)

59. Statement I. The prices of sugar had risen very sharply in Indian markets last year.

Statement II. The government imported large quantities of sugar as per trade agreements with other countries last year.

Answer: (d)

60. Statement I. The government has recently fixed the fees for professional courses offered by the unaided institutions which are much lower than the fees charged last year.

Statement II. The parents of the aspiring students launched a severe agitation last year protesting against the high fees charged by the unaided institutions.

Answer: (b)

Directions (Q. Nos. 61-64) Study the following information and answer the questions given below.

J, K, L, M, N, O, P and Q are sitting around a circular table facing the centre. Each of them was born in a different year-1971, 1975, 1979, 1980, 1981, 1984, 1985 and 1990, not necessarily in the same order.

M is sitting second to the right of K. L is sitting third to the right of J. Only the one born in 1984 is sitting exactly between J and K.

N, who is the eldest is not an immediate neighbour of J and M. Q is older than only M. Q is sitting second to the left of P. P is not an immediate neighbour of N. J is younger than L. K was born before O.

61. Who is the second eldest in the group?

(a)  J

(b)  L

(c)  K

(d)  P

(e)  O

Answer: (c)

62. Who is sitting third to the right of O?

(a)  The one born in 1979

(b)  The one born in 1980

(c)  The one born in 1985

(d)  The one born in 1984

(e)  The one born in 1981

Answer: (c)

63. In which year was Q born

(a)  1979

(b)  1975

(c)  1980

(d)  1985

(e)  1981

Answer: (c)

64. Which of the following statements is true regarding K?

(a)  K is sitting fourth to the right of Q.

(b)  The one who was born in 1975 is sitting to the immediate left of K.

(c)  K is younger than Q

(d)  There are three people sitting between N and K

(e)  K sits to the immediate right of J

Answer: (d)

65. Who is younger to L?

(a)  K

(b)  M

(c)  N

(d)  P

(e)  None of these

Answer: (d)

Directions (Q. Nos. 66-70) Study the following information carefully and answer the given questions. Eight persons I, J, K, L, M, N, O and P are sitting around a circular table but not necessarily in the same order. Three of them are facing outward while five are facing towards the centre.

M is third to the right of K. N is sitting third to the left of M. Three persons are sitting between N and J. O is sitting third to the right of N, who is not facing the centre. L is sitting third to the right of I, who is not facing the centre.

66. Who sits between N and I?

(a)  J

(b)  K

(c)  M

(d)  O

(e)  None of these

Answer: (e)

67. Who among the following sits second to the right of J?

(a)  L

(b)  K

(c)  M

(d)  N

(e)  O

Answer: (b)

68. Which of the following statements is/are true with respect to P?

(a)  P sits opposite to that of O.

(b)  P sits fourth to the right of O.

(c)  P sits fourth to the left of O.

(d)  None of the given statements is true with respect to P.

(e)  All the given statements are true

Answer: (e)

69. Which of the following represents the group, of those facing away from the centre?

(a)  IKN

(b)  KNL

(c)  IJK

(d)  LMN

(e)  JKL

Answer: (b)

70. What is the position of K with respect to P?

(a)  K sits third to the right of P

(b)  K is second to the left of P

(c)  K is third to the left of P

(d)  K is in front of P.

(e)  None of the given statements is true

Answer: (c)

Directions (Q. Nos. 71-73) Each of the question below consists of a questions and three statements numbered I, II and III. You have to decide whether the data provided in the statements are sufficient to answer the questions. Read all the tree statements and

Given answer

(a) if the statements II and III are sufficient to answer the question, but statement I alone is not sufficient to answer the question

(b) if all the statement taken together are not sufficient to answer the question

(c) if the statement I and II is sufficient to answer the question, but statement III is not sufficient to answer the question

(d) if all the statements are sufficient enough to answer the question

(e) if statement I and III are sufficient to answer the question but statement II alone is not sufficient to answer the question

71. Which of the following represents ‘come’ in a code language?

I. ‘pit na ja od’ means ‘you may come here’ in that language.

II. ‘ja ta ter’ means ‘come and go’ in that cod language.

III. ‘od na pit ter’ means ‘you may go home’ in that code language.

Answer: (c)

72. How many sons does D has?

I. B and F are brothers of A.

II. C is the sister of A and F.

III. C and E are daughters of D.

Answer: (d)

73. Who amongst P, Q, R, S and T is in the middle while standing in a line?

I. Q is to the right of T. II. S is between P and T.

III. Q is between T and R.

Answer: (d)

Directions (Q. Nos. 74-75) In each of the questions below is given a statement followed by courses of action numbered I and II. A course of action is a step or an administrative decision to be taken for improvement follow up or further action in regard to the problem policy etc. On the basis of the information given in the statement, you have to assume everything in the statement to be true and then decide, which of two given courses of action logically follow(s).

Given answer

(a) if only I follows                              (b) if only II follows

(c) if either I or II follows           (d) if neither I nor II follows

(e) if both I and II follow

74. Statement The officer Incharge of a company had a lunch that some money was missing from the almirah.

Course of Action

I. They should get it recounted with the staff and check it with the balance sheet.

II. He should inform the police.

Answer: (d)

75. Statement Exporters in the capital are alleging that commercial banks are violating a Reserve Bank of India directive to operate a post shipment export credit denominated in foreign currency at international rates from January this year.

Course of Action

I. The officers concerned in the commercial banks are to be suspended.

II. The RBI should be asked to stop giving such directives to commercials banks.

Answer: (b)

76. Read the following information to answer the given question.

A tea manufacturing brand V, introduces a new ready to mix packaging of tea. But it had to withdraw its product from the market in a short span soon after the product was launched, it faced heavy losses.

Which of the following cannot be a reason for the withdrawal of new product from the market?

(a)  The tea was available only in a single flavour.

(b)  Media preservatives said that the tea has contained preservatives more than the permissible limit.

(c)  The ready to mix tea was only in large packaging.

(d)  The prices of the tea were unusually higher than other such items of regular use.

(e)  The introductory scheme of free gift voucher with the tea was withdrawn from the market within 15 days f launch of the product.

Answer: (c)

Directions (Q. Nos. 77-80) Read the given passage carefully answer the given questions.

There are seven persons in family namely A, B, C, D, E, F and G. All of them related with each other in on

77. If the total age of B and D is 75 years, then what is the age of B?

(a)  40 yr

(b)  37 yr

(c)  6 yr

(d)  10 yr

(e)  15 yr

Answer: (c)

78. How is D related to A?

(a)  Grandfather

(b)  Mother

(c)  Grandmother

(d)  Father

(e)  Cannot e determined

Answer: (c)

79. Who among the following person is 35 years old?

(a)  A

(b)  E

(c)  D

(d)  F

(e)  C

Answer: (b)

80. What may be possible age of C?

(a)  52 yr

(b)  39 yr

(c)  37 yr

(d)  29 yr

(e)  2 yr

Answer: (c)

Part III Computer Awareness

 

81. The scrambling of code is known as

(a)  Encryption

(b)  Firewalling

(c)  Scrambling

(d)  Deception

(e)  Permuting

Answer: (a)

82. What is the full form of VoIP?

(a)  Voice of Internet Power

(b)  Voice over Internet Protocol

(c)  Voice on Internet Protocol

(d)  Voice on Optimized internet Protocol

(e)  Very Official Internet Protocol

Answer: (b)

83. Why do you log-off from your computer hen going out from your office?

(a)  Someone might steal your file, passwords etc.

(b)  In order to save electricity

(c)  logging off is essential to increase performance.

(d)  logging off is mandatory you before go out

(e)  logging off is a good exercise to perform regularly.

Answer: (a)

84. Which of the following is not responsible for the performance of the computer?

(a)  number of keys in the keyboard.

(b)  name of the video/graphics word.

(c)  memory in the video/graphics word.

(d)  the lock speed of the processor.

(e)  number of cores available in the processor

Answer: (a)

85. Which of the following is used to access a file from the computer store?

(a)  Insert

(b)  Retrieve

(c)  File

(d)  Print

(e)  Find

Answer: (b)

86. Hard drive is used to store

(a)  volatile data

(b)  non-volatile data

(c)  permanent data

(d)  temporary data

(e)  intermediate data

Answer: (b, c)

87. Which of the following is the combination of numbers, alphabets along with username used to get access to user account?

(a)  Password

(b)  Username

(c)  Title name

(d)  Host-Id

(e)  Screen name

Answer: (a)

88. Which of the following is the type of software that has self-replicating software that causes damage to files and system?

(a)  Viruses

(b)  Trojan horses

(c)  Bots

(d)  Worms

(e)  Backdoors

Answer: (d)

89. ………….. is used for very large files or where a fast response time is not critical. The files to be transmitted are gathered over a period and then send together as a batch.

(a)  Batch processing

(b)  Online processing

(c)  File processing

(d)  Data processing

(e)  Text processing

Answer: (a)

90. Which of the following system is a function of dedicated PCs?

(a)  Meant for a single user

(b)  Meant for the single task

(c)  Deal with single software

(d)  Deal with only editing

(e)  Deal for music purpose

Answer: (a)

91. What can possibly be the drawback of e-mails?

(a)  e-mails requires being physically delivered to the user.

(b)  e-mails infects computer.

(c)  e-mails are very expensive to transmit.

(d)  e-mails are slow to load.

(e)  people don’t check e-mails regularly.

Answer: (b)

92. Which of the following is a valid e-mail address?

(a)  name.website@info@ed

(b)  [email protected]

(c)  [email protected]

(d)  [email protected]

(e)  [email protected]

Answer: (d)

93. Which of the following character set supports Japanese and Chinese fonts?

(a)  EBCDIC

(b)  ASCII

(c)  BC

(d)  ECBI

(e)  Unicode

Answer: (b)

94. What is the full form of RTF?

(a)  Richer Text-Formatting

(b)  Rich Text Format

(c)  Right Text Fishing

(d)  Right Text Font

(e)  Rich Text Font

Answer: (b)

95. Which of the following is the text alignment available in word processing software that adjusts the left margin while keeping the right margin in any way?

(a)  Justify

(b)  Left justify

(c)  Right justify

(d)  Centre

(e)  Orientation

Answer: (b)

96. Which of the following is false about the clock of the system?

(a)  It is the property of the toolbar.

(b)  The system can periodically refresh the time by synchronizing with a time source.

(c)  System time is the current date and time of day.

(d)  The system keeps time so that your applications have ready access to accurate time.

(e)  The system bases system time on Coordinated Universal Time (UTC).

Answer: (a)

97. Which of the following are properties of USB?

(a)  Platform independent

(b)  Platform dependent

(c)  Source dependent

(d)  Software dependent

(e)  Software independent

Answer: (a)

98. An act of sending e-mails or creating webpages that are designed to collect an individual’s online bank, credit card, or other login information?

(a)  Phishing

(b)  Spam

(c)  Hacking

(d)  Cracking

(e)  Malware

Answer: (a)

99. Which of the following is the organized collection of large amount of interrelated data stored in a meaningfully way used for manipulation and updating?

(a)  Database

(b)  File

(c)  Folder

(d)  Data-mining

(e)  Data source

Answer: (a)

100. Which among the following cycle consists of an input, processing, output and storage as its constituents?

(a)  Processing

(b)  Output

(c)  Input

(d)  Storage

(e)  Data

Answer: (e)

101. The part of a computer that coordinates all its functions, is called its

(a)  ROM program

(b)  System board

(c)  Arithmetic logic unit

(d)  Control unit

(e)  None of these

Answer: (d)

102. Oracle is a

(a)  RDBMS

(b)  hardware

(c)  system software

(d)  high level language

(e)  None of these

Answer: (a)

103. Which of the following computer languages is a mathematically oriented language used for scientific problems?

(a)  Fortran

(b)  Cobol

(c)  Lisp

(d)  Prolog

(e)  Android

Answer: (a)

104. Coded entries, which are used to gain access to a computer system, are called

(a)  Entry codes

(b)  Passwords

(c)  Security commands

(d)  Codewords

(e)  None of these

Answer: (b)

105. Communication between a computer and keyboard involves ………. transmission.

(a)  Automatic

(b)  Half-duplex

(c)  Full-duplex

(d)  Simplex

(e)  None of these

Answer: (d)

106. In which of the following level of OSI model, data encryption is done?

(a)  Application layer

(b)  Session layer

(c)  Transport layer

(d)  Network layer

(e)  Data link layer

Answer: (*)

107. ………… controls the way in which the computer system functions and provides a means by which users can interact with the computer.

(a)  The operating system

(b)  The motherboard

(c)  The platform

(d)  Application software

(e)  Hardware

Answer: (a)

108. Which of the following is not a property of fibre optic cabling?

(a)  Transmits at faster speeds than copper cabling

(b)  Easier to capture a signal from than copper cabling

(c)  Very resistant to interference

(d)  Carries signals as light waves

(e)  less attenuation

Answer: (a)

109. DEL command is used to

(a)  delete files

(b)  delete directory

(c)  delete lables

(d)  delete contents of file

(e)  Both (a) and (b)

Answer: (a)

110. ………… are software which used to do particular task.

(a)  Operating system

(b)  Program

(c)  Data software

(d)  Data

(e)  Application software

Answer: (e)

111. …………. is the process of dividing the disk into task and sectors.

(a)  Alloting

(b)  Crashing

(c)  Formatting

(d)  Tracking

(e)  Decrypting

Answer: (c)

112. What type of software creates a smaller files that is faster to transfer over the internet?

(a)  Compression

(b)  Fragmentation

(c)  Encapsulation

(d)  Unzipped

(e)  Abstraction

Answer: (a)

113. A process known as……………. is used by large retailers to study trends.

(a)  data mining

(b)  data election

(c)  POS

(d)  data conversion

(e)  data inversion

Answer: (a)

114. The ability to recover and read deleted damaged files from a criminal’s computer is an example of a law enforcement specially, is called

(a)  robotics

(b)  simulation

(c)  computer

(d)  animation

(e)  encapsulation

Answer: (c)

115. ………… is data that has been organized and presented in a meaningful fashion.

(a)  A process

(b)  Software

(c)  Storage

(d)  Information

(e)  Data

Answer: (d)

116. The components that process data are located in which of the following?

(a)  Input devices

(b)  Output devices

(c)  System unit

(d)  Storage component

(e)  Expansion board

Answer: (c)

117. The CPU and memory are located in which of the following devices?

(a)  Motherboard

(b)  Expansion board

(c)  Storage device

(d)  Output device

(e)  System unit

Answer: (a)

118. …………… is the revolving around the use of nano structures to build devices on an extremely small scale.

(a)  Nano-technology

(b)  Micro-technology

(c)  Computer forensics

(d)  Artificial Intelligence

(e)  Very small scale technology

Answer: (a)

119. The steps and tasks need to process data, such as response to questions or clicking n icon, are called

(a)  instructions

(b)  the operating system

(c)  application software

(d)  the system unit

(e)  the hardware unit.

Answer: (a)

120. Which of the meal or plastic case that holds all the physical parts of the computer?

(a)  System unit

(b)  CPU

(c)  mainframe

(d)  platform

(e)  microprocessor

Answer: (a)

IV English Language

 

Directions (Q. Nos. 121-125) Each sentence below has two/three blanks. There are five pair/group of words below each sentence. Each pair/group is indicated as 9a), (b), (c), (d) and (e). Choose the pair/group of words which can be filled up in the blanks in the sentence in the same order so as to complete the sentence meaningfully.

121. Activities in the country have long protested its …………. society that essentially………… women from travelling, marrying or attending college without permission from a male relative, who is called their

(a)  benevolent, forbid; steward

(b)  pre-adamite; prevent; custodian

(c)  pre-eminent; restrict; protector

(d)  venerable; condemns; manciple

(e)  patriarchial; prohibits; guardian

Answer: (e)

122. Individual tax rates in the country are substantially ………. compared to tax rates in the US and Western Europe but slightly ……… than those in most emerging markets.

(a)  slow; more

(b)  decreasing; increased

(c)  lesser; much

(d)  lower; higher

(e)  stricter; lower

Answer: (d)

123. The regular body is likely to ……. stricter trading norms, create special liquidity window on the budget day to combat potential market………..

(a)  implement; versatile

(b)  let; flexibility

(c)  remove; steady

(d)  ignore; dynamic

(e)  impose; volatility

Answer: (e)

124. Sales of consumer packaged goods companies are ……….. to record their slowest growth in two years as customers cutback their expenditure even on …………. items and groceries.

(a)  expected; essential

(b)  willing; regularly

(c)  estimate; necessary

(d)  approximate; daily

(e)  required; urgent

Answer: (a)

125. The bad loan ratios are ……. to be uglier in the fourth quarter given the ……….. loan growth.

(a)  moves; frail

(b)  likely; weaker

(c)  tend; skewed

(d)  intend; meagre

(e)  going; swiftly

Answer: (b)

Directions (Q. Nos. 126-133) Read the passage carefully and answer the given questions. Certain words/phrases have been given in bold to help you locate them while answering some of the questions.

Though global multinationals account for only 2% the world’s jobs, they own or orchestrate the supply chains that account for over 50% of world trade, they make up 40% of the value of the West’s stockmarkets; and they own most of the world’ intellectual property. Although the idea of being at the top of the food chain makes these companies sound all-conquering, rickety and overextended are often more fitting adjectives.

Companies became obsessed with internationalizing their customer’s production, capital and management. In 2016, multinationals’ cross-border investment fell by 10-15% the share of trade accounted for by cross-border supply chains has stagnated since 2007 and the proportion of sales that Western firms make outside their home region has shrunk.

They are in retreat. To understand why this is, consider the three parties that made the boom possible. Investors, the ‘headquarters countries’ in which global firms are domiciles, and the ‘host countries’ that received multinational investment. Each though that multinational firms would provide superior financial or economic performance.

Investors saw a huge potential for economies of scale. As China, India and the Soviet Union opened up. and as Europe liberalized itself into single market, firms could sell this same product to more people. Moreover, they saw ‘geographical arbitrage’ i.e., from the rich world they could get management, capital, brands and technology and from the emerging world they could get cheap workers and raw materials as well as lighter rules on pollution.

These advantages led investors to think global firms would grow faster and make higher profits. That was true for a while, not today. The profits of the top 700-odd multinational firms have dropped by 25% over the past five years, according to FTSE, an index firm. The weakness of many currencies against the dollar is part of the story; the slump in commodity prices, and thus the profits of oils firms, mining firms and the like is a factor too. Another 10% of deterioration is due to the collapse of banks. Individual bosses will often blame one-off factors, currency moves, the economic collapse of Venezuela currency swaps and the like can be thought of, a depression in Europe, a crackdown on graft in China, and so on.

But the deeper explanation is that both the advantages of scales and those of arbitrage have worn away. As a result, firms with a domestic focus are winning market share by 2%. In Brazil, two local banks, have trounced global lenders what about the ‘headquarters countries’? In the 1990s and 2000s, they wanted their national champions to go global in order to become bigger and brainier.

The mood changed after the financial crisis. Multinational firms started to be seen as agents of inequality. They created jobs abroad, but not at home. As a result, the tapestry of rules designed to help business globally is fraying. Takeovers of western firm now often come with string attached by governments to safeguard local jobs and plants. There are gathering clouds in host countries as well. China has been turning the screws on foreign firms in a push for ‘Indigenous innovation’.

Bosses say that more products have to be sources locally and intellectual property often ends up handed over to local partners. Strategic industries, including the internet, are out of bounds to foreign investment. Many fear that China’s approach will be mimicked around the developing world, forcing multinational firms to invest more locally and create more jobs a mirror image of the pressure placed on them at home.

126. Which of the following is true in the context of the passage?

A. Multinationals wanted their champions to go global in 1990s and 2000s.

B. Financial measures to address the situation in Venezuela.

C. Breaking of large corporate entities.

(a)  Only C

(b)  Only B

(c)  Only A

(d)  A and B

(e)  All of these

Answer: (a)

127. Which of the following can be said about the multinationals in the context of the passage?

A.Investors in multinational firms may stop investing.

B. There is resistance to multinational firms in their own domestic markets.

C. Multinationals are struggling.

(a)  Only A

(b)  Only B

(c)  A and B

(d)  All of these

(e)  A and C

Answer: (d)

128. Which of the following phrases from the passage can be replaced by ‘trouble’ in the context of the passage?

A. Out of bounds. B. Economies of scale

C. Gathering clouds D. In retreat

(a)  Only A

(b)  All of these

(c)  B and D

(d)  A and B

(e)  Only C

Answer: (e)

129. Which of the following is responsible for and is best explanations(s) for the current health of global firms?

A. Economic trouble in some countries.

B. Advantages of labour that were available earlier have diminished.

C. Failure of banks.

(a)  Only C

(b)  Only B

(c)  A and B

(d)  A and C

(e)  All of these

Answer: (d)

130. Which of the following is/are reason(s) for the author’s mention of various countries in the passage?

A. To illustrate the changing climate for global businesses.

B. To elucidate the similarities in response multinational businesses.

C. To negate the popular assumption that developed overtaken developing economies have overtaken developed ones.

(a)  Only A

(b)  Only B

(c)  Only C

(d)  B and C

(e)  A and B

Answer: (a)

131. Which of the following words is the opposite the word ‘Lighter’ as used in the passage?

(a)  Tighter

(b)  Stricter

(c)  Insignificant

(d)  Robust

(e)  Familiar

Answer: (b)

132. Which of the following cannot be said about ‘geographical arbitrage’?

(a)  It has facilitated the spread of cross-border trade.

(b)  It caused stagnation of trade in the 1990s.

(c)  Its effects have worn off.

(d)  Western firms could increase profits.

(e)  All the given options can be said.

Answer: (b)

133. Which of the following do(es) the passage centrally address?

A rosperity of multinational firms

B. Governments’ increasingly protecting the interest of local firms.

C. Concern for global trade and industry.

(a)  A and C

(b)  Only B

(c)  Only C

(d)  A and B

(e)  All of these

Answer: (b)

Directions (Q. Nos. 134-138) in each question sentence is given with three words/group of words in bold type. One or more of them may have a certain error. Below the sentence is given three combinations of words/group of words i.e., A, B and C. You have to find out the correct word/group of words from among A, B and C given below each sentence to replace the incorrect words/and make the sentence grammatically correct and meaningful. One, two all three or none of them may be correct. Decide upon which is/are correct, if any and mark the correct option which denotes your answer. If the sentence is correct as it is, mark ‘No correction required’ as your answer.

134. In 2015, two years after hurricane Sandy hit his city, the mayor announced to set-up a $ 3 billion restoration fund, partly was intended to pay for sea walls that would help city from storms ahead.

A. was setting up-which-any storms in the future

B. setting up of that storms

C. creation of part of which future storms

(a)  Only A

(b)  All of these

(c)  A and B

(d)  Only C

(e)  No correction required

Answer: (d)

135. Since ancient times, medics have relied on their sense of smell to help them work out what is wrong with their patients-fruity odours on the breath, for example, let them monitor the condition of diabetics, while odours assist in the diagnosis of respiratory-tract infections.

A. relying-to work out anything-diagnosing

B. were reliant-work out whether anything-how to diagnose

C. had to rely-in working out anything-any diagnosis

(a)  Only A

(b)  All of these

(c)  A and B

(d)  Only C

(e)  No correction required

Answer: (e)

136. Thailand’s performing dismally is not drastically out of step with countries of similar incomes, not strange given its unusually generous spending on

A. dismal performance-bit it is-expenditure on

B. having a dismal performance-being-budget

C. dismally performing-is not-amount spent on

(a)  Only C

(b)  All of these

(c)  A and B

(d)  Only A

(e)  No correction required

Answer: (d)

137. There is a government survey, about 12.6 million Japanese aged 60 or older now option working, up from 8.7 million in 2000 and two-thirds of Japan’s over-65s want to stay gainfully employed.

A. In-have an option-and gain employment

B. As revealed by-are option-gains from employment

C. According to-opt to keep-gainfully employed

(a)  Only B

(b)  All of these

(c)  A and B

(d)  Only C

(e)  No correction required

Answer: (d)

138. The hope is that the torrents of data which generate will contain some crucial nuggets that let neuroscientists get more understanding how exactly the brain does all it does.

A. these schemes-closer to-what it does

B. which will able to function

C. that these studies-a better-work

(a)  Only A

(b)  All of the above

(c)  A and B

(d)  Other than those given as options

(e)  None of the above

Answer: (a)

Directions (Q. Nos. 139-143) In these questions, there are four/five sentences A, B, C, D and/or E. Three or Four of these sentences contribute to one main idea when these sentences are formed into a paragraph. One sentence does not contribute to that main idea. That sentence is your answer.

139. Which of the following does not contribute to the main idea of the given passage?

A. The cost of production for luxury goods is not usually a prime concern and capital investment is generally modest, except for watches.

B. Some of the normal rules do not apply to luxury-goods makers, even though in many ways they are similar to other consumer-good companies.

C. To view the world through the lens of luxury is to see it subtly offered.

D. A really prestigious item can b e a ‘Veblen good’, named after an American economist born in themed-19th century who noticed that demand for some good actually rises as they get more expensive because they confer yet more status.

(a)  Only A

(b)  Only B

(c)  A and B

(d)  Only C

(e)  A and C

Answer: (d)

140. Which of the following does not contribute to the main idea of the given passage?

A. These present drivers of its economy, however, are under threat from technology.

B. Africa is a hopeful continent with an exuberance driven by minerals, hydrocarbon and commodities.

C. Without the ability to create knowledge through hands on learning the quality of education, in the country is unlikely to improve.

D. The long-view trajectory of electric vehicles in Africa suggests a future where electrons will power more cars than carbon compounds.

(a)  Only A

(b)  Only C

(c)  Only B

(d)  B and C

(e)  All those given as options contribute

Answer: (e)

141. Which of the following does not contribute to the main idea of the given passage?

A. There is a growing body of research which revealed that the number of vigilance equipment security agencies purchased has decreased over the years.

B. Many parents fret that their offsprings will die unless ceaselessly watched.

C. In 1893, philosopher John Locke warned that children should not be given too much ‘unwholesome fruit’ to eat.

D. By most objective measures, modern parents are far more conscientious than previous generations.

E. Four centuries later, misguided ideas about child-rearing are rife.

(a)  Only A

(b)  Only C

(c)  Only B

(d)  B and C

(e)  All those given as options contribute

Answer: (a)

142. Which of the following does not contribute to the main idea of the passage?

A. Research suggests that pear packets working indoors were slowed by air pollution even at levels well below current air-quality standards.

B. For anyone who as has tried jogging through smog, the physically sapping impacts of air pollution should come as no surprise.

C. On days with more air pollution, workers spend more time on breaks and complete fewer cells.

D. But pollution doesn’t just slow down runners, it hampers workers too.

(a)  Only A

(b)  Only B

(c)  A and B

(d)  B and C

(e)  All those given as options contribute

Answer: (e)

143. Which of the following does not contribute to the main idea of the passage?

A. Under this model, consumers are paying to access someone else’s goods or services for a particular period of time and at cheaper value.

B. The sharing economy has been widely hailed as a major growth sector by sources.

C. Millennial today are willing to pay more for fresh and healthy food.

D. Third model has disrupted nature industries, such as hotels and automotives, by providing consumers with convenient and cost efficient access to resources without the financial, emotional, or social burdens of ownership.

E. The sharing economy isn’t really a “sharing” economy at all. It’s an access economy.

(a)  B and C

(b)  Only B

(c)  Only C

(d)  All those given as option contribute

(e)  Only A

Answer: (c)

Directions (Q. Nos. 144-150) Read the following passage carefully and answer the questions given. Certain words/phrases are given in bold to help you locate them while answering some of the questions.

Intended to give its workers more rest, this year, the Taiwanese government has brought about some labour amendments, such as the ‘one fixed day off, one flexibly day off’. It has steadily garnered raked reactions in the media.

News of businesses moving to enact measures to meet the new regulations has emerged, including price hikes and reduced business hours. Reports, such as popular food chains upping their prices have coloured media. On the other hand, Taiwan’s two major convenience stores denied reports that stores would be closing cover the weekend in response to the regulations.

The latest circumstances only presented examples of how the new holiday policy would hinder Taiwanese business’ operations, pose price hikes and create a more difficult environment for Taiwanese nationals to go about their daily lives in an already wage-stagnant economy.

Increased cost are inevitable. But should it be looked upon with disdain and diversion? Let us look at the issue from a different perspective. Foreigners and tourists have commented that the 24-hour convenience stores and high quality of services have left a positive impression about the country. That should be a point of pride for Taiwan, but on the other hand, the new holiday policy also provides an opportunity to spur change as Taiwan’s laudable conveniences also come along with their own set of burdens.

Many of the everyday conveniences were built upon the hard work and long hours of employees who provided quality and tireless service to the masses. It is understandable that all kinds of enterprises avoid uncertainty as it does not bode well for business operations. The new policy provides uncertainty in spades.

But the status quo that businesses have long upheld has not worked for Taiwan in the past decade. No longer have the long working hours either transferred into benefits for the working people or helped propel Taiwan’s GDP back to the figures during its ‘industrial miracle’ era. Taiwan’s working environment and policies protecting both enterprises and workers have long needed and overhaul.

While initial reports indicate that the new holiday policy has led to rise in basic commodity prices. It should become an opportunity for business to gripe about. Hey should not increase prices but fail to increase staff costs and adjust inferior budget planning, ultimately failing to increase employment. This i s the easy way to face the new policy and for enterprises to stay afloat in tough times.

It should not, however, continue at the expense of the everyday workers and the economy at large. Increasing employment and providing adequate overtime pay to keep working hours in check are factors in the government’s labour revision efforts last y ear, which they hope could improve existing working conditions. It’s still too early to say whether President and his party’s holiday policy would improve working conditions.

Taiwan’s businesses should look out for its employees’ well-being instead of their costs, while the government must similarly look out for businesses and the people at large to ensure effective policy progress. There’s no doubt that the Taiwan’s working environment is in need of change, and the new holiday policy provides a platform for that. However, whether it would bring about a good or bad change ultimately is left up to how the key players- businesses, the government and the people-respond.

144. Which of the following is most nearly the same in meaning of the word ‘Gripe’ as used in the passage?

(a)  Greedy

(b)  Placate

(c)  Complain

(d)  Rejoice

(e)  Satisfied

Answer: (c)

145. Which of the following statements is/are the central idea(s) of the passage?

A. Taiwan’s businesses are suffering due to low exports.

B. Taiwan’s government would not be able to remain in power for long.

C. Taiwan is currently facing an economic boom.

D. Taiwan’s new labour policy has its own set of pros and cons.

(a)  A and C

(b)  Only D

(c)  C and D

(d)  Only B

(e)  All of these

Answer: (b)

146. Which of the following is most nearly the opposite in meaning of the world Disdain as used in the passage?

(a)  Thoughtlessness

(b)  Admiration

(c)  Proportionately

(d)  Disinterest

(e)  Scorn

Answer: (b)

147. According to the passage, what is/are reason(s) the labour amendments were made?

A. Many people were emigrating due to longer working hours and stagnant wages in China.

B. In the recent years, the export volume of Taiwan was showing a downward trend.

C. Commodity prices were missing beyond acceptable limits.

(a)  A and B

(b)  Only C

(c)  B and C

(d)  Only B

(e)  None of these

Answer: (e)

148. What does the author suggest regarding the new policy of Taiwan?

(a)  It should be implemented judiciously and must not be taken unique advantage of.

(b)  Such a change had become  imperative over that few years.

(c)  It has caused uncertainty in businesses on a temporary basis.

(d)  The announcement of the new holiday policy has brought about mixed responses.

(e)  None of the above

Answer: (d)

149. Which of the following will be a consequence of the new holiday policy in Taiwan, as inferred from the passage?

(a)  The motivational levels of Taiwanese will improve therefore enhancing their productivity

(b)  Commodity prices across the world will reduce.

(c)  Taiwan’s transport infrastructure will suffer.

(d)  Employment opportunities in Taiwan will cease.

(e)  Wage stagnation will cease to exist in the long-run.

Answer: (e)

150. Which of the following is/are not true with respect to the passage?

A. The existing policy of long working hours had been boosting Taiwan’s GDP consistently over the years.

B. Most businesses in Taiwan will prosper, thanks to the new policy on increased working hours.

C. The new holiday policy has boosted Taiwan’s GDP this year.

(a)  Only A

(b)  B and C

(c)  A and C

(d)  Only C

(e)  All are not true

Answer: (e)

Directions (Q. Nos. 151-155) In these questions, there are two/three statements which can be combined into  a single statement in a number of different ways without changing their meaning. Below these two sentences are given three probable starters A, B and C of such combined sentence. One, two, three or none of them may be correct. Decide upon which is/are correct, if any and mark the option which denotes your answer. If none of the three starters is suitable, mark ‘None’ as your answer.

151. After years of slow growth and overspending, in part to prop up a state pension scheme, the Central government nearly ran out of cash late last year. The government has called in the IMF, which may now broke a resolution to the crisis.

A. On account of years … .

B. Through the crisis … .

C. A resolution to … .

(a)  Only A

(b)  All of these

(c)  A and B

(d)  A and C

(e)  None

Answer: (a)

152. A professor of economics examined the cost structure of the tournament. He found that the cost of building of the necessary stadiums, transport and accommodations were not properly taken into account.

A. Although the costs of … . B. On examining … .

C. When he … .

(a)  Only B

(b)  All of these

(c)  A and B

(d)  B and C

(e)  Other than those given as options

Answer: (e)

153. Working lives are so lengthy and so fast-changing the simply cramming more schooling in at the start is not enough. People must also be acquire new skills throughout their careers.

A. Not Only … . B. With working lives … .

C. Simple cramming. …

(a)  Only A

(b)  All of these

(c)  A and B

(d)  B and C

(e)  None of these

Answer: (d)

154. Openness delivers benefits. Migrants improve not just their own lives but the economies of host countries. European migrants who arrived in Britain since 2000 are contributors to the exchequer, adding more than 120 billion ($34 billion) to the public finances between 2001 and 2011.

A. However openness … .

B. Since 2000 … .

C. Though migrants … .

(a)  Only A

(b)  All of these

(c)  B and C

(d)  A and B

(e)  None of these

Answer: (e)

155. Most electricity is transmitted today as alternating current (AC), which works well over short and medium distances. But transmission over long distances requires very high voltages, which can tricky for AC systems and ultra-high-voltage direct- current (UHV) connectors are better suited to such spans.

A. While most electricity … .

B. Despite AC systems are better … .

C. Alternating current … .

(a)  A and C

(b)  A and B

(c)  Other than those given as options

(d)  All of these

(e)  None of these

Answer: (a)

Directions (Q. Nos. 156-160) Read each of the following four sentences A, B, C and D to find out whether there is any grammatical error in it. Choose the sentence with no grammatical error in it. Choose the sentences with no grammatical error in it. Choose the sentence with no grammatical error as the correct answer. If all the given sentences are grammatically correct, mark ‘No error’ as your answer.

156. John wonamaker, founder of the stores that bear his name. once confessed, I learnt 30 year ago that it is foolish to scold.

A. Whether rich or poor, American or Indian, we all have the same emotions deep with us.

B. This research indicate that we can no longer avoid significant warning during this century.

C. Every modern society depends on the trust in the skills and ethics of a variety of institutions such as schools and colleges, hospitals and markets.

(a)  A

(b)  B

(c)  C

(d)  D

(e)  No error

Answer: (c)

157. Many of us have learnt not to make a public display of our emotions, particularly when they are negative.

A. Though Indians have been increasingly taking up corner office in the international companies, Rahul is the first one Indian to lead one of the top four accounting firms globally.

B. Democracy was suspended not because it has become an obstacle to political ambitions trotted out as the nation’s solution.

C. A virtual takeover of a bankrupt civil society be a coterie or individuals whom cornered tremendous power by being able to represent the state.

(a)  A

(b)  B

(c)  C

(d)  D

(e)  No error

Answer: (a)

158. Indian has decided to away with the practice of put each imported consignment through lab checks and switch to the international norm of random and risk based inspections.

A P Sumit singh, along with Devesh Pandy, and DSP Hasan, headed separate terms that include cops from six police stations and investigators from the district police crime cell.

B. Out emissions standards are one-tenth that of global averages and far more stringent than even Europe.

C. The university has received about 500 applicants for the course; out of them 50 will be shortlisted for the programme.

(a)  B

(b)  A

(c)  C

(d)  D

(e)  No error

Answer: (e)

159. RBI lifted the ban upon imports of gold coins and medallions by banks and trading houses.

A. President Pranab Mukherjee inaugurated Indian International Trade fair 2016 in Pragati Maidan in New Delhi.

B. Three Indian, Two women and one man were among the shortlisted 100 applicants whom were chosen for one-way trip to mars under one mission.

C. Fasting not only detoxifies the system but also gives a person training in endurance, a spirit of acceptance and self-control.

(a)  A

(b)  B

(c)  C

(d)  D

(e)  No error

Answer: (d)

160. The Reserve Bank of India (RBI) has notified that rescheduling of payment period of education loans by banks due to unemployment of borrower will not be treated as restricted accounts for computing NPAs.

A. China has started the operations of the longest East-West high speed railways.

B. Former Serbian professional tennis player Ana Ivanovic announced her retirement from tennis at the age of 29 years.

C. The Union Ministry of Sports has suspended the Indian Olympic Association (IOA) on December 30, 2016.

(a)  A

(b)  B

(c)  C

(d)  D

(e)  No error

Answer: (e)

Part V Quantitative Aptitude

 

161. Raj while going by bus from home to airport (without any halt) takes 20 minutes less than time taken when the bus halts for some time. The average speed is 8 km/h more than the average speed of the bus when it halts. If the distance from home to airport is 60km, what is the speed of the bus when it is traveled?

(a)  24

(b)  46

(c)  20

(d)  e

(e)  42

Answer: (e)

162. Every month, Mr. Duggabati spends 24% of his monthly income in paying rent and 30% on shopping of groceries, Out of the remaining, he invests in fixed deposit and the lottery in the respective ratio of 9 : 7 respectively. If in a year he deposited a total of Rs. 124200 in fixed deposit, how much did he pay as rent in a year?

(a)  Rs. 124000

(b)  Rs. 110400

(c)  Rs. 117600

(d)  Rs. 124560

(e)  Rs. 115200

Answer: (e)

163. A boat can travel upstream 13 km and downstream 28 km taking 5 hours each time. The velocity of the current of the current is

(a)  0.5 km/h

(b)  1 km/h

(c)  1.5 km/h

(d)  2 km/h

(e)  2.5 km/h

Answer: (c)

Directions (Q. Nos. 164-168) Read the graph carefully and answer the given questions.

The data given below are regarding percentage of fiction books sold by five book stores in the months of June and December in the year 2002. All the book stores sold either fiction or non-fictions.

Note Total number of books sold = Fiction + Non-fiction

164. Number of fiction books sold by store B in December was 200 more than that sold by the same store in June. If the total number of books sold by the store in both the given months together was 2000, how many books (fiction + non-fiction), did the store sell in December?

(a)  2000

(b)  1600

(c)  1200

(d)  1800

(e)  1500

Answer: (c)

165. Total number of books sold by store C in June was 1500 and that in December was 2000. What is the total number of non-fiction books sold by the store in both the given months together?

(a)  1200

(b)  1720

(c)  1800

(d)  1600

(e)  1900

Answer: (e)

166. Number of fiction books sold by store D in both the given months are equal. Total number of books sold by the same store in December was what percentage more than that in June?

(a)  16%

(b)  18%

(c)  10%

(d)  12.5%

(e)  8%

Answer: (d)

167. Total number of books sold by E in June was 1800 which was 25% less than that sold by the same store in December. What is the total number of fiction books sold by the same store together?

(a)  1442

(b)  1620

(c)  1572

(d)  1640

(e)  1548

Answer: (e)

168. Total number of books sold by store A in December are 60 more than that sold by the same store in June. If the number of fiction books sold b y store A in both the given months were equal, what is the total number of books sold by store in June?

(a)  840

(b)  800

(c)  660

(d)  900

(e)  720

Answer: (c)

Directions (Q. Nos. 169-173) Refer to the pie charts and answer the given questions.

The following data are regarding number of students (boys + girls) studying in standard Xth of various schools (A, B, C, D and E in the year 2010)

169. What is the difference between the total number of students (boys + girls) in school A and B together and the total number of students (boy + girls) in schools D and E together?

(a)  640

(b)  600

(c)  620

(d)  680

(e)  660

Answer: (e)

170. Number of girls studying in school C, are what percent of total number of stu dents (boys + girls) studying in the same school?

(a)  48

(b) 

(c)  50

(d) 

(e) 

Answer: (d)

171. What is the average number of boys studying in school A and B?

(a)  280

(b)  288

(c)  272

(d)  248

(e)  278

Answer: (b)

172. Number of girls studying in school E are approximately what percent more than the number of boys studying in the same school?

(a)  45%

(b)  30%

(c)  50%

(d)  40%

(e)  35%

Answer: (e)

173. What is the respective ratio between the number of boys and the number of girls studying in school D?

(a)  12 : 7

(b)  16 : 9

(c)  16 : 7

(d)  18 : 7

(e)  14 : 9

Answer: (d)

Directions (Q. Nos. 174-178) Studying the following information carefully and answer the given questions.

The following data are regarding total number of male and female employees (MBSs and non-MBSs, working in three organizations A, B and C)

In organization A 30% of the total numbers of employees are females, of which 10% are MBA’s.

In organization B 40% of the total numbers of employees are females, of which 25% are MBAs. 10% of the total male employees in organization B are MBAs. Number of female MBA employees in organization B are 126 more than that of female MBA employees in organization A. The total number of female employees in organization B are equal to that in organization A.

In organization C total number of female employees are twice of that in organization B. 20% of the total number of male employees and 20% of the number of female employees are MBAs. Number of male employees who are MBAs are 672 more than that of female employees who are not MBAs.

174. The respective ratio between the number of male MBA employees, who are science graduates, in organization B and C is 1 : 3. Total number of male MBA employees who are science graduates in both the organization together is 262. What is the number of male MBA employees who are science graduates in organization B?

(a)  87

(b)  104

(c)  64

(d)  92

(e)  Other than those given as options

Answer: (a)

175. In organization A, if 20% of the total male employees are MBAs, then what is the difference between total number of employees who are non-MBA in organization A and B?

(a)  420

(b)  560

(c)  540

(d)  640

(e)  Other than those given as options

Answer: (e)

176. What is the average number of female employees working in organizations A, B and C?

(a)  1160

(b)  1225

(c)  1260

(d)  1080

(e)  1120

Answer: (c)

177. What percent of the total number of organizations in company C, are males?

(a)  60%

(b)  70%

(c)  80%

(d)  75%

(e)  58%

Answer: (e)

178. Total number of employees in organization C, are what percent more than tht in organization A?

(a)  20%

(b)  50%

(c)  25%

(d)  75%

(e)  80%

Answer: (a)

Directions (Q. Nos. 179-183) Study the table and answer the given questions.

Note (I) Few data are missing (indicated by-). You need to calculate the value based on given data, if required, to answer a given question.

(II) Sum total of ratings obtained = Number of raters × Average rating obtained

179. For movie C, the respective ratio of sum total of ratings from city X and that from city Y is 16 : 21. What is the average rating obtained for move C from city Y?

(a)  73.5

(b)  72.5

(c)  75.5

(d)  75

(e)  74

Answer: (a)

180. For movie D, the sum total of ratings obtained from city X and that from city Y are equal. The respective ratio of number of raters in city Y and city Z is 4 : 5. Number of raters in city X is what percent more than number of raters in city Z?

(a)  30%

(b)  15%

(c)  25%

(d)  20%

(e)  Other than those given as options

Answer: (d)

181. For movie B, average rating obtained from city Y is 10% less than that obtained from city X. Sum total of ratings obtained from city Y is what percent more than that obtained from city X?

(a)  35%

(b)  32%

(c)  30%

(d)  40%

(e)  Other than those given as options

Answer: (e)

182. For move E, the average rating given by male raters in cities X and Y are equal and that by female raters in the two cities are equal. If the respective ratio of male and female in city X is 5 : 3 and that in city Y is 2 : 1, what i s the average rating given by male raters of city X?

(a)  28

(b)  36

(c)  32

(d)  30

(e)  Other than those given as options

Answer: (b)

183. Combining city X and city Y, the average rating obtained for movie A is 58. Number of raters in city Y is what percent more than the number of rates in city X?

(a) 

(b) 

(c) 

(d) 

(e)  Other than those given as options

Answer: (e)

184. 18 women complete a project in 24 days and 24 men complete the same project in 15 days. 16 women worked for 3 days and then they left. 20 men work for next 2 days and then they are joined by 16 women. In how many days will they complete the remaining work?

(a) 

(b) 

(c) 

(d) 

(e)  Other than those given as options

Answer: (e)

185. The cost price of article B is 20% more than that of article A. Articles A and B were marked up by 50% and 25% respectively. Article A was sold at a discount of 4% and article B was sold at a discount of ‘0.5%. If the selling price of article A was Rs. 30 more than the selling price of article B, what was the cost price of article A?

(a)  Rs. 600

(b)  Rs. 400

(c)  Rs. 500

(d)  Rs. 540

(e)  Rs. 400

Answer: (c)

186. Ram’s age after 17 years will be 6 years less than twice his present age. The respective ratio between Vivian’s present age and Amit’s present age is 21 : 19. If Amit’s age 31 years hence will be same as three times Ram’s present age, what will be Vivian’s age 4 years hence?

(a)  44 yr

(b)  45yr

(c)  46yr

(d)  48 yr

(e)  Other than those given as options

Answer: (c)

187. Jar has ‘X’ litre of mixture of apple juice and water in the respective ratio of 5 : 1 and jar B has ‘X’ litre of mixture of mango juice and water in the respective ratio of 21 : 30. 30 litre mixture was taken from jar A and jar B each and mixed in jar C. If 12 litre mixture was taken out from jar C, what was the final quantity of water in jar C?

(a)  15 L

(b)  6 L

(c)  4 L

(d)  9 L

(e)  12 L

Answer: (c)

Directions (Q. Nos. 188-192) What approximate value will come in place of question marks in the given questions? (You are not expected to calculate the exact value)

188. 67% of 369.87 ÷ 15.01 = 221 ÷ ?

(a)  7

(b)  18

(c)  13

(d)  21

(e)  10

Answer: (c)

189. ?% of (6274 ÷ 6.14) = 646.51 – 21.58

(a)  60

(b)  45

(c)  39

(d)  65

(e)  74

Answer: (c)

190. 56 * ? ÷ 3 = 576.73 * 2.02

(a)  328

(b)  295

(c)  412

(d)  381

(e)  235

Answer: (a)

191. 

(a)  745

(b)  910

(c)  980

(d)  790

(e)  722

Answer: (d)

192. ?2 * 124.03 – 19.07 * 5.98 = 131

(a)  17

(b)  21

(c)  16

(d)  11

(e)  9

Answer: (d)

Directions (Q. Nos. 193-197) Each question below has a question and two/three statements. You have to decide whether the data provided in the statements are sufficient to answer the question. Choose appropriate option in each case.

193. There are certain number of cards in a bag which are numbered serially (1, 2, 3, 4 and so on). How many cards are there in the bag?

I. If two cards are drawn at random, one after another, without replacement, the probability that both the cards are even numbered is 7/31.

II. The respective ratio between number of odd numbered cards and number of even numbered cards in the bag is 16 : 15.

(a)  Only statement I is sufficient to answer the question.

(b)  Only statement II is sufficient to answer the question.

(c)  Either statement I or II is sufficient to answer the question.

(d)  Both statement I and II are sufficient to answer the question.

(e)  None of the statements is sufficient to answer the question.

Answer: (e)

194. What is the rate of interest per annum?

I. The amount becomes Rs. 9331.20 in 2 years at compound interest.

II. The difference between CI and SI at the same rate of interest in 2 years is Rs. 51.20

III. The amount invested is Rs. 8000.

(a)  Only statement I and III are sufficient to answer the question.

(b)  Only statement I and II are sufficient to answer the question.

(c)  Only statement II and either statement I or III is sufficient to answer the question.

(d)  Only statement III and either statement I or II is sufficient to answer the question.

(e)  Any two of the three statements is sufficient to answer the question.

Answer: (a)

195. Find the number of days in which Q and do a job if P can do the same job in 8 days.

I. Q is 60% more efficient than P.

II. P and Q together can do the job in

III. P is  less efficient than Q.

(a)  Only statement I is sufficient to answer the question.

(b)  Only statement II is sufficient to answer the question.

(c)  Either statement I or II is sufficient to answer the question.

(d)  Any of the statements is sufficient to answer the question.

(e)  Only statements I and III are sufficient to answer the question.

Answer: (c)

196. A and B are two positive integers. What is the sum of the cubes of A and B?

I. The sum of A and B is 14.

II. The sum of the squares of A and B is 106.

III. The value of A is 4 less than the value of B.

(a)  Only statements I and III are sufficient to answer the question.

(b)  Only statement I and II re sufficient to answer the question.

(c)  Only statement II and either statement I or statement III is sufficient to answer the question.

(d)  Only statement III and either statement I or statement II is sufficient to answer the question.

(e)  Any two of the three statements is sufficient to answer the question.

Answer: (854)

197. There are two positive numbers. What is the smallest number?

I. The LCM of two numbers is 24 times of their HCF.

II. The sum of HCF and LCM of two numbers is 400.

III. The bigger number is 126.

(a)  Only statements I and III are sufficient to answer the question.

(b)  Only statements I and II are sufficient to answer the question.

(c)  Only statement II and either statement I or statement III is sufficient to answer the question.

(d)  Only statement III and either statement I or statement II is sufficient to answer the question.

(e)  Any two of the three statements is sufficient to answer the question.

Answer: (49)

198. Circle C1 is inscribed inside a square, which in turn in inscribed inside circle C2. The sum of circumference of both the circles is 88(1 + σ2) cm. What is the perimeter of the square?

(a)  120 cm

(b)  102 cm

(c)  116 cm

(d)  112 cm

(e)  Other than those given as options

Answer: (e)

199. Sum of money was invested for ‘T’ years in Scheme A offering simple interest. The amount received after ‘T’ years was twice the sum of money invested in the scheme. What will be amount received for scheme A, when a sum of Rs. 5450 is invested for ‘2T’ years?

(a)  Rs. 16290

(b)  Rs. 15500

(c)  Rs. 15050

(d)  Rs. 16350

(e)  Rs. 16500

Answer: (d)

200. A, B and C started a business with investments in the respective ratio of 2 : 3 : 4. At the end of 6 months from the start of the business, A, B and C invested additional amounts, after which their final investments were in the respective ratio of 3 : 2 : 1. If the ratio between B’s and C’s share in annual profit was 8 : 9 and the difference between A’s and B’s share in annual profit was Rs. 200, what was the amount that A invested initially?

(a)  Rs. 400

(b)  Rs. 500

(c)  Rs. 600

(d)  Rs. 200

(e)  Other than those given as options

Answer: (d)

IBPS RRBs-V Officer (Scale I) Common Recruitment Process Preliminary Online Examination Held on November 4, 2016

IBPS RRBs-V Officer (Scale I) Common Recruitment Process Preliminary Online Examination 2016
IBPS RRBs-V Officer (Scale I) Common Recruitment Process Preliminary Online Examination Held on November 4, 2016

IBPS RRBs-V Officer (Scale I) Common Recruitment Process Preliminary Online Examination

Held on November 4, 2016

Part I Reasoning Ability

1. If only one meaningful English word can be made with the first, the fourth, the ninth and the tenth letters of the word CORRUPTION, using each letter only once, then which of the first letter of the word from the left. If no such word can be formed then your answer is X. If more than one such word can be formed then your answer is Z?

(a)  X

(b)  Z

(c)  R

(d)  O

(e)  C

Ans: (e)

Directions (Q. Nos. 2-6) In these questions, relationship between different elements is shown in the statements. The statements are followed by conclusions I and II. Study the conclusions based on the given statements and select the appropriate option.

a. if only conclusion I is true

b. if only conclusion II is true

c. if both conclusions are true

d. if none of the conclusions is true

e. if either conclusion I or II is true

2. Statements P < E < T ≤ R; T ≥ K

Conclusions  I. K > P                II. R ≥ K

Ans: (b)

3. Statements X < W; A > C ≥ H = W

Conclusions   I. C > X               II. B < W

Ans: (c)

4. Statements B ≤ N < D; K = R < D ≤ W

Conclusions  I. N = R                II. B < W

Ans: (b)

5. Statements J ≤ L < B ≤ S > Y < M

Conclusions  I. J < M                II. L ≥ Y

Ans: (d)

6. Statements B ≤ N < D; K = R < D ≤ W

Conclusions  I. W > K               II. N ≥ W

Ans: (a)

Directions (Q. Nos. 7-11) Study the given information carefully to answer the given questions.

Eight people- A< B, C, D, M, N, O and P are sitting around a square table (but not necessarily in the same order) in such a way that four of them sit at four corners while four sit in the middle of each of the four sides. The ones sitting in the middle of the sides are facing the centre and the ones sitting at the corners are facing outside (i.e., opposite to the centre)

A sits in the middle of one of the sides. Only one person sits between AA and M. A sits third to the right of B. Only three people sit between B and N. C sits second to the right of N. O and C face the same direction. M is not an immediate neighbour of O. P sits second to the right of D.

7. Who is to the immediate left of M?

(a)  N

(b)  B

(c)  D

(d)  P

(e)  C

Ans: (e)

8. What is the position of O with respect to P?

(a)  Fourth to the left

(b)  Third to the left

(c)  Third to the right

(d)  Immediate right

(e)  Immediate left

Ans: (b)

9. Which of the following statements is true with respect to the given arrangement?

(a)  None of the given statements is true.

(b)  Only three people sit between O and D.

(c)  D sits third to the left of A.

(d)  M sits at one of the corners of the table.

(e)  M is an immediate neighbour of P.

Ans: (a)

10. As per the given arrangement, four of the following five are alike in a certain way and hence form a group. Which of the following does not belong to the group?

(a)  D

(b)  O

(c)  P

(d)  A

(e)  M

Ans: (b)

11. Who amongst the following sit exactly between B and the one who sits to the immediate right of N, when counted from the left of B?

(a)  O, D

(b)  M, P

(c)  C, P

(d)  M, C       

(e)  A, C

Ans: (c)

12. How many such pairs of digits are there in the number 67594138 (both in forward and backward directions), each of which has as many digits between them as in the arithmetic series?

(a)  Two

(b)  None

(c)  One

(d)  Three

(e)  More than three

Ans: (e)

13. In a queue of fifteen people facing North, Rahul’s position is ningth from the end of the queue. Only four people are standing between Rahul and Tom, karan is standing immediately after Tom. Bharti is standing exactly between Karan and Rahul. Sonali is standing before Bharti, but after Tom. What is position of Sonali from the beginning of the queue? (Note : All people are standing one behind the other)

(a)  Cannot be determined

(b)  Seventh

(c)  Eleventh

(d)  Fourth

(e)  Sixth

Ans: (d)

14. The following series is based on the English alphabets. Which one of the following will come in place of the question mark in th given alphabetical series?

XBF  UDK  RFO  OHR ?

(a)  LKU

(b)  MKS

(c)  LJT

(d)  MJS

(e)  LJV

Ans: (c)

Directions (Q. Nos. 15-17)  Study the given information carefully to answer the given questions.

S is 11 m East of N. S is 8 m North of P. P is 4 m West of O. Point O is the midpoint of points P and R, such that P, O and R form a straight line. Q is 13 m South of R.

15. If L is 7 m East of R and J is 5 m South of L, then what is the distance between L and S?

(a)  6 m

(b)  10 m

(c)  17 m

(d)  5 m

(e)  3 m

Ans: (c)

16. In which direction is N with respect to Q?

(a)  North-West

(b)  West

(c)  East

(d)  North-East

(e)  South-West

Ans: (a)

17. Hiten walks 2 m towards North form point P, takes a right turn and walks for 8 m. How far will he be from point Q?

(a)  13 m

(b)  7 m

(c)  17 m

(d)  8 m

(e)  15 m

Ans: (e)

Directions (Q. Nos. 18-22) Study the following information to answer the given questions.
Nine persons, E, F, G, H, K, I, L, M, N, and O are seated in a straight line facing North, with equal distance between each other, but not necessarily in the same order.

Only two people sit between E and the one sitting at extreme ends of the line. K sits second to the right of E. H sits fourth to the left of M. M does not sit at any of the extreme ends of the line. M is not an immediate neighbour of E. The number of people sitting between H and K is double than that between M and O. More than two people sit between G and E. G is not an immediate neighbour o M. F is an immediate neighbour of L, but not H.

18. Four of the following five are alike in a certain way based on the given arrangement and thus form a group. Which is the one that does not belong to that group?

(a)  N, E

(b)  M, H

(c)  K, L

(d)  O, N

(e)  F, O

Ans: (d)

19. Which of the following is true with respect to N as per the given arrangement?

(a)  N is an immediate neighbour of H.

(b)  N sits second to the left of M.

(c)  N sits at one of the extreme ends of the line.

(d)  More than two people sit between N and G.

(e)  None of the given options is true.

Ans: (e)

20. Who sits second to the left of H?

(a)  N

(b)  K

(c)  F

(d)  E

(e)  No one as H sits at one of the extreme ends of the line.

Ans: (c)

21. Who amongst the following sit exactly between N and O?

(a)  H, L

(b)  F, O

(c)  F, E

(d)  M, K

(e)  K, O

Ans: (d)

22. In which of the given pairs of people, is even number of people sitting between them?

(a)  G, E

(b)  K, L

(c)  F, M

(d)  O, M

(e)  K, N

Ans: (a)

Directions (Q. Nos. 23-27) Study the following information to answer the given questions.

Sagar sells mobiles of seven different companies viz. Samsung, HTC, Lenovo, Intex, Micromax, Nokia and Oppo starting from Monday and ending on Sunday, but not necessarily in the same order.

Sagar sold HTC on Monday. He sold only three mobiles between HTC and Samsung. He does not sell any mobile between the day he sold Samsung and Lenovo. He sold only two mobiles between Lenovo and Oppo. He sold Micromax mobiles the day immediately before the day he sold Oppo mobiles. He sold Intex on one of the days after Oppo, but not on Sunday.

23. On which day Sagar sold Micromax?

(a)  Thursday

(b)  Tuesday

(c)  Friday

(d)  Saturday

(e)  Wednesday

Ans: (b)

24. How many mobiles did Sagar sell between HTC and Lenovo?

(a)  Four

(b)  Two

(c)  One

(d)  None

(e)  Three

Ans: (a)

25. Which of the following is not true as per the given arrangement?

(a)  Sagar sells mobiles of anyone company between Samsung and Nokia.

(b)  Sagar sells Intex on Thursday.

(c)  Sagar did not sell mobile of any other company between Oppo and Intex.

(d)  All the given statements are true.

(e)  Sagar sells Lenovo on Friday.

Ans: (e)

26. Four of the following five are alike in a certain way based on the given arrangement and thus form a group. Which is the one that does not belong to that group?

(a)  Nokia-Friday

(b)  Samsung-Thursday

(c)  Micromax-Monday

(d)  Intex-Wednesday

(e)  Oppo-Tuesday

Ans: (a)

27. Which mobile did Sagar sell on Sunday?

(a)  Nokia

(b)  Lenovo

(c)  Micromax

(d)  None of those given as options

(e)  Oppo

Ans: (a)

Directions (Q. Nos. 28-32) In these questions, two/three statements followed by two conclusions are given. You have to take the given statements to be true even if they seem to be at variance from commonly known facts and then decide, which of the given conclusions logically follows disregarding commonly known facts.

a. if only conclusion I follows

b. if only conclusion II follows

c. if either conclusion I or II follows

d. if neither conclusion I or II follows

e. if both conclusions I and II follow

28. Statements All copies are book.

            No book is a pencil.

            All pencils are rubbers.

Conclusions

I. No rubber is a book.

II. Some copies are rubbers.

Ans: (d)

29. Statements Some drinks are eatables.

                    All eatables are sweets.

Conclusions

I. At least some sweets are drinks.

II. All drinks are sweets.

Ans: (a)

30. Statements All copies are book.

                    No book is a pencil.

                    All pencils are rubbers.

Conclusions

I. No copy is a pencil.

II. Some books are rubbers.

Ans: (a)

31. Statements Some mobiles are calculators.

                   Some calculators are pens.

                   Some pens are scales.

Conclusions

I. No calculator is a scale.

II. At least some calculators are scales.

Ans: (c)

32. Statements Some mobiles are calculators.

                     Some calculators are pens.

                    Some pens are scales.

Conclusions

I. Some mobiles are sales.

II. All pens being calculators is a possibility.

Ans: (b)

Directions (Q. Nos. 33-37) Study the following information and answer the questions.

Nine friends-A, B, C, D, E, F, G, H and I live on nine different floors of a building, but not necessarily in the same order. The lower most floor of the building is numbered one, the one above that is numbered two and, so on till the topmost floor is numbered nine. I lives on floor numbered six.

E lives on an odd numbered floor above, I. Only three people live between E and G. A lives on an even numbered floor immediately below D, but not on the floor numbered eight. Only one person lives between A and F. C lives on one of the floors below F. The number of people living above C is equal to the number of people living below H.

33. Four of the following five are alike in a certain way as per the given arrangement and thus form a group. Which of the following does not belong to that group?

(a)  HB

(b)  FC

(c)  DE

(d)  EI

(e)  AG

Ans: (c)

34. How many persons live between A and the person living on the floor numbered seven?

(a)  More than three

(b)  Two

(c)  One

(d)  None

(e)  Three

Ans: (b)

35. H lives on which of the following floor numbers?

(a)  Eight

(b)  One

(c)  Five

(d)  Nine

(e)  Other than those given as options

Ans: (d)

36. Which of the following is not true about B as per the given arrangement?

(a)  B lives on an even numbered floor.

(b)  All the given statements are true.

(c)  Only one person lives between B and E

(d)  H lives immediately above B.

(e)  Only one person lives above B.

Ans: (c)

37. In which of the given pairs of people, is even number of people living between them?

(a)  G, C

(b)  H, E

(c)  E, D

(d)  I, H

(e)  B, I

Ans: (d)

Directions (Q. Nos. 38-40) Study the following information carefully to answer the given questions.

S is the husband of Q. A is the brother of D. A is the only son of B. D is the sister of Q. R is married to D. M is the father of R. N is the daughter of Q.

38. If V is the grandfather of N, then how is B related to R?

(a)  Uncle

(b)  Mother-in-law

(c)  Grandmother

(d)  Aunt

(e)  Father-in-law

Ans: (e)

39. How is S related to A?

(a)  Father

(b)  Grandfather

(c)  Brother-in-law

(d)  Uncle

(e)  Nephew

Ans: (c)

40. How is D related to N?

(a)  Mother

(b)  Sister-in-law

(c)  Cousin

(d)  Mother-in-law

(e)  Aunt

Ans: (e)

Part II Quantitative Aptitude

Directions (Q. Nos. 41-45) What approximate value will come in place of question marks in the given equations?

(You are not expected to calculate the exact value)

41. 9 ÷ 4.02 + 13.04 × 28.978 = ?

(a)  1430

(b)  1470

(c)  1410

(d)  1490

(e)  1240

Ans: (*)

42. ?% of (767 ÷ 6) = 7.8892

(a)  25

(b)  60

(c)  40

(d)  50

(e)  75

Ans: (d)

43. 89 – 15.03 ÷ 3.693 = 1039.88 ÷ ?

(a)  6

(b)  8

(c)  4

(d)  2

(e)  12

Ans: (c)

44. 99 × 4.98 + 129.992 – 75.05 = ?

(a)  540

(b)  780

(c)  680

(d)  620

(e)  760

Ans: (c)

45. 

(a)  35

(b)  510

(c)  25

(d)  15

(e)  20

Ans: (c)

46. Jar A contains 78 L of milk and water in the respective ratio of 6 : 7. 26 L of the mixture was taken out from jar A. What quantity of milk should be added to jar A, so that water constitutes 40% of the resultant mixture in jar A?

(a)  8 L

(b)  361 L

(c)  12 L

(d)  14 L

(e)  18 L

Ans: (e)

47. At its usual speed, a 150 m long train crosses a platform of length L m in 24 seconds. At 75% of its usual speed, the train crosses a vertical pole in 12 seconds. What is the value of L ?

(a)  250

(b)  225

(c)  240

(d)  260

(e)  280

Ans: (a)

48. In a class, the respective ratio between the number of boys and the number of girls is 3 : 1. A test was conducted, where in the average score of the boys was 73, while that of the entire class was 71. What was the average score of the girls?

(a)  68

(b)  71

(c)  67

(d)  65

(e)  63

Ans: (d)

49. Two years ago, A’s age was 1/2 of B’s age at that time. A’s age four years hence will be 22 years less than B’s age eight years hence. What is B’s present age?

(a)  30 yr

(b)  24 yr

(c)  36 yr

(d)  42 yr

(e)  38 yr

Ans: (e)

Directions (Q. Nos. 50-54) In these questions, two equations numbered I and II are given. You have to solve both the equations and choose the appropriate option.

a. x > y                 b. x , y        c. x ≥ y

d. x ≤ y      e. x = y or cannot be determined

50. I. 3x2 + 14x + 15 = 0 II. 2y2 + 19y + 44 = 0

Ans: (a)

51. I. x2 = 196 II. y2 + 2y – 48 = 0

Ans: (e)

52. I. 2x2 + 17x + 36 = 0 II. 3y2 + 20y + 33 = 0

Ans: (b)

53. I. x2 + 12x + 35 = 0 II. 3y2 + 19y + 20 = 0

Ans: (d)

54. I. x2 – 7x + 10 = 0 II. y2 – 5y + 6 = 0

Ans: (c)

55. When 9 is subtracted from a two digit number, the number so formed is reverse of the original number. Also, the average of the digits of the original number. Also, the average of the digits of the original number is 7.5. What is definitely the original number?

(a)  87

(b)  92

(c)  90

(d)  69

(e)  96

Ans: (a)

56. A, B and C have a certain amount of money with themselves. C has 3/4 of what A has and B has Rs. 50 less than C. If A, B and C together have Rs. 250, then how much does A alone have?

(a)  Rs. 75

(b)  Rs. 160

(c)  Rs. 80

(d)  Rs. 120

(e)  Rs. 140

Ans: (d)

57. A boat takes six hours to travel a certain distance downstream and five hours to travel a certain distance upstream. The distance travelled upstream is half of the travelled downstream. If the speed of the current is 4 km/h, what is the speed of the boat in still water?

(a)  16 km/h

(b)  20 km/h

(c)  24 km/h

(d)  10 km/h

(e)  18 km/h

Ans: (a)

58. It takes Rs. 3159 to plant synthetic grass in a square lawn, 1/4 of which is paved (and thus does not require grass.) If each side of this lawn measures 18m, what is the rate that the gardener charges for planting synthetic grass?

(a)  Rs. 18 m2

(b)  Rs. 11 m2

(c)  Rs. 16 m2

(d)  Rs. 15 m2

(e)  Rs. 13 m2

Ans: (e)

Directions (Q. Nos. 59-63) Refer to the pie charge and answer the given questions.

59. What is the difference between the total number of tickets sold by theatres A and D together and then sold by theatres B and C together?

(a)  82

(b)  78

(c)  55

(d)  75

(e)  85

Ans: (e)

60. The number of tickets sold by theatre B, on Saturday, was 32 more than the number of tickets sold on Friday. The number of tickets sold by theatre E, on Saturday, was 59 more than the number of tickets sold on Friday. What was the respective ratio between the number of tickets sold by theatres B and E on Saturday?

(a)  5 : 3

(b)  7 : 6

(c)  11 : 6

(d)  7 : 5

(e)  11 : 5

Ans: (b)

61. If the total number of tickets sold by all the theatres together on Friday was 70% more than that sold on Thursday, what was the total number of tickets sold by all the theatres together on Thursday?

(a)  400

(b)  450

(c)  500

(d)  300

(e)  550

Ans: (c)

62. What is the central angle corresponding to the number of tickets sold by theatre B?

(a)  56.8°

(b)  57.6°

(c)  58.5°

(d)  55.6°

(e)  54.2°

Ans: (b)

63. Out of the number of tickets sold by theatre C, 5/12 were purchased by females and out of the total number of tickets sold by theatre D, 1/17 were purchased by females. What was the total number of tickets purchased by females from theatres C and D together?

(a)  100

(b)  108

(c)  110

(d)  120

(e)  104

Ans: (a)

Directions (Q. Nos. 64-67) Based on the following table, answer the given questions.


Note The institutes offer classes for French and Spanish languages only.

64. The number of male students, who have enrolled for French classes in institute R is what percent of the total number of students (male and female) who have enrolled for both the classes in institute T?

(a)   

(b)   

(c)   

(d)   

(e)   

Ans: (e)

65. In institutes S, the percentage of male students, who have enrolled for Spanish class, is 70% of the total number of students (Male and female both) who have enrolled for Spanish class. What is the total number of male students, who have enrolled for both the classes together in institute S?

(a)  413

(b)  419

(c)  405

(d)  423

(e)  428

Ans: (*)

66. The number of male students, who have enrolled for French class in institute S is approximately, what percent more than the number of female students, who have enrolled for the same class in institute U?

(a)  25%

(b)  44%

(c)  38%

(d)  51%

(e)  Other than those given as options

Ans: (d)

67. What is the respective ratio between number of students (both male and female) who have enrolled for Spanish class in institute R and those enrolled for the same classes in institute U?

(a)  11 : 9

(b)  16 : 7

(c)  15 : 4

(d)  18 : 7

(e)  Other than those given as options

Ans: (e)

68. A bag contains 20 tickets numbered from 1 to 20. Two tickets are drawn at random. What is probability that both numbers are prime?

(a)  8/20

(b)  14/95

(c)  7/20

(d)  21/190

(e)  21/25

Ans: (a)

Directions (Q. Nos. 69-73) What will come in place of question mark in the given number series?

69. 10  11  15  24  40  ?

(a)  90

(b)  87

(c)  114

(d)  95

(e)  65

Ans: (e)

70. 25  35  49  67   89  ?

(a)  24

(b)  81

(c)  115

(d)  107

(e)  93

Ans: (c)

71. 11  9  15  41  159

(a)  740

(b)  607

(c)  751

(d)  709

(e)  789

Ans: (e)

72. 5  10.5  16.5  28.5  48.5  ?

(a)  57.5

(b)  78.5

(c)  93.5

(d)  64.5

(e)  85.5

Ans: (b)

73. 18  9  9  18  72  ?

(a)  460

(b)  372

(c)  576

(d)  484

(e)  380

Ans: (c)

Directions (Q. Nos. 74-78) Refer to the graph and answer the given questions.

74. What is the respective ratio between total number of shirts sold by store B in January and February together and that by the same store in April and May together?

(a)  10 : 9

(b)  7 : 5

(c)  5 : 3

(d)  3 : 2

(e)  Other than those given as options

Ans: (e)

75. What is the average number of shirts sold by store A in January, April and May?

(a)  270

(b)  220

(c)  260

(d)  240

(e)  230

Ans: (d)

76. Number of shirts sold by store A increased by what percent from February to May?

(a)  

(b)   

(c)   

(d)   

(e)   

Ans: (b)

77. If the total number of shirts sold by store A and B together in June is 120 more than that sold in March, what was the total number of shirts sold by stores A and B together in June?

(a)  680

(b)  690

(c)  650

(d)  670

(e)  660

Ans: (d)

78. If the total number of shirts sold by stores A and B together in December is 25% less than that in April, what is the total number of shirts sold by stores A and B together in December?

(a)  415

(b)  400

(c)  405

(d)  395

(e)  425

Ans: (c)

79. A starts a business with Rs. 2500. After one month from the start of the business, B joined with Rs. 4500 and A withdrew completely after eleven months from the start of the business. If the difference between A’s and B’s respective shares in the annual profit was Rs. 4800, what was the annual profit earned?

(a)  Rs. 14800

(b)  Rs. 15800

(c)  Rs. 14400

(d)  Rs. 11400

(e)  Rs. 15600

Ans: (c)

80. A completes 5/6th of a given task in 10 days and is then replaced by B. The entire task is completed in 13 days. What is the respective ratio of the number of days in which A and B independently can complete the entire task?

(a)  2 : 7

(b)  3 : 8

(c)  1 : 4

(d)  2 : 3

(e)  6 : 11

Ans: (d)

IBPS CWE-VI PO/MT Preliminary Online Examination Held on October 23, 2016 Question Paper with Answer Key

IBPS CWE-VI PO & MT Preliminary Online Examination 2016
IBPS CWE-VI PO/MT Preliminary Online Examination Held on October 23, 2016 Question Paper with Answer Key

IBPS CWE-VI PO/MT Preliminary Online Examination Held on October 23, 2016

Part I Reasoning Ability

Directions (Q. Nos. 1-5) In the following questions, relationships between different element is shown in the statements. The statements are followed by conclusions. Study the conclusions based on the given statements and select the appropriate answer.

a. if only conclusions II is true

b. if only conclusion I is true

c. if both conclusions are true

d. if either conclusion I or II is true

e. if neither conclusion I nor II is true

1. Statements S ≤ L ≤ I = P ≥ E > R; L > Q

Conclusions  I. P ≥ S                II. I > R

Ans: (c)

2. Statements G > R ≤ E = A ≤T ≤ S; D ≤ A ≤ J

Conclusions  I. T ≥ D      II. R > S

Ans: (b)

3. Statements A ≥ B > C ≤ D ≤ E < F

Conclusions  I. A ≥ E      II. C < F

Ans: (a)

4. Statements G > R ≥ E = A ≤ T ≤ S; D ≤ A ≤ J

Conclusions  I. J > G       II. J = G

Ans: (e)

5. Statements S ≤ L ≤ I = P ≥ E > R; L > Q

Conclusions  I. L < R       II. E ≥ Q

Ans: (e)

Directions (Q. Nos. 6-10) Study the following information carefully to answer the questions based on it.

A, B, C, D, W, X, Y and Z are sitting is a circle. (But not necessary in the same order) there faces are on the centre. W is sitting third to the left of Y. The person, who is from Dwarka is to the immediate right of W and W is not from Okhla. B is sitting fourth to the right of Z. Z is not the neighbour of Y. Neither B nor Z is an immediate neighbour of W. X is from Chanakyapuri and is sitting third to the right of the person from Dwarka. The person from Mehrauli is sitting second to the left of person from Chanakyapuri. The persons from Rohini is sitting second to the left of W. A, who is from Lajpat Nagar, is sitting exactly between X and Z. The person from Saket is sitting second to the right of the person from Lajpat Nagar, C is sitting third to the left of X.

6. Who amongst the following persons belongs to Okhla?

(a)  Y

(b)  D

(c)  C

(d)  B

(e)  Z

Ans: (a)

7. What is A’s position with respect to B?

(a)  Third to the right

(b)  Second to the right

(c)  Third to the left

(d)  Second to the left

(e)  Fourth to the right

Ans: (a)

8. How many people are sitting between Z and C when counted in an anticlockwise direction From C?

(a)  One

(b)  Two

(c)  Three

(d)  Four

(e)  None

Ans: (d)

9. Four of the following five pairs are alike in a certain way based on their positions in the given arrangement and, so form a group. Which of the following does not belong to the group?

(a)  B – Rohini

(b)  Z – Mehrauli

(c)  D – Okhla

(d)  Y – Saket

(e)  X – Dwarka

Ans: (e)

10. Which of the following statements is false according to the mentioned arrangement?

(a)  C is to the immediate right of the person from Karol Bagh.

(b)  The person from Lajpat Nagar is third to the right of the person from Mehrauli.

(c)  The person from Dwarka is sitting exactly between the person from Karol Bagh and the person from Saket.

(d)  D is neither from Chanakyapuri nor from Karol Bagh.

(e)  There are only three people between A and C.

Ans: (c)

Directions (Q. Nos. 11-15) In each question below are given two/three statements followed by two conclusions I and II. You have to take the given statements to be true even if they seem to be at variance with commonly known facts. Read all the conclusions and then decide, which of the given conclusions logically follows from the given statements, disregarding commonly known facts.

Given answer

a. if only conclusion I follows

b. if only conclusion II follows

c. if either conclusion I or II follows

d. if neither conclusion I nor II follows

e. if both conclusions I and II follow

11. Statements All circles are triangles.

                          Some triangles are rectangles.

                           All rectangles are squares.

Conclusions  I. All rectangles being triangles is possibility.

                       II. All circles being square is a possibility.

Ans: (d)

12. Statements Some chairs are tables.

                           Some bed are tables.

                            No furniture is bed.

Conclusions  I. All chairs being furniture is a possibility.

                       II. Some tables are not bed is possibility.

Ans: (b)

13. Statements All circles are triangles.

                           Some triangles are rectangles.

                            All rectangles are squares.

Conclusions  I. Some triangles are not rectangles.

                        II. No square is a circle.

Ans: (e)

14. Statements All arts are theatre.

                           Some arts are drama.

Conclusions  I. All drama being theatre is a possibility

                        II. Some dramas are theatre.

Ans: (b)

15. Statements Some chairs are table.

                           Some bed are tables.

Conclusions  I. Some tables are not furniture.

                       II. All tables being furniture is a possibility.

Ans: (a)

Directions (Q. Nos. 16-18) Read the given information carefully and answer the given questions.

Point B is 25 m South of point A. Point C is 10 m East of point B. Point D is 30 m North of point C. Point E is 7 m East of point D. Point X is 18 m South of point E. Point M is 12 m South of point X. Point C is 7 m West of point M.

16. B is in which direction from point D?

(a)  South

(b)  South-West

(c)  North-West

(d)  South-East

(e)  North

Ans: (b)

17. If point W is 3 m to the North of point A, then what is the distance between point B and point W?

(a)  28 m

(b)  15 m

(c)  22 m

(d)  24 m

(e)  17 m

Ans: (a)

18. What is distance between point B and point M?

(a)  17 m

(b)  15 m

(c)  21 m

(d)  19 m

(e)  13 m

Ans: (a)

Directions (Q. Nos. 19-23) Study the following information to answer the given questions.

S, T, U, V, W, X, Y and Z are sitting in a straight line equidistant from each other (but not necessarily in the same order). Some of them are facing South While some are facing North.

S faces North. Only two people sit to the right of S. T sits third to the left of S. Only one person sits between T and X. X sits to the immediate right of W. Only one person sits between W and Z. Both the immediate neighbours of T face the same direction. U sits fourth to the left of X. T faces the opposite direction as S. Y does not sit at any of the extremes ends of the line. V faces the same direction as W. Both Y and U face the opposite direction of Z.

19. How many person in the given arrangement are facing North?

(a)  More than four

(b)  Four

(c)  One

(d)  Three

(e)  Two

Ans: (d)

20. Four of the following five are alike in a certain way, and so form a group. Which of the following does not belong to the group?

(a)  W, X

(b)  Z, Y

(c)  T, S

(d)  T, Y

(e)  V, U

Ans: (a)

21. What is the position of X with respect to Z?

(a)  Second to the left

(b)  Third to the right

(c)  Third to the left

(d)  Fifth to the right

(e)  Second to the right

Ans: (b)

22. Who amongst the following sits exactly between Z and W?

(a)  T

(b)  Y

(c)  X

(d)  W

(e)  U

Ans: (a)

23. Who is sitting 2nd to the right of T?

(a)  Z

(b)  V

(c)  X

(d)  W

(e)  None of these

Ans: (c)

Directions (Q. Nos. 24-26) Study the following information and answer the given questions.

D is daughter of N. E is wife of N. G is sister of D. C is married to G. N has no son. K is m other of E. Q is only daughter of C.

24. How Q is related to D?

(a)  Daughter

(b)  Cousin

(c)  Niece

(d)  Sister in law

(e)  Cannot be determined

Ans: (c)

25. How N is related to K?

(a)  Brother-in-law

(b)  Cousin

(c)  Son-in-law

(d)  Sister

(e)  Brother

Ans: (c)

26. How many daughter N have?

(a)  One

(b)  Three

(c)  Two

(d)  Cannot be determined

(e)  None of these

Ans: (c)

Directions (Q. Nos. 27-29) Study the following information and answer the given question.

There are six wires in an extension box A, B, C, D E and F they have different length, but not necessarily in the same order. E is greater than C, but less than D and B. A is greater than D and B. A is not longest wire. F is 13 cm long E is 4 cm long.

27. If D is 5 cm less than F what would be the length of D?

(a)  7 cm

(b)  8 cm

(c)  9 cm

(d)  Can’t be determined

(e)  None of these

Ans: (b)

28. Which wire has least length?

(a)  B

(b)  A

(c)  C

(d)  E

(e)  None of these

Ans: (c)

29. If A is 10 cm length and B carry 5 cm length, then what would be the length of C?

(a)  6 cm

(b)  2 cm

(c)  7 cm

(d)  9 cm

(e)  None of these

Ans: (b)

Direction (Q. Nos. 30-35) Study the given information carefully to answer the given questions.

M, N, O, P, Q, R and S are seven people live on seven different floors of a building but not necessarily in the same order. The lower most floor of the building is numbered 1, the one above that is numbered 2 and so on till the topmost floor is numbered 7. Each one of them have different income i.e. Rs. 3500, Rs. 15000, Rs. 7500, Rs. 9000, Rs. 11000, Rs. 13500 and Rs. 5000. (But not necessarily in the same order.) M lives on an odd numbered floor, but not on the floor numbered 3.

The one, how has income of Rs. 11000 lives immediately above M. only two people live between M and the one who has income of 7500. The one, who has income of Rs. 15000, lives on one of the odd numbered floors above P. Only three people live between O and the one who has income of Rs. 3500. The one who has income of 7500 lives immediately above O. The one who has income of Rs. 3500 lives immediately above the one who has income of Rs. 5000. S lives on an odd numbered floor. Only one person lives between N and Q. N lives on the one of the floors above Q. Neither O nor M has income of Rs.9000. Q does not has income of Rs. 7500.

30. How much income M has?

(a)  Rs. 13500

(b)  Rs. 5000

(c)  Rs. 7500

(d)  Rs. 15000

(e)  Rs. 3500

Ans: (e)

31. Which of the following combinations is true with respect to the given arrangement?

(a)  Rs. 3500-O      

(b)  Rs. 15000-R

(c)  Rs. 5000-S

(d)  Rs. 11000-P

(e)  Rs. 9000-N

Ans: (b)

32. If all the people are made to sit in alphabetical order from top to bottom, the positions of how many people will remain unchanged?

(a)  Four

(b)  None

(c)  Two

(d)  One

(e)  Three

Ans: (d)

33. Which of the following statements is false with respect to the given arrangement?

(a)  The one who has income of 5000 lives immediately below M.

(b)  R has income of 15000.

(c)  None of the given options is true.

(d)  Only four people live between P and S.

(e)  S lives immediately below Q.

Ans: (e)

34. Who amongst the following lives on the floor numbered 2?

(a)  N

(b)  The one who has income of Rs. 3500.

(c)  The one who has income of Rs. 5000.

(d)  P

(e)  R

Ans: (d)

35. How much income R has?

(a)  Rs.13500

(b)  Rs. 5000

(c)  Rs. 7500

(d)  Rs. 15000

(e)  Rs. 3500

Ans: (d)

Part II Quantitative Aptitude

Directions (Q. Nos. 36-39) Study the table carefully and answer the given questions.


36. If the male population above poverty line for State R is 1.9 million, then, what is the total population of State R?

(a)  4.5 million

(b)  4.85 million

(c)  5.35 million

(d)  6.25 million

(e)  6 million

Ans: (d)

37. What will be the number of females above poverty line in the State S if it is known that the population of State S is 7 million?

(a)  3 million

(b)  2.13 million

(c)  1.33 million

(d)  5.7 million

(e)  4 million

Ans: (b)

38. What is the male population above poverty line for State P, if the female population below poverty line for State P is 2.1 million?

(a)  2.1 million

(b)  2.7 million

(c)  3.3 million

(d)  2.3 million

(e)  3 million

Ans: (c)

39. If the population of males below poverty line for State Q is 2.4 million and that for State T is 6 million, then what is the respective ratio of the total population of State Q and T?

(a)  1 : 3

(b)  2 : 5

(c)  3 : 7

(d)  4 : 9

(e)  3 : 9

Ans: (b)

40. Average runs of a cricketer in 10 innings is 60. How many runs should he make in 11th innings that his average runs would be 62?

(a)  80

(b)  81

(c)  82

(d)  83

(e)  84

Ans: (c)

41. Find the probability that a number from 1 to 300 is divisible by 3 or 7?

(a)  37/75

(b)  32/75

(c)  36/75

(d)  28/75

(e)  26/75

Ans: (b)

42. 14 men can do a work in 18 days, 15 women can do a work in 24 days. If 14 men work for first 3 days and 10 women work after that for 3 days, find the part of work left after that?

(a)  3/4

(b)  1/4

(c)  1/2

(d)  1/6

(e)  1/5

Ans: (a)

43. Perimeter of a rectangle is x and circumference of a circle is 8 more than the perimeter of the rectangle. Ratio of radius of circle and length of the rectangle is 1 : 2 and ratio of length and breadth of rectangle is 7 : 3. Find the length of the rectangle?

(a)  14

(b)  21

(c)  28

(d)  35

(e)  7

Ans: (c)

44. A invest on some scheme at 5% and B at 3% for two year. If the total sum invested by A and B is Rs. 4000 and the simple interest received by both is same then find the amount invested by A?

(a)  Rs. 1300

(b)  Rs. 1500

(c)  Rs. 2500

(d)  Rs. 2700

(e)  Rs. 2100

Ans: (b)

45. Two trains crosses each other in 14 seconds when they are moving in opposite direction, and when they are moving in same direction, they crosses each other in 3 minute 2 seconds. Find the speed of the faster train by what percent more than the speed of the slower train?

(a)  16.67%

(b)  17.33%

(c)  16.33%

(d)  17.67%

(e)  18.33%

Ans: (a)

Directions (Q. Nos. 46-50) What will come in place of the question marks in the following number series?

46. 11  20  38  74

(a)  146 

(b)  154

(c)  128

(d)  132

(e)  136

Ans: (a)

47. 15  21  38  65  101  ?

(a)  124

(b)  145

(c)  136

(d)  158

(e)  162

Ans: (b)

48. 24  28  19  35  10  ?

(a)  26

(b)  36

(c)  16

(d)  46

(e)  15

Ans: (d)

49. 7  16  45  184  915  ?

(a)  2092

(b)  5496

(c)  1048

(d)  4038

(e)  3268

Ans: (b)

50. 12  19  35  59  90  ?

(a)  134

(b)  127

(c)  132

(d)  98

(e)  114

Ans: (b)

Directions (Q. Nos. 51-55) Study the following graph and answer the questions that follow.


51. The incomes of company X and Y in 2010 were in the ratio of 3 : 4 respectively. What was the respective ratio of their expenditure in 2010?

(a)  7 : 22

(b)  14 : 19

(c)  15 : 22

(d)  27 : 35

(e)  35 : 49

Ans: (c)

52. If the expenditure of company Y in 2007 was Rs. 220 crore, what was its income in 2007?

(a)  Rs. 312 crore

(b)  Rs. 297 crore

(c)  Rs. 283 crore

(d)  Rs. 275 crore

(e)  Rs. 250 crore

Ans: (b)

53. If the expenditures of company X and Y in 2006 were equal and the total income of the total companies in 2006 was Rs. 342 crore, what was the total profit of two companies together in 2006?

(Profit = Income – Expenditure)

(a)  Rs. 240 crore

(b)  Rs. 171 crore

(c)  Rs. 120 crore

(d)  Rs. 102 crore

(e)  Rs. 150 crore

Ans: (d)

54. The expenditure of company X in the year 2008 was Rs. 200 crore and the income of company X in 2008 was the same as it expenditure in 2011. What is the income of company X in 2011?

(a)  Rs. 385 crore

(b)  Rs. 465 crore

(c)  Rs. 335 crore

(d)  Rs. 295 crore

(e)  Rs. 250 crore

Ans: (b)

55. If the incomes of two companies were equal in 2009, then what was the ratio of expenditure of company X to that of company Y in 2009?

(a)  6 : 5

(b)  5 : 6

(c)  11 : 6

(d)  16 : 15

(e)  20 : 11

Ans: (d)

Directions (Q. Nos. 56-60) In each of these questions, two equations I and II are given. You have to solve both the equations and give answer.

a. if x > y    b. if x ≥ y   c. d=if x < y

d. if x ≤ y

e. if x = y or relationship between x and y cannot be established

56. I. 3x2 – 22x + 7 = 0        II. y2 – 15y + 56 = 0

Ans: (d)

57. I. 2x2 – 17x + 36 = 0 II. 2y2 – 19y + 44 = 0

Ans: (d)

58.       II. y2 – 169 = 0

Ans: (e)

59. I. 3x2 + 20x + 25 = 0 II. 3y2 + 14y + 8 = 0

Ans: (c)

60. I. 3x2 + 5x + 2 = 0 II. 3y2 + 18y + 24 = 0

Ans: (a)

61. A seller mark the price 50% above the cost price and given 10% discount on an item. While selling, the cheats customer by giving 20% less in weight. Find his overall profit percent (approximate)?

(a)  26%

(b)  35%

(c)  68%

(d)  72%

(e)  76%

Ans: (b)

62. There are 81 L pure milk in container. One-third of milk is replaced by water in the container. Again one-third of mixture is extracted and equal amount of water is added. What is the ratio of milk to water in the new mixture?

(a)  1 : 2

(b)  1 : 1

(c)  2 : 1

(d)  4 : 5

(e)  None of these

Ans: (d)

63. A is 2 years older than B while B is 3 year younger than C. The ratio of age of A, 6 years hence and B, 2 years ago is 5 : 3. What was age of C, 6 years ago?

(a)  12 yr

(b)  19 yr

(c)  15 yr

(d)  14 yr

(e)  21 yr

Ans: (c)

64. A, B and C started a business with their investments in the ratio 1 : 2 : 4. After 6 month A invested the half amount more as before and B invested same the amount as before while C withdrew 1/4th of his investment. Find the ratio of their profits at the end of the year.

(a)  5 : 12 : 13

(b)  5 : 11 : 14

(c)  5 : 12 : 14

(d)  5 : 12 : 10

(e)  None of these

Ans: (e)

65. The average marks in Science subject of a class of 20 students is 68. If the marks of two students were misread as 48 and 65 of the actual marks 72 and 61 respectively, then what would be the correct average?

(a)  68.5

(b)  69

(c)  69.5

(d)  70

(e)  66

Ans: (b)

Directions (Q. Nos. 66-70) What should come in the place of question mark in the given questions?

66. 40% of 265 + 35% of 180 = 50% of ? + ?% of 80

(a)  80

(b)  95.5

(c)  130

(d)  125.5

(e)  115

Ans: (c)

67. 

(a)  31.4

(b)  28.6

(c)  27.6

(d)  24.2

(e)  22.4

Ans: (a)

68. 

(a)  576

(b)  676

(c)  784

(d)  1024

(e)  1156

Ans: (b)

69. (682% of 682) ÷ 856 = ?

(a)  4.50

(b)  10.65

(c)  2.55

(d)  8.75

(e)  6.25

Ans: (e)

70. 5% of 850 + 24.8% of 650 = ?

(a)  295

(b)  330

(c)  270

(d)  375

(e)  220

Ans: (a)

Part III English Language

Directions (Q. Nos. 71-77) Read the following passage carefully and answer the questions given below it.

Governments looking for easy popularity have frequently been tempted into announcing give-a-ways all sorts; free electricity, virtually free water, subsidised food, cloth at half price, and so on. The subsidy culture has gone to extremes. The richest farmers in the country get subsidised fertilisers, University education, typically accessed by the wealthier sections, is charged at a fraction of cost. Postal services are subsidised, and so are railway services. Bus fares cannot be raised to economical levels because there will be violent protest, so bus travel is subsidised too. In the past, price control on a variety of times, from steel to scent, meant that industrial consumer of these items got them at less than actual cost, while the losses of the public sector companies that produced them were borne by the taxpayer! A study done a few years ago, came to the conclusion that subsidies in the Indian economy total as much as 14.5% of gross domestic product. At today’s level, that would work out to about Rs. 150000 crore.

And who pay the bill? The theory-and the political fiction on the basis of which it is sold to unsuspecting voters-is that subsidies go the poor, and are paid for by the rich. The fact is that most subsidies go the ‘rich’ (defined in the Indian context as those who are above the poverty line), and much of the table goes indirectly to the poor. Because the hefty subsidy bill results in fiscal deficits, which in turn push up rates of inflation-which, as everyone knows, hits the poor the hardest of all. That is why taxmen call inflation the most regressive form of taxation.

The entire subsidy system is built on the thesis that people cannot help themselves, therefore governments must do so. That people cannot afford to pay for variety of goods and services, and therefore the government must step in. This thesis has been applied not just in the poor countries, but in the rich ones as well; hence the birth of the welfare State in the West, and an almost Utopian social security system; free medical care, food aid, old age security, et.al. But with the passage of time, most of the wealthy nations have discovered that their economies cannot sustain this social safety net, which in fact reduces the desire among people to pay their own way, and takes away some of the incentive to work, in short, the bill was  unaffordable, and their societies were simply not willing to pay. To the regret of many, but because of the laws of economies are harsh, most Western societies have been busy pruning the welfare bill.

In India, the lessons of this experience over several decades, and in many countries-do not seem to have been learnt or they are simply ignored in the pursuit of immediate votes. People who are promised cheap food or clothing do not in most cases look beyond the gift horses-to the question of who picks up the tab. The uproar over higher petrol, diesel and cooking gas prices ignored this basic question; if the user of cooking gas does not want to pay for its cost, who should pay? Diesel in the country is subsidies, and if the user of cooking gas does not want to pay for its full cost, who does he or she think should pay the balance of the cost? It is a simple question, nevertheless if remains unasked.

The Deva Gowda government has shown some courage in biting the bullet when it comes to the price of petroleum products. But it has been bitten by much bigger subsidy bug. It wants to offer food at half its cost to everyone below the poverty line, supposedly estimated at some 380 million people.

What will be the cost? And of course, who will pick up the tab? The Andhra Pradesh government has been bankrupted by selling rice as Rs. 2 per kg. Should the Central government be bankrupted too, before facing up to the question of what is affordable and what is not? Already, India is perennially short of power because the subsidy on electricity has bankrupted most electricity boards, and made private investment wary unless it gets all manner of State guarantees. Delhi’s subsidized bus fares have bankrupted the Delhi Transport Corporation, whose buses have slowly disappeared from the capital’s streets. It is easy to be soft and sentimental, by looking at programmes that will be popular. After all, who does’ not like a free lunch? But the evidence is surely mounting that the lunch isn’t free at all. Somebody is paying the bill. And if you want to know who, take at the country’s poor economic performance over the years.

71. Which of the following should not be subsidised over the years?

(a)  University education

(b)  Postal services

(c)  Steel

(d)  Other than those given as options

(e)  All of the above options

Ans: (d)

72. The statement that ‘subsidies are paid by the rich and go to the poor’ is

(a)  fiction

(b)  fact

(c)  fact, according to the author

(d)  fiction, according to the author

(e)  Other than those given as options

Ans: (d)

73. Why do you think that the author calls the western social security system Utopian?

(a)  The countries belief in the efficacy of the system was bound to turn out to be false.

(b)  The system followed by these countries is the best available in the present context.

(c)  Everything under this system was supposed to be free, but people were charging money for them.

(d)  The theory of system followed by these countries was devised by Dr. Utopia.

(e)  Al the options are responsible.

Ans: (c)

74. It can be inferred from the passage that the author

(a)  believes that people can help themselves and do not need the government.

(b)  believes that the theory of helping with subsidy is very destructive.

(c)  believes in democracy and free speech.

(d)  is not a successful politician.

(e)  believes that subsidies are the best way to help poor.

Ans: (b)

75. Which of the following is not a victim of extreme subsidies?

(a)  The poor

(b)  The Delhi-Transport Corporation

(c)  The Andhra Pradesh government

(d)  Other than those given as options

(e)  The rich

Ans: (e)

76. Which of the following is not true in the context of the passage?

(a)  Where subsidies are concerned, the poor ultimately pay the tab.

(b)  Inflation is caused by too much subsidies.

(c)  Experts call subsides the most regressive form of taxation.

(d)  Fiscal deficits are caused due to heavy subsidy bills.

(e)  None of the following is true in the context of the passage.

Ans: (a)

77. A suitable title to the passage would be :

(a)  There’s No Such Thing as a Free Lunch

(b)  The Economic Overview

(c)  Deva Gowda’s Government and its Follies

(d)  It takes Two to Tango

(e)  The Rich and The Poor: Extreme Partiality

Ans: (a)

Directions (Q. Nos. 78-82) Rearrange the following six sentences A, B, C, D, E and F in the proper sequence to form a meaningful paragraph and then answer the questions given below.

A. It is the only country in the world that is carbon negative, which means, it produces more oxygen than it consumes.

B. Bhutan, sandwiched between the two most populous nations on Earth, suffers for their sins.

C. So far, so good. But then, two things happened.

D. Carbon sinks, 70% forest cover, powered almost entirely by mountain streams-Bhutan is a poster child for green living.

E. Glaciers are beginning to melt, flash floods and heavy rains – and even droughts – are common, and temperatures are climbing.

F. One, India and China got richer.

78. Which of the following should be the FIRST sentence of the given paragraph?

(a)  E

(b)  D

(c)  C

(d)  B

(e)  A

Ans: (b)

79. Which of the following should be the THIRD sentence of the given paragraph?

(a)  A

(b)  B

(c)  C

(d)  D

(e)  E

Ans: (c)

80. Which of the following should be the LAST sentence of the given paragraph?

(a)  A

(b)  D

(c)  C

(d)  B

(e)  E

Ans: (e)

81. Which of the following should be the FOURTH sentence of the given paragraph?

(a)  F

(b)  C

(c)  B

(d)  E

(e)  D

Ans: (a)

82. Which of the following should be the SECOND sentence of the given paragraph?

(a)  B

(b)  D

(c)  A

(d)  C

(e)  E

Ans: (c)

Directions (Q. Nos. 83-90) In the following passage, you have a brief passage. In the following passage, some of the words have been left out. First read the passage over and try to understand what it is about. Then fill in the blanks with the help of the alternatives given.

Big ideas come from tackling (83) problems. When one is confronted with an overwhelming task, it’s pieces. Business jargon is full of phrases about that, like “pilot projects” and “low-hanging fruit.” They have their place, but in the repertory of management (84), they should share their place with bold approaches to big challenges. Much of today’s most valuable management knowledge came from wrestling with such issues. The most complicated workplace in the middle of the last century was the automobile assembly plant. Drawn to its complexity where Peter F. Drucker, W. Edwards Deming, and Taiichi Ohno, among others, the work they and their disciples did, applied in industry after industry, is the basis of the best that we know about operations, managing people, innovation, organizational design, and much more. The most complex workplaces are tertiary care hospitals. These vast (85) employ tens of thousands of people who, under one roof, do everything from neurosurgery to laundry, Each patient – that is to say, each “job” – calls on a different set of people with a different constellation of (86); eve when the tow patients have the same diagnosis, success may be (87) differently. This is complexity of an order of magnitude greater than automobile assembly, and anyone, who (88) hospitalised knows that management has thus far been unequal to the scope of task. The workers, managers, consultants, and scholars (89) crack this nut will reshape industries and institutions just as (90) as Drucker, Deming, and Ohno Did.

83. (a)  small

(b)  big

(c)  irrelevant

(d)  buildings

(e)  minor

Ans: (b)

84. (a)  weakness

(b)  strength

(c)  power

(d)  practice

(e)  symptom

Ans: (b)

85. (a)  houses

(b)  institute

(c)  demagogue

(d)  forts

(e)  enterprises

Ans: (e)

86. (a)  barbarity

(b)  talent

(c)  skills

(d)  unskilled

(e)  barbaric

Ans: (c)

87. (a)  managed

(b)  officious

(c)  delivered

(d)  measured

(e)  postponed

Ans: (d)

88. (a)  are been

(b)  have being

(c)  have been

(d)  has been

(e)  is be

Ans: (d)

89. (a)  who

(b)  whom

(c)  whose

(d)  which

(e)  whomsoever

Ans: (a)

90. (a)  profoundly

(b)  gradually

(c)  superficially

(d)  speciously

(e)  earnest

Ans: (*)

Directions (Q. Nos. 91-100) Identify the error in the sentences given below, if there is no error, choose option (e).

91. (a)  The need to set-up

(b)  a good library in the locality

(c)  has been in the minds of people

(d)  for some time now

(e)  No error

Ans: (a)

92. (a)  Most people would have

(b)  attended the union meeting

(c)  if they had

(d)  had longer notice of it

(e)  No error

Ans: (d)

93. (a)  He took to

(b)  reading times

(c)  for better knowledge

(d)  of the facts

(e)  No error

Ans: (c)

94. (a)  When children have difficulty understanding

(b)  a certain mathematical process, it is often because

(c)  their teachers do not understand it conceptually

(d)  themselves and do not present it in a way that children can understand

(e)  No error

Ans: (e)

95. (a)  Studies show that the lives of millions of mothers

(b)  and their children could be saved if countries would

(c)  invest in programmes that ensures a healthy pregnancy

(d)  and safe childbirth

(e)  No error

Ans: (b)

96. (a)  Film viewers claim that

(b)  the number of scenes depicting alcohol consumption

(c)  have increased dramatically over

(d)  the last decade

(e)  No error

Ans: (c)

97. (a)  Forty percent of the people alive today have

(b)  never made a phone call, but

(c)  thirty percent still have no electricity connections

(d)  to their homes

(e)  No error

Ans: (b)

98. (a)  Workers with less

(b)  personal problems are

(c)  likely to be

(d)  more productive in their work

(e)  No error

Ans: (a)

99. (a)  Everyone who visits Singapore

(b)  is impressed by its cleanliness

(c)  which is mainly a result of rigorous implementation

(d)  of their strict laws

(e)  No error

Ans: (d)

100. (a)  The bridal dress was

(b)  most unique: the prince

(c)  designed it and his

(d)  mother provided the lace fabric

(e)  No error

Ans: (b)

IBPS CWE-VI Clerical Cadre Preliminary Online Examination Held on December 4, 2016 Question Paper with Answer Key

IBPS CWE-VI Clerical Cadre Preliminary Online Exam 2016
IBPS CWE-VI Clerical Cadre Preliminary Online Examination Held on December 4, 2016 Question Paper with Answer Key

IBPS CWE-VI Clerical Cadre Preliminary Online Examination

Held on December 4, 2016

Part I English Language

Directions (Q. Nos. 1-10) Read these sentences to find out whether there is any grammatical error in them. The error, if any, will be in one part of the sentence. Mark the part with the error as your answer. If there is no error, mark ‘No error’ as your answer. Ignore the errors of punctuation, if any.

1. This company has been/served a legal notice for/using banned substances in/ the manufacture of its products.

(a)   This company has been

(b)   Served a legal notice for

(c)   using banned substances in

(d)   the manufacture of its products

(e)   No error

Ans: (d)

2. All coastal villages have/been evacuated to proven/ any loss of life due/ to the incoming cyclone.

(a)   All coastal villages have

(b)   been evacuated to prevent

(c)   any loss of life due

(d)   to the incoming cyclone

(e)   No error

Ans: (d)

3. The reporter was arrested and/ his camera seized by the police,/ after he entered the prohibited/ zone and started taken photos.

(a)   The reporter was arrested and

(b)   his camera seized by the police

(c)   after he entered the prohibited

(d)   zone and started taken photos

(e)   No error

Ans: (d)

4. After been caught drinking/ in their hostel room, the/ students were suspended from/ the college for a week.

(a)   After been caught drinking

(b)   in their hostel room, the

(c)   students were suspended from

(d)   the college for a week

(e)   No error

Ans: (a)

5. Once the party ended, we/ realized that it was/ midnight and that there was/ no way we could return home.

(a)   Once the party ended, we

(b)   realized that it was

(c)   midnight and that there was

(d)   no way we could return home

(e)   No error

Ans: (a)

6. There has been a/ shortage in the supply of/ water in the city/ due for lack of rainfall.

(a)   There has been a

(b)   shortage in the supply of

(c)   water in the city

(d)   due for a lack of rainfall

(e)   No error

Ans: (d)

7. This move has resulted/ in bringing commodity/ prices down and uplift/ the country’s economy.

(a)   This move has resulted

(b)   in bringing commodity

(c)   prices down and uplift

(d)   the country’s economy

(e)   No error

Ans: (b)

8. Raj could not take the/call as he have put his cell/ phone on ‘silent’ mode and thus/ could not hear it ringing.

(a)   Raj could not take the

(b)   calls as he have put his cell

(c)   phone an ‘silent’ mode and thus

(d)   could not hear it ringing

(e)   No error

Ans: (b)

9. He has a passion/ for climbing mountains and/ is contemplates to scale/ Everest next year.

(a)   He has a passion

(b)   for climbing mountains and        

(c)   is contemplates to scale

(d)   Everest next year

(e)   No error

Ans: (c)

10. In their statement, the/ official have blamed/ their supervisors for/having mismanaged the situation.

(a)   In their statement, the official have blamed

(b)   official have blamed

(c)   their supervisors for

(d)   having mismanaged the situation

(e)   No error

Ans: (d)

Directions (Q. Nos. 11-20) Read the following story carefully and answer the questions given below. Certain word/phrases have been given in bold to help you locate them while answering some of the questions.

  Duanji was a magistrate who lived a hundred years ago. He was famed or the fairness of his judgements. One day, while the magistrate was walking through the market, he saw a crowd outside a poultry shop. On inquiring what the matter was, he learnt that a peasant had accidentally dropped a heavy sack on a chicken, crushing it to death.

The chicken was small, worthy only about five coins, but the owner of the shop had caught the peasant by his throat and was demanding a hundred. His argument was that the chicken would have grown into a plump bird in another two y ears and then it would have fetched him the amount he was asking for.

Somebody recognized the magistrate and everybody made way for him. “Judge our case, MY lord”, said the owner of the chicken, letting go of the peasant and bowing obsequiously to the magistrate. “This man, through his carelessness has caused the death of a chicken that would have fetched me a hundred coins in another two years!”

Pest had made the peasant’s speech incoherent. Nobody could understand what he was saying. “The price put on the chicken is hundred coins”, said the magistrate, to the peasant. “I advise you to pay.” There was a gasp from the crowd.

Everybody had expected the magistrate to rule in the peasant’s favour as he was penniless. The owner of the chicken was overjoyed. “They said you were fair in your judgements. My Lord,” He said, nibbling his hands in glee, “now I can say there is no one fairer than you”.

“The law is always fair”, smiled the magistrate. “Tell me, how much Grain does a chicken eat in a year.” “About half a sack”, said the poultry shop owner. “So in two years the chicken who died would have whole sack of grain,” said Duanji. “Please give the sack of grain you have saved to this man”.

The owner turned pale. A sack of grain would cost more than hundred cons. Cowed by the jeers of the crowd, he declared he would not take any money from the peasant, and retreated into the safety of h is shop.

11. Which of the following is most nearly the same in meaning as the word ‘Overjoyed’ as used in the story?

(a)   Depressed

(b)   Charmed

(c)   Overburdened

(d)   Dreamy

(e)   Thrilled

Ans: (e)

12. Which of the following is mot nearly the opposite in meaning to the word ‘Incoherent’ as used in the story?

(a)   Illegible

(b)   Identified

(c)   Clear

(d)   Confused

(e)   loud

Ans: (c)

13. As mentioned in the story, everyone was disappointed when Duanji informed the peasant that he had to pay a price to the poultry shop owner, because

1. they know the peasant could not afford to p ay the fine

2.they were expecting him to protect the peasant

3. they know it was not a fair decision

(a)   Only 1

(b)   1 and 2

(c)   Only 2

(d)   Only 3

(e)   All of these

Ans: (e)

14. Which of the following is/are not true in the context of the story?

1. The poultry shop owner and the peasant became friends eventually.

2. Duanji subsequently fulfilled the expectations of the crowd by taking the right decision.

3. The peasant decided to give half the money to the poultry shop owner.

(a)   Only 1

(b)   Only 2

(c)   Only 3

(d)   1 and 3

(e)   All the given options are not true.

Ans: (d)

15. Which of the following correctly explains the meaning of the phrase ‘Made way for him’ as used in the story?

(a)   Did not allow him to move ahead

(b)   Gave him his best place to sit

(c)   Succeeded

(d)   Moved carefully

(e)   Move aside

Ans: (e)

16. Which of the following is most nearly the same in meaning as the phrase ‘Turned pale’ as used in the story?

(a)   Was shaken

(b)   Confident

(c)   Annoyed

(d)   Haunted

(e)   Feeling rosy

Ans: (a)

17. Which of the following can be said about the poultry shop owner?

(a)   He was forgiving.

(b)   He was annoying.

(c)   He was charming.

(d)   He was devious.

(e)   He was patient.

Ans: (d)

18. As mentioned in the story, the poultry shop owner decided not to take money from-the peasant because

(a)   he was afraid of the crowd

(b)   he realised he would be at a loss

(c)   he respected the magistrate’s decision

(d)   he decided to forgive the peasant

(e)   he decided to take it later on from his relatives

Ans: (b)

19. Which of the following is most nearly the opposite in meaning as the word ‘Fair’ as used in the story?

(a)   Function

(b)   Indecisive

(c)   Unacceptable

(d)   Expected

(e)   Biased

Ans: (e)

20. Which of the following can be a suitable title for the story?

(a)   The peasant who lost his chicken

(b)   The faithful chicken

(c)   The poor poultry shop owner     

(d)   The final verdict

(e)   The rich magistrate

Ans: (d)

Directions (Q. Nos. 21-30) In the given passage, there blanks, each of which has been numbered. Against each, five words/group of words are suggested, one of which fits the blank appropriately. Find out the appropriate word/group of words in each case.

The old man was sitting under a tree. ‘Sit’ his voice rang out. ‘Close your eyes and listen the story will serve you will.’

There was a handsome prince whose best friend as a washerman’s son. The king’s father believed that everyone was equal whether born in a palace or a hut. (21) his son to school with the other children- if the school was good enough for them. It was good enough for his son. Now the prince grew up to be good and kind and (22) a girl from the kingdom.

But the king had reservations, Ruling the kingdom required a great strength of character. Was the prince (23) this responsibility? But he served his son and agreed to the marriage. The marriage ceremony was a grand affair and the reception (24). In all the merriment and joy there was no unhappiness. Time went on and the prince and his bride were happy.

The one night, the royal party (25) to the palace when it was attacked by thieves. There was a fierce battle in which the prince (26). The prince was beheaded and the leader of the gang too. The princess prayed fervently for help and Goddess of protection defeated the wicked thieves. She was (27) the princess’ tears that she granted her a boon. The princess immediately asked for the life of her husband.

There was a (28) thunder and the prince was restored to life. But then the Goddess realized that in bringing him back to life she had attached his head as that of the thief. It could not be (29). When they returned to the palace, the king, his ministers and all who encountered them from (30). The princess said to them, ‘It is the head which controls the body which determines character. This is your prince.

21. (a)   So he sent

(b)   He refused

(c)   Instead

(d)   Then he put

(e)   Therefore

Ans: (a)

22. (a)   married

(b)   decided to marry

(c)   choose his bride

(d)   prefer to marry

(e)   fixed

Ans: (b)

23. (a)   capable

(b)   handle

(c)   worthy of

(d)   accomplish

(e)   deal with

Ans: (c)

24. (a)   held

(b)   not held

(c)   well

(d)   even grander

(e)   still impressive

Ans: (d)

25. (a)   arriving at

(b)   was returning

(c)   leaving

(d)   visited

(e)   lack

Ans: (b)

26. (a)   fall down

(b)   injured

(c)   was slain

(d)   killed

(e)   murdered

Ans: (c)

27. (a)   so moved by

(b)   heard by

(c)   troubled

(d)   so unhappy

(e)   crying

Ans: (a)

28. (a)   sounded

(b)   noisy

(c)   sudden

(d)   loud clap of

(e)   lightning

Ans: (d)

29. (a)   ruined

(b)   reverse

(c)   incomplete

(d)   change

(e)   undone

Ans: (e)

30. (a)   upset

(b)   taken shock

(c)   were thrilled

(d)   were shocked

(e)   alarmed

Ans: (d)

Part II Reasoning Ability

Directions (Q. Nos. 31-35) Study the following arrangement of letters numbers and symbols carefully to answer the given questions.

N A 3 K 2 W 8 G 4 @ Z * D % 9 H 7 $ 5 U Ω I L & S R = Y 6 © E

31. As per the given arrangement , four of the following five are alike in a certain way and hence form a group. Which of the following does not belong to the group?

(a)   UHQ

(b)   9D7

(c)   @G*

(d)   &1R

(e)   KAW

Ans: (a)

32. Which of the following will be the seventh to the right of the thirteenth element from the right end of the given arrangement?

(a)   R

(b)   V

(c)   L

(d)   6

(e)   &

Ans: (a)

33. How many letters are there between the seventh element from the left end and the fourteenth element from the right end of the given arrangement?

(a)   Four

(b)   Two

(c)   Seven

(d)   Six

(e)   Five

Ans: (a)

34. What will be the sum of all the numbers between the tenth element from the left end and the eighth element from the right end of the given arrangement?

(a)   22

(b)   15

(c)   17

(d)   20

(e)   19

Ans: (a)

35. If all the letters from the given arrangement are deleted, then which of the following will represent the third element to the left of 4 and fifth element to the right of Ω respectively?

(a)   2, #

(b)   B, #

(c)   3, ©

(d)   3, 6

(e)   2, ©

Ans: (c)

 

36. If in the word FRACTION, first all the consonants are arranged in alphabetical order from left to right and then all the vowels are arranged in alphabetical order from left to right, which of the following is the fifth letter from the right?

(a)   F

(b)   N

(c)   T

(d)   R

(e)   A

Ans: (d)

37. If all the digits of the number ‘6379452’, are arranged in ascending order from left to right, what will be the sum of the fourth digit from the left and second digit from the right after rearrangement?

(a)   10

(b)   12

(c)   15

(d)   13

(e)   11

Ans: (b)

38. The position of the first and the fifth letters of the word HERITAGE are interchanged, similarly, the position of second and sixth letters and third and seventh letters and fourth and eight letters are interchanged. In the new arrangement thus formed, how many letters are there between the letters which is fourth from the left and first from the right in the English alphabetical series.

(a)   Two

(b)   None

(c)   More than three

(d)   Three

(e)   One

Ans: (d)

39. Point D is 7 m to the West of Point R. Anand starts walking form Point R towards East. He walks for 3 m and takes a right turn. He further walks for 4 m takes another right turn. He walks for 5 m and turns to his right. He further walks for 4 m and stops at Point T. In which direction is Point T with respect to Point D?

(a)   South

(b)   West

(c)   East

(d)   Cannot be determined

(e)   North

Ans: (c)

Directions (Q. Nos. 40-44) In these questions, relationship between different elements is shown in the statements. The statements are followed b y two conclusions. Study the conclusions based on the given statement (s) and select the appropriate answer.

Given answer

a. if either conclusion I or ii is true

b. if both conclusions I and II are true

c. if neither conclusion I nor II is true

d. if only conclusion II is true

e. if only conclusion I is true

40. Statements : F < W = J ≥ K; W ≤ D

Conclusions: I. D > K      II. K = D

Ans: (a)

41. Statements: V ≤ E = U ≤ X > A

Conclusions: I. X ≥V      II. E > A

Ans: (e)

42. Statements: O = M ≥ G; M ≤ P; S > M

Conclusions: I. S > G      II. O ≤ P

Ans: (b)

43. Statements: X ≥ W < D = U ≥ R

Conclusions:  I. X < U     II. R > W

Ans: (c)

44. Statements: N > H = R ≥ B; R ≤C

Conclusions: I. C < H      II. H > B

Ans: (c)

Directions (Q. Nos. 45-49) These questions are based on five words given below

            ROD   ITS  MUG    RAY    SEW

45. If the third alphabet in each of the words is changed to next alphabet in the English alphabetical order, in how many words thus formed will an alphabet appear twice?

(a)   One

(b)   Two

(c)   Four

(d)   Three

(e)   None

Ans: (a)

46. If the third letter in each of the words becomes the first letter, the first becomes the second and the second becomes the third letter, which of the following will form meaningful English words?

(a)   Both MUG and RAY

(b)   None of the given options will form a meaningful English word.

(c)   Both ROD and SEW

(d)   Only RAY

(e)   Only ITS

Ans: (e)

47. Which of the following represent the letters immediately following, (in the English alphabetical order) the third letter of the word which is third from the right and the second letter of the word which is second from the left of the given words? (the counting is done from left to right)

(a)   H, U

(b)   F, R

(c)   R, H

(d)   K, N

(e)   Y, T

Ans: (a)

48. If the given words are arranged in the order as they would appear in the dictionary from left to right, the position of which of the following will not change when compared to the given positions?

(a)   Both ITS and RAY

(b)   The positions of all the given words would change.

(c)   Only MUG

(d)   Only SEW

(e)   Only ROD

Ans: (d)

49. If in each of the given words, each of the consonant is changed to the previous letter and each vowel is changed to next letter in the English alphabetical series, how many words thus formed will have at least one vowel?

(a)   More than three

(b)   None

(c)   Three

(d)   Two

(e)   One

Ans: (b)

Directions (Q. Nos. 50-54) Study the given information carefully and answer the given questions.

Eight people are sitting in two parallel rows, containing four people in such a way that there is an equal distance between adjacent persons. In row-1, M, N, O and P are seated and all of them are facing North and in row-2, W, X, Y and Z are seated and all are facing South (but not necessarily in the same order.)

Thus, in the given arrangement each person seated in a row faces another person of the other row. O sits second to the right of one who sits at an extreme ends of the line. O faces an immediate neighbour of W, X sits to the immediate right of W. The one who faces X sits at one of the positions to the left of P. P does not face W. M sits to the immediate left of the one who faces Z.

50. Which of the following represents the immediate neighbours of the one who faces Z?

(a)   Only O

(b)   N and M

(c)   Only N

(d)   Only M

(e)   M and P

Ans: (e)

51. Which of the following represents the people sitting to the immediate right of O and immediate right of Z respectively?

(a)   P, W

(b)   N, W

(c)   N, Y

(d)   P, X

(e)   M, W

Ans: (a)

52. Who sits to the immediate right of the one who faces an immediate enighbour of W?

(a)   Only P

(b)   P and N

(c)   Only M

(d)   Only O

(e)   M and P

Ans: (e)

53. Which of the following represents the correct position of N?

(a)   Immediate left of the one who faces V

(b)   Second to the right of O

(c)   Immediate left of P

(d)   Second to the left of the one who faces Z

(e)   None of the given options

Ans: (d)

54. Who amongst the following faces P?

(a)   Either Z or W

(b)   X

(c)   Z

(d)   Y

(e)   W

Ans: (d)

Directions (Q. Nos. 55-59) Read the following passage and sovle the question based on it.

A, B, C, D, E, F and G are seven members of a family standing in a row (not necessarily in the same order) facing a particular direction. C and B have as many members between them as G and C have between them. D, who is 3rd from the extreme left, is 3rd to the left of E. A and D are neighbours and F and C are neighbours. B is the neighbour of E.

55. Which of the following statements is false?

(a)   A is 3rd to the left of F.

(b)   D is 3 rd to the left of E.

(c)   F is 3rd to the right of A.

(d)   B is 3rd to the left of C.

(e)   C is 3rd to the left of F.

Ans: (d)

56. Which of the following statement is true?

(a)   C and E are neighbours

(b)   E is to the immediate left of F.

(c)   C is to the immediate left of D.

(d)   A is the immediate left of D.

(e)   None of the above

Ans: (d)

57. Who is at the extreme right?

(a)   G

(b)   B

(c)   E

(d)   Data Inadequate

(e)   F

Ans: (b)

58. Which of the following gives two pairs of neighbours?

(a)   A, C and D, C

(b)   A, B and E, G

(c)   D, C and E, F

(d)   C, F and C, E

(e)   None of these

Ans: (c)

59. Who sits immediate right of F?

(a)   E

(b)   C

(c)   G

(d)   A

(e)   B

Ans: (a)

Directions (Q. Nos. 60-64)  Study the following information carefully and answer the given questions.

Eight people- J, K, L, M, N, O, P and Q are sitting around a circular table with equal distance between each other, but not necessarily in the same order. Q sits to the immediate left of N. Only four persons sit between Q and L. P sits second to the right of M. M is not an immediate neighbour of L. Only three people sit between O and K. K is neither an immediate neighbour of M nor Q.

60. Which of the following statements is not true as per the given arrangement?

(a)   P sits third to the left of K.

(b)   All the given statements are true.

(c)   Both M and L are immediate neighbours of J.

(d)   Only three people sit between N and P

(e)   K sits to the immediate left of L.

Ans: (d)

61. As per the given arrangement, J : L in the same way as K : N. Following the same pattern, Q:_?

(a)   J

(b)   M

(c)   K

(d)   P

(e)   O

Ans: (d)

62. What is Q’s position with respect to J?

(a)   Second to the left

(b)   Fourth to the left

(c)   Second to the right

(d)   Immediate right

(e)   Third to the right

Ans: (b)

63. Who sits exactly between the one who is to the immediate right of P and K when counted from the left of K?

(a)   N

(b)   O

(c)   J

(d)   K

(e)   Q

Ans: (a)

64. In the given arrangement, if O interchanges its position with N and P interchanges its position with K, then how many persons will sit between P and N, when counted from the right of P?

(a)   Two

(b)   One

(c)   More than three

(d)   Three

(e)   None

Ans: (d)

65. The following series is based on the English alphabets and numbers, which of the following will come in place of the question mark in the given series?

    XW2TS6PO10LK14

(a)   HG18

(b)   JI18

(c)   HG16

(d)   IH18

(e)   JG12

Ans: (a)

Part III Quantitative Aptitude

66. The respective ratio between two positive numbers (X and Y) is 3 : 5. When 2 is added to both the numbers, the ratio between X and Y becomes 5 : 8. What is the difference between both the numbers?

(a)   2

(b)   12

(c)   9

(d)   6

(e)   3

Ans: (b)

67. 

(b)   8

(c)   2

(d)   11

(e)   9

Ans: (a)

68. ? % of 750 = 75% of 600

(a)   60

(b)   75

(c)   80

(d)   20

(e)   40

Ans: (a)

69. 3720 ÷ 20 = 508 + ?

(a)   454

(b)   422

(c)   −322

(d)   444

(e)   452

Ans: (c)

70. 60% of (? + 173.5 + 157.5) = 21 × 12

(a)   83

(b)   109

(c)   89

(d)   79

(e)   99

Ans: (c)

71. (154 × 2.5 ÷ 0.5) ÷ ? = 192.5

(a)   6

(b)   8

(c)   2

(d)   4

(e)   18

Ans: (d)

72. 

(a)   429

(b)   411

(c)   423

(d)   413

(e)   417

Ans: (e)

73. 60% of 725 = 174 × ?

(a)   0.2

(b)   0.9

(c)   2.5

(d)   0.8

(e)   0.3

Ans: (c)

74. 

(a)   372

(b)   324

(c)   354

(d)   328

(e)   347

Ans: (a)

75. 

(a)   2

(b)   2.4

(c)   0.8

(d)   1.6

(e)   4.8

Ans: (b)

76. ? – (0.6)2 = (0.7)2 ÷35

(a)   1.54

(b)   1.32

(c)   1.92

(d)   1.62

(e)   1.76

Ans: (e)

77. 68% of 320 + 30 ÷8 = ?

(a)   254.4

(b)   258.2

(c)   260.2

(d)   252.7

(e)   255.1

Ans: (e)

78. 

(a)   1158

(b)   784

(c)   578

(d)   1024

(e)   484

Ans: (d)

79. 

(a)   17

(b)   21

(c)   23

(d)   19

(e)   11

Ans: (d)

80. 

(a)   4

(b)   5

(c)   8

(d)   2

(e)   3

Ans: (b)

81. ?2 × 8 – 250 = 395

(a)   6

(b)   14

(c)   12

(d)   3

(e)   9

Ans: (e)

82. Jim invested Rs. 1700 and Rs. 2300 in schemes A and B respectively for 3 years. If schemes A and B offer simple interest @ 6 percent per annum and @ 9 percent per annum respectively, what is the total interest earned by Jim from both the schemes together after 3 years?

(a)   Rs. 927

(b)   Rs. 933

(c)   Rs. 949

(d)   Rs. 935

(e)   Rs. 945

Ans: (a)

Directions (Q. Nos. 83-87) What will come in place of question marks in the given number series?

83. 5  9  25  89  ?  1369

(a)   343

(b)   355

(c)   349

(d)   341

(e)   345

Ans: (e)

84. 6  17  50  149  ?   1337

(a)   454

(b)   446

(c)   442

(d)   452

(e)   432

Ans: (b)

85. 61  62  54  81  17   ?

(a)   142

(b)   145

(c)   138

(d)   144

(e)   140

Ans: (a)

86. 6  11  18  29  46  ?

(a)   60

(b)   75

(c)   69

(d)   71

(e)   79

Ans: (d)

87. 26  13  13  5  ?  97.5

(a)   39

(b)   45

(c)   48

(d)   41

(e)   35

Ans: (a)

88. In 2003, the total population of a village was 4800, out of which 40% were females. In 2004, the total population increased by 10% as compared to the previous year. If the number of females remained the same in 2004, what was the percentage increase in the number of males in 2004 as compared to 2003?

(a)    

(b)   20%

(c)   15%

(d)    

(e)   

Ans: (d)

89. A boat can travel 114.4 km downstream in 32 minutes. If the speed of the current is 3 km per hour, how much time the boat will take to travel 84 km upstream?

(a)   12 h

(b)   8 h

(c)   4 h

(d)   3 h 30 min

(e)   7 h

Ans: (c)

90. A and B both start a business. Amount invested by B is one-third of that invested by A. Three months from the start of the business, A withdrew one-third of his investment and B tripled his investments. If from the total annual profit earned, A receives Rs. 1800 as his share from the profit, what was the total annual profit earned?

(a)   Rs. 2850

(b)   Rs. 4750

(c)   Rs. 3800

(d)   Rs. 3600

(e)   Rs. 5400

Ans: (c)

91. A tank (of capacity 160 L) has one inlet, A and one outlet B. Inlet A, alone can fill the empty tank in 6 hours and outlet B alone can empty the full tank in 24 hours. In how many hours will inlet A fill 16 L water in the tank when outlet B is also open?

(a)   1

(b)   4/5

(c)   11/5

(d)    

(e)   1/5

Ans: (b)

92. The respective ratio between length and breadth of a rectangle is 8 : 5 and its perimeter is 208 cm. If the side of the square is 40% less than the breadth of the rectangle, what is the perimeter of the square?

(a)   54 cm

(b)   68 cm

(c)   88 cm

(d)   96 cm

(e)   92 cm

Ans: (d)

93. The respective ratio of cost price of articles A and B is 7 : 9. Article A was sold at a profit of 40% and article B was sold at a profit of 10%. If the total profit earned after selling both the articles (A and B) is Rs. 148, what is the difference between cost price of articles A and B?

(a)   Rs. 120

(b)   Rs. 160

(c)   Rs. 200

(d)   Rs. 80

(e)   Rs. 40

Ans: (d)

94. Two years hence, the respective ratio between A’s age at that time and B’s age at that time will be 6 : 5, A’s age thirteen years ago was half of B’s present age. What is A’s present age?

(a)   16 yr

(b)   40 yr

(c)   28 yr

(d)   22 yr

(e)   34 yr

Ans: (d)

95. Ranu’s monthly salary is four-fifth of Ali’s monthly salary. Ranu and Ali save one-fourth and two-fifth amount from their respective monthly salary. If the difference between the amount save by Ranu and that saved by Ali is Rs. 7000, what is Ranu’s monthly salary?

(a)   Rs. 30000

(b)   Rs. 35600

(c)   Rs. 35000

(d)   Rs. 28000

(e)   Rs. 21000

Ans: (d)

Directions (Q. Nos. 96-100) Study the table and answer the given questions.

96. What is the average number of muffins sold by bakery T in March, April and July?

(a)   179

(b)   177

(c)   176

(d)   172

(e)   174

Ans: (b)

97. What is the respective ratio between the total number of muffins sold by bakeries Q and S together in May and the total number of muffins sold by bakeries R and T together in the same month?

(a)   21 : 23

(b)   45 : 88

(c)   19 : 23

(d)   20 : 23

(e)   22 : 29

Ans: (b)

98. If the number of muffins sold by bakery T in August was 70% more than that sold by the same bakery in June, what was the number of muffins sold by the bakery T in August?

(a)   255

(b)   221

(c)   323

(d)   238

(e)   306

Ans: (e)

99. The total number of muffins sold by bakeries Q and S together in June is, what percent more than the number of muffins sold by bakery P in March?

(a)   70%

(b)   74%

(c)   72%

(d)   73%

(e)   75%

Ans: (c)

100. What is the difference between the total number of muffins sold by bakeries P and Q together in April and the total number of muffins sold by bakeries R and S together in July?

(a)   175

(b)   152

(c)   165

(d)   143

(e)   155

Ans: (a)

 

IBPS Specialist Officers (IT) Examination 2015 Held on 14-2-2016 Morning Shift Quantitative Aptitude Question Paper with Answer Key

IBPS Specialist Officer (IT) Examination
IBPS Specialist Officers (IT) Examination 2015 Held on 14-2-2016 Morning Shift Quantitative Aptitude Question Paper with Answer Key

IBPS Specialist Officers (IT) Exam., 2015 (Morning Shift)

Held on 14-2-2016

Quantitative Aptitude

1. 15 men and 16 women together can complete a piece of work in 6 days. If 12 women can complete the same project in 32 days, in how many days will 10 men complete the same project ?

(A)  12

(B)  20

(C)  16

(D)  8

(E)  14

Ans: (A)

2. The interest earned when Rs. ‘P’ is invested for four years in a scheme offering 9% p.a. simple interest is more than the interest earned when the same sum (Rs. P) is invested for two Years in another scheme offering 12% p. a. simple interest, by Rs. 360. What is the value of P ?

(A)  2000

(B)  3500

(C)  2500

(D)  4000

(E)  3000

Ans: (E)

Directions – (Q. 3 – 8) Refer to the pie charge and answer the given questions.

Percentage of employees in different departments of branch ‘LMN’ in the  year 2014

3. In 2014, there were 30% post graduate employees in department B. In 2015, 20 employees of the same department were shifted to branch ‘PQR’. If in 2015, the percentage of post graduate employees in department B were the same, how many post graduate employees were shifted to branch ‘PQR’ ?

(A)  4

(B)  10

(C)  7

(D)  6

(E)  8

Ans: (D)

4. The number of employees in Department D is what percent less than the total number of employees in Department A and F together ?

(A)  50

(B)  80

(C)  75

(D)  25

(E)  85

Ans: (C)

5. In department C, the respective ratio between the number of female employees and male employees was 5 : 6. There were equal number of unmarried males and unmarried females in department C. If the respective ratio between married females and married males was 4 : 5, what is the number of unmarried females ?

(A)  20

(B)  8  

(C)  10

(D)  12

(E)  16

Ans: (D)

6. In 2014, the number of female employees in department E has 5/11 of the total number of employees in the same department. If the number of female employees in department D was 16 less than that in Department E, what is the number of male employees in Department D ?

(A)  32

(B)  31

(C)  28

(D)  20

(E)  24

Ans: (D)

7. What is the average number of employees in Departments C, E and F ?

(A)  89

(B)  87

(C)  97

(D)  109

(E)  99

Ans: (E)

8. What is the central angle corresponding to the number of employees in Department F ? (in Degrees)

(A)  55.9

(B)  56.5

(C)  50.4

(D)  52.8

(E)  51.6

Ans: (C)

9. Four years ago, the respective ratio between the age of Ram and that of Sonu, was 4 : 9. Tina is ten y ears older to Ram. Tina is ten years younger to Sonu. What is Tina’s present age ?

(A)  40 years

(B)  36 years

(C)  30 years

(D)  20 years

(E)  42 years

Ans: (C)

10. A jar has 60 It milk. From the jar, 12 lts of m ilk was taken out and replaced by an equal amount of water. If 12 lts of the newly formed mixture is taken out of the jar, what is the final quantity of milk left in the jar ?

(A)  38.4 lts

(B)  40 lts

(C)  36 lts

(D)  28.6 lts

(E)  36.5 lts

Ans: (A)

Directions – (Q. 11-16) Refer to the graph and answer the given question.

Data regarding number of tourists in two cities – A and B in five different months in the year 2005. Data are given in hundreds

11. Number of tourists in City B in April are what percent more than that in City A in the same month ?

(A)  25

(B)  35

(C)  20

(D)  30

(E)  45

Ans: (E)

12. In the month of December of the previous year (2004), the total number of tourists in both the cities together were 25% more than that in January 2005. What was the number of tourists in both the cities together in December 2004

(A)  57,500

(B)  52,000

(C)  60,500

(D)  54,250

(E)  60,000

Ans: (A)

13. What is the difference between total number of tourists in cities A and B together in January and that in February ?

(A)  2, 500

(B)  2,200

(C)  1,000

(D)  1,500

(E)  2,000

Ans: (E)

14. What is the average number of tourists in city A in February and March ?

(A)  25,000

(B)  28,000

(C)  27,000

(D)  26,000

(E)  28,500

Ans: (C)

15. What is the respective ratio between the total number of tourist in city A in April and May together and that in City Bin the same month together ?

(A)  7 : 16

(B)  7 : 12

(C)  5 : 12

(D)  8 : 13

(E)  7 : 8

Ans: (B)

16. The number of tourists in City B in March is what percent less than that in the same city in May ?

(A)  25

(B)  20

(C) 

(D)   

(E)  

Ans: (C)

Directions – (Q. 17-21) What approximate value will come in place of question mark (?) in the given questions ? (You are not expected to calculate the exact value)

17. √? = (1346.92 + 46.94) ÷ 99.9 – 6.98

(A)  121

(B)  441

(C)  1024

(D)  49

(E)  196

Ans: (D)

18. 012 × 5121/3 × 33.992 ÷ (29 × 16.972) = 2?

(A)  3

(B)  4

(C)  9

(D)  10

(E)  6

Ans: (E)

19. (14.99% of 4799.995) ÷ ? = (170% of 7.111)2

(A)  150

(B)  25

(C)  100

(D)  50

(E)  5

Ans: (E)

20. 3/20 of 239 = ? ÷ (1.6 ×499)

(A)  30

(B)  300

(C)  600

(D)  120

(E)  80

Ans: (A)

21. 

(A)  80

(B)  8

(C)  4

(D)  120

(E)  40

Ans: (E)

22. In the year 2013, the population of a village A was 30% more than the population of village B. The population of village A in 2014 increased by 20% as compared to the previous year. If the population of Village A in 2014 was 7176, what was the population of village B in 2013 ?

(A)  4000

(B)  5000

(C)  4800

(D)  4600

(E)  5200

Ans: (D)

23. Tom gave 20% of a certain amount of money to Ali. From the money Ali received, he spent 25% on school fees and 35% on buying school uniform. After the mentioned expenses, Ali had Rs. 2800 remaining with him. How much money did Tom have initially ?

(A)  Rs. 21,000

(B)  Rs. 35,000

(C)  Rs. 30,000

(D)  Rs. 27,000

(E)  Rs. 28,000

Ans: (B)

24. A and B started a business with an investment of Rs. 3500 and Rs. 2500 respectively. After 4 months C joined with Rs. 6000. If the difference between C’s share and B’s share in the annual profit was Rs. 1977, what was the total annual profit ?

(A)  Rs. 15,620

(B)  Rs. 16,240

(C)  Rs. 14,490

(D)  Rs. 12,770

(E)  Rs. 13,180

Ans: (E)

Directions – (Q. 25-31) Study the table and answer the given questions.

Data regarding number of books sold in either hard bounds or paperback editions and also the categories of books sold in fiction and non-fiction category by four different shops, in a particular month (February 2015).

Book

Shops

Respective ratio between number of hard bounds sold and number of paperbacks sold Number of paperbacks sold out of total %of fictions (hard-bound + paperback) sold out of total books sold
A

B

C

D

2 : 3

1 : 5

1 : 3

3 : 5

1200

1000

600

675

60

65

70

75

Note – (i) Total books sold = number of hard bounds + number of paperbacks sold

25. Number of hard bound editions sold by shop C is what percent less than that sold by shop A ?

(A)  90

(B)  50

(C)  75

(D)  70

(E)  80

Ans: (C)

26. Number of non-fictions sold by shop B is what percent of number of non-fictions sold by shop A ?

(A)  52.5

(B)  64.75

(C)  50

(D)  40

(E)  60

Ans: (A)

27. Total number of books sold by shop B is what percent more than that sold by shop D ?

(A)  

(B)  25

(C)  30

(D)  10

(E)  

Ans: (E)

28. What is the average number of fictions sold by shops A and B ?

(A)  900

(B)  720

(C)  880

(D)  1100

(E)  990

Ans: (E)

29. In March 2015, the number of paperback editions sold by shop D was 20% more than the same sold by the same shop in the previous month. The number of paperback editions sold in March 2015 by shop D constituted 50% of the total number of books sold by shop D in March 2015. What was the total number of books sold in March 2015 by shop D ?

(A)  1860

(B)  1240

(C)  1861

(D)  1600

(E)  1620

Ans: (E)

30. What is the respective ratio between the number of non-fictions sold by shop C and number of non-fictions sod by shop D ?

(A)  7 : 9

(B)  8 : 9

(C)  6 : 7

(D)  8 : 11

(E)  3 : 5

Ans: (B)

31. The distance between two places A and B is 140 kms. 1st scooter departs from place A to B, at a speed of 50 kmph at 10 a.m. 2nd scooter departs from place B to A at a speed of 30 kmph at 12 pm. At what time will both the scooters meet each other ?

(A)  12 : 30 pm

(B)  01 : 50 pm

(C)  1 : 00 pm

(D)  12 : 50 pm

(E)  1 : 30 pm

Ans: (A)

32. Two mobile phones were purchased at the same price. One was sold at a profit of 30% and the second was sold at a price which was Rs. 2500 less than the price at which the first was sold. If the overall profit earned by selling both the mobile phones was 5%, what was the cost price of one mobile phone ?

(A)  Rs. 8000

(B)  Rs. 5000

(C)  Rs. 6000

(D)  Rs. 4500

(E)  Rs. 5500

Ans: (B)

Directions (Q. 33-38) In these questions two equation numbered I and II are given. You have to solve both the equations and mark the appropriate option.

33. I. 2x2 – 21x + 52 = 0

II. 2y – 11y + 12 = 0

(A)  x > y

(B)  x ≤ y

(C)  x ≥ y

(D)  x < y

(E)  Relationship between x and y cannot be determined

Ans: (C)

34. I. 3x2 – 13x + 14 = 0

II. 2y2 – 5y + 3 = 0

(A)  x > y

(B)  x ≤ y

(C)  x ≥ y

(D)  x < y

(E)  Relationship between x and y cannot be determined

Ans: (A)

35. I. 4x2 – 8x + 3 = 0

II. 4y2 – 15y + 14 = 0

(A)  x > y

(B)  x ≤ y

(C)  x ≥ y

(D)  x < y

(E)  Relationship between x and y cannot be determined

Ans: (D)

36. I. 2x2 – 9x + 9 = 0

II. y2 – 7y + 12 = 0

(A)  x > y

(B)  x ≤ y

(C)  x ≥ y

(D)  x < y

(E)  Relationship between x and y cannot be determined

Ans: (B)

37. I. 4x2 + 19x + 22 = 0

II. 2y2 + 11y + 15 = 0

(A)  x > y

(B)  x ≤ y

(C)  x ≥ y

(D)  x < y

(E)  Relationship between x and y cannot be determined

Ans: (E)

38. I. x2 + 8x + 15 = 0

II. y2 + 3y + 2 = 0

(A)  x > y

(B)  x ≤ y

(C)  x ≥ y

(D)  x < y

(E)  Relationship between x and y cannot be determined

Ans: (D)

Directions – (Q. 39-44) What will come in place of (?) in the given number series ?

39. 606 201  66  21  6  ?

(A)  0.5

(B)  2

(C)  5

(D)  1

(E)  3

Ans: (D)

40. 19 10  11  18  38  ?

(A)  97.5

(B)  110

(C)  115

(D)  124.5

(E)  99.5

Ans: (A)

41. 6 4  5  11  ?  189

(A)  44

(B)  39

(C)  65

(D)  96

(E)  62

Ans: (B)

42. 24 26  20  32  12  ?

(A)  42

(B)  54

(C)  56

(D)  46

(E)  64

Ans: (A)

43. 18 19  24  37  66  ?

(A)  158

(B)  224

(C)  219

(D)  192

(E)  127

Ans: (E)

44. 19 6  20.8  23.2  28  ?

(A)  35.7

(B)  37.6

(C)  31.8

(D)  39.8

(E)  38.2

Ans: (B)

Directions – (Q. 45-49) These questions consists of a question and two statements I and II given below it. You have to decide whether the data provided in the statements are sufficient to answer the question.

Read the both the statements and choose the appropriate option.

(A) if statement I alone in sufficient to answer the question.

(B) if statement II alone is sufficient to answer the question.

(C) if both statement I and II are needed to answer the question.

(D) If the given data is not sufficient to answer the question.

(E) Either statement I or statement II in needed to answer the question.

45. What is the curved surface area of the right circular cylinder ?

I. Area of the base of the cylinder is 154 cm2.

II. Volume of the cylinder is 1232 cm2.

Ans: (C)

46. What is the area of the circular field ?

I. Area of the largest square that can be inscribed in the given square field is 2450 cm2.

II. Are of the smallest square in which the given circular field can be inscribed is 4900 cm2.

Ans: (B)

47. In how many days can ‘B’ alone complete the work ?

I. A , B and C together can complete the work in  days, A and B together can complete the work in  days, B and C together can complete it in 

II. A, B and C together can complete the piece of work in  days, A and C together can complete the work in 

Ans: (E)

48. What was the initial quantity of mixture of juice and water ?

I. Juice and water were in the ratio of 8 : 3 respectively in the mixture initially.

II. When 11 litres of mixture is taken out and 6 litres of juice is added, the ratio between juice and water becomes 10 : 3 respectively.

Ans: (C)

49. How much money did Mr. Rajesh receive as retirement funds ?

I. Out of the total money received Mr. Rajesh invested 51% in various schemes, gave 35% of the total money received to his wife and remaining money he spent on miscellaneous items.

II. Out of the total money received, Mr. Rajesh spent 5% on son’s wedding and gave 35% to his wife. Out of the remaining, he invested 28% in insurance policies, 22% in pension fund scheme, 35% in share market and remaining Rs. 4,500,000 he spent on furnishing the house.

Ans: (B)

50. The speed of a boat in still water is 15 km/h and the speed of the current is 3 km/h. The distance travelled by the boat from point A to point B downstream is 24 km more than the distance covered by the same boat from point B to point C upstream in the same time. How much time will the boat take to travel from C to B downstream ?

(A)  2 hrs

(B)  2 hrs 30 mins

(C)  2 hrs 40 mins

(D)  2 hrs 10 mins

(E)  3 hrs 20 mins

Ans: (C)

IBPS Specialist Officer (Marketing) Examination Held on 1-2-2016 Professional Knowledge Question Paper with Answer Key

IBPS Specialist Officer (Marketing) Examination
IBPS Specialist Officer (Marketing) Examination Held on 1-2-2016 Professional Knowledge Question Paper with Answer Key

IBPS Specialist Officer (Marketing) Exam., 2015

(Held on 1-2-2016)

Professional Knowledge

1. A tangible product that the consumer feels comfortable purchasing without gathering additional information and then actually buys with minimum of effort is termed a(n)-

(A)  Shopping good

(B)  Convenience good

(C)  Business product

(D)  Specially good

(E)  Unsought good

Ans: (B)

2. The process of dividing a market into distinct groups of buyers who have different needs, characteristics or behaviours and who might require separate products or marketing programs is called –

(A)  Market segmentation

(B)  Market positioning

(C)  Market differentiation

(D)  Market targeting

(E)  Marketing mix

Ans: (A)

3. …………. is not a stage of product life cycle in marketing.

(A)  introduction

(B)  maturity

(C)  Growth

(D)  Decline

(E)  Planning

Ans: (E)

4. Which of the following is not a characteristic important in influencing an innovator’s rate of adoption ?

(A)  Organization type of the innovator

(B)  Compatibility

(C)  Divisibility

(D)  Communicability

(E)  Complexity

Ans: (A)

5. The financial contracts whose values are obtained from the values of underlying assets are…..

(A)  mortgage

(B)  Commercial papers

(C)  Stocks

(D)  Derivatives

(E)  Bonds

Ans: (D)

6. All of the following are considered to be broad market follower strategies EXCEPT-

(A)  innovator

(B)  imitator

(C)  cloner

(D)  counterteller

(E)  adapter

Ans: (E)

7. A company has four choices when it comes to developing brands. Which one of the following is not one among them ?

(A)  New brands

(B)  Multi brands

(C)  Brand extension

(D)  Line extension

(E)  Brand sponsoring

Ans: (E)

8. ……….. is a demand state where consumer may share a strong need that cannot be satisfied by an existing model ?

(A)  declining demand

(B)  irregular demand

(C)  negative demand

(D)  non-existent demand

(E)  latent demand

Ans: (E)

9. …….. is the development, design and implementation of marketing programme, processes and activities that recognises the breadth and interdependencies of today’s marketing environment.

(A)  Niche marketing

(B)  Supply chain marketing

(C)  Holistic marketing

(D)  Relationship marketing

(E)  Demand-centred marketing

Ans: (C)

10. The ………. holds that marketing strategy should deliver value to customers in a way that maintain or improve both the consumer’s society’s well-being.

(A)  Societal marketing concept

(B)  Society centered marketing

(C)  Customer-centered marketing

(D)  Focused business model

(E)  Production centered business

Ans: (C)

11. Major oil producers carry an oil exploration, oil drilling, oil refining, chemical manufacture and service-station operation. When an organization does all of these separate tasks distribution channel they can be said to have achieved what is called –

(A)  parallel marketing

(B)  horizontal integration

(C)  vertical integration

(D)  concentric integration

(E)  conglomerate marketing

Ans: (C)

12. Which of the following describes the tendency of people to interpret information in a way that will support what they already believe ?

(A)  Selective creativity

(B)  Selective distortion

(C)  Selective matching

(D)  Selective attraction

(E)  Selective attribute

Ans: (B)

13. The ……… stage of a product is characterized as being one where there is period of rapid climb in sales and substantial profit improvement.

(A)  introduction

(B)  decline

(C)  maturity

(D)  saturation

(E)  growth

Ans: (E)

14. The discomfort caused by post purchase conflict is called –

(A)  Dissatisfaction

(B)  Cognitive Dissonance

(C)  Post purchase behaviour

(D)  Buyer’s remorse

(E)  Comparative tension

Ans: (B)

15. ……… is the study of how individuals, groups and organizations select, buy, use and dispose of goods, services, ideals or experiences tot satisfy their needs and wants.

(A)  Psychology

(B)  Product differentiation

(C)  Psychographic segmentation

(D)  Target marketing

(E)  Consumer behaviour

Ans: (E)

16. Mutual Funds are regulated in the country by-

(A)  IRDA

(B)  Association of Mutual Funds of India (AMFI)

(C)  NABARD

(D)  Securities and Exchange Board of India

(E)  Reserve Bank of India

Ans: (D)

17. Situation in which a firm introduces new products to stimulate sales but the profit comes at the expense of other products sold by that firm is called –

(A)  Repositioning

(B)  Push marketing

(C)  Differentiated marketing

(D)  Cannibalization

(E)  Cartelization

Ans: (D)

18. Cultivating opinion leaders and getting them to spread information about a product or service to others in their communities is –

(A)  Buzz Marketing

(B)  Leader Marketing

(C)  Niche Marketing

(D)  Complex Marketing

(E)  Selective Marketing

Ans: (A)

19. Marketing management is defined as-

(A)  Monitoring the profitability of the company’s products and services

(B)  The art and science of choosing target markets and building profitable relationship with them

(C)  Managing the market process

(D)  Developing marketing strategies to move the company forward

(E)  Selecting target markets to deploy marketers

Ans: (D)

20. The measure of the brand’s ability to capture consumer performance and loyalty is termed –

(A)  Brand superiority

(B)  Brand equity

(C)  Brand version

(D)  Brand loyalty

(E)  Brand preference

Ans: (D)

21. Mission statements are at their best when they reflect a ……. an almost impossible dream that provides direction for the next 10 to 30 years.

(A)  Market

(B)  Strength

(C)  Competency

(D)  Vision

(E)  Value

Ans: (D)

22. A strategy in which the initial price of the product is set low in relation to the target market’s range of expected price is termed –

(A)  Market Skimming Pricing

(B)  Old Pricing

(C)  Discount market pricing

(D)  Market Penetration Pricing

(E)  Special market Pricing

Ans: (C)

23. A group of tradition bound consumers who are the tool to adapt an innovation is called –

(A)  Challengers

(B)  Leaders

(C)  Innovators

(D)  Laggards

(E)  Loyal agents

Ans: (D)

24. Low prices on products for which you have accurate price knowledge lets you know that the product has to be a bargain. This type of pricing is termed-

(A)  Reference pricing

(B)  Psychological pricing

(C)  Volume pricing

(D)  Promotional pricing

(E)  Signpost pricing

Ans: (A)

25. The want satisfying power of product is called its –

(A)  Price

(B)  Utility

(C)  Bond

(D)  Purpose

(E)  Goal

Ans: (B)

26. Based on the rates firms play in the target market in a specific industry, they are classified as following except in a-

(A)  Market challenger

(B)  Market plotter

(C)  Market leader

(D)  Market nicher

(E)  Market follower

Ans: (B)

27. TQM approach in which all the people of the company are involved in constantly improving the performance of products, services and business processes. TQM stands for-

(A)  Tata Quality Management

(B)  Total Quality Marketing

(C)  Total Quantity Management

(D)  Total Queries Management

(E)  Total Quality Manipulation

Ans: (*)

28. When backed by buying power, wants become-

(A)  self-esteem needs

(B)  demands

(C)  exchanges

(D)  physical needs

(E)  social needs

Ans: (B)

29. The philosophy of product centered concept is……….

(A)  Right products for the customers

(B)  Make and sell

(C)  Make what the market wants

(D)  Sense and respond

(E)  Selling on product benefits

Ans: (B)

30. The customer’s evaluation of the difference between all the benefits and all the costs of a market offering relative to those of competing offers is called –

(A)  Customer perceived value

(B)  Customer expectation

(C)  Customer satisfaction

(D)  Customer Loyalty

(E)  Customer relationship

Ans: (A)

31. Button market is where ………

(A)  capital is disposed

(B)  gold is purchased and sold

(C)  shares are purchased

(D)  production of silver takes place

(E)  investments are made

Ans: (E)

32. Especially in large firms, a sales force frequently is specialized in some organizational fashion. Which of the following is not one of such specializations ?

(A)  Customer specialization

(B)  Major accounts specialization

(C)  Product specialization

(D)  Geographic specialization

(E)  Economic order specialization

Ans: (B)

33. Greater consumer control means that, in building customer relationships, companies can no longer rely on marketing by intrusion. The most practice marketing is-

(A)  Conversion

(B)  Delivery

(C)  Retention

(D)  Attraction

(E)  Attention

Ans: (D)

34. According to Maslow’s Hierarchy of Needs, sense of belonging is grouped in-

(A)  Esteem needs

(B)  Social needs

(C)  Safety needs

(D)  Physiological needs

Ans: (B)

35. Michael Porter has identified five forces that determine the intrinsic long-run attractiveness of market segment. Which of the following would not be among Porter’s Five forces

(A)  Threat of buyer’s growing bargaining power

(B)  Threat of substitute products

(C)  Threat of technological partners

(D)  Threat of intense segment rivalry

(E)  Threat of new entrants

Ans: (C)

36. The unplanned static or distortion during the communication process, which results in the receivers’ getting a different message then the one that the sender sent is the-

(A)  Response

(B)  Feedback

(C)  Noise

(D)  Decoding

Ans: (C)

37. If a MNC like Nestle varied the elements of the marketing mix to suit the countries in which it sold its products, which type of strategy would it be following ?

(A)  Differentiated

(B)  undifferentiated        

(C)  Standardized

(D)  Adapted

(E)  Concentrated

Ans: (C)

38. Under which of the following company orientations towards the market place would we expect to get ‘better economical’ talacy?

(A)  Marketing concept

(B)  Holistic marketing concept

(C)  Production concept

(D)  Selling concept

(E)  Product concept

Ans: (B)

39. If a company focuses only one or a few market segments instead of small share of a large market, the same is-

(A)  Segment marketing

(B)  Conceptual marketing

(C)  Mass Marketing

(D)  Micro Marketing

(E)  Niche Marketing

Ans: (A)

40. An estimate the total financial value of the brand is-

(A)  Brand valuation

(B)  Brand tracking

(C)  Brand auditing

(D)  Brand partitioning

(E)  Brand equity

Ans: (E)

41. The total combined customer value of all the company’s current and potential customers is called –

(A)  Customer share

(B)  Customer delight

(C)  Customer loyalty

(D)  Customer orientation

(E)  Customer equity

Ans: (C)

42. We can say that a………… has three characteristics. (1) It is source of competitive advantage in that it makes a significant contribution to perceived customer benefits; (2) It has application in a wide variety of markets, and (3) It is difficult for competitors to imitate.

(A)  Strategic business unit

(B)  Core competency

(C)  Business strategy

(D)  Winning strategy

(E)  Core technology

Ans: (B)

43. Which demographic segmentation divides buyers into different groups based on social class, lifestyle or personality characteristics ?

(A)  Age and life cycle segmentation

(B)  Gender segmentation

(C)  Age segmentation

(D)  Psychographic segmentation

(E)  Income segmentation

Ans: (D)

44. The internet version of word of mouth marketing is termed –

(A)  Viral marketing

(B)  Web marketing

(C)  Channel marketing

(D)  Network marketing

(E)  Virtual marketing

Ans: (A)

45. ……….. is the development, interpretation and communication of decision oriented information to be used in all phases of marketing process.

(A)  Marketing forecast

(B)  Market intelligence

(C)  Market Planning

(D)  Marketing research

(E)  Marketing information system

Ans: (E)

46. GATT is more than 60 years old treaty designed to promote world  trade by reducing tariff and other international trade barriers. GATT stands for-

(A)  General Aspects on Tariff and Trade

(B)  General Agreement on Traffic and Trade

(C)  Geneva Agreement on Trade and Tariff

(D)  General Allowance for Trade and Tariff

(E)  Geneva Agreement on Traffic or Trade

Ans: (B)

47. People differ greatly in their readiness to try new products. Which of the following do not belong to the adapter categorization on the basis of relation time of adopter of innovators ?

(A)  Innovators

(B)  Laggards

(C)  Early Majority

(D)  Late Majority

(E)  Super Majority

Ans: (C)

48. As per BCG Matrix of classification of SBUs of an organization, Cash cows are SBUs that typically generate-

(A)  Large Awareness levels but few sales

(B)  Problems for product managers often

(C)  High industry growth

(D)  More cash than that can be invested profitably in its own business

(E)  Paper losses in the long run

Ans: (D)

49. A strategy of ……… pricing involves using prick in a competitive weapon in order to push competitors out of market or eliminate competitors.

(A)  Bargain

(B)  Psychological

(C)  Premium

(D)  Advanced

(E)  Predatory

Ans: (B)

50. Behavioural basis of market segmentation includes-

(A)  Loyalty or brand loyalty

(B)  Attitude

(C)  Buying occasions     

(D)  All of these

Ans: (B)

IBPS R.R.Bs. Probationary Officers (Pre.) Examination Held on 5-11-2016 Reasoning Question Paper and Answer Key

IBPS R.R.Bs. Probationary Officers (Pre.) Examination Held on 5-11-2016 Reasoning Questin Paper and Answer Key
IBPS R.R.Bs. Probationary Officers (Pre.) Examination Held on 5-11-2016 Reasoning Question Paper and Answer Key

IBPS R.R.Bs. Probationary Officers (Pre.) Exam., 2016

Held on 5-11-2016

Reasoning

1. Among five people – A, B, C, D and E, each scoring different marks, only one person scored less marks than B. D scored more than B but less than A. A did not score the highest. Who scored the second highest ?

(A)  E

(B)  Cannot be determined

(C)  A

(D)  C

(E)  D

Ans: (C)

Directions – Study the given information carefully to answer the given question :

A is 14 m east of B, C is 6 m South of A. P is 4 m West of C. Point C is the midpoint of Points P and H. Such that points P, C and H form a straight line. Q is 6 m South of Point H.

2. Kunal walks 10 m towards North from Point H, takes a left turn and walks for 4 m. How far will he be from Point C ?

(A)  2 m

(B)  10 m

(C)  4 m

(D)  6 m

(E)  7 m

Ans: (B)

3. If A is 2 m to the North of L and R is 4 m West of Q, how far is Point L from Point R ?

(A)  4 m

(B)  10 m

(C)  7 m

(D)  14 m

(E)  9 m

Ans: (B)

4. In which direction is B with respect to H ?

(A)  North-east

(B)  South-east

(C)  North-west

(D)  North

(E)  West

Ans: (C)

Directions – Study the following information and answer the given question :

Nine friends – P, Q, R, S, T, U, V, W and X live on nine different floors of a building but not necessarily in the same order. The lower most floor of the building is numbered one, the one above that is numbered two and so on till the topmost floor is numbered nine.

T lives on an odd numbered floor below the floor numbered five. Only three people live between T and R. Only one person lives between X and V. V lives above X. Q lives on an odd numbered floor immediately below P. Q does not live on the floor numbered five. The number of persons living between X and P is equal to the number of people living between T and S. W lives on an even numbered floor immediately above R.

5. S is related to the floor numbered six and R is related to the floor numbered eight in a certain way X is related to which of the following floor numbers following the same way ?

(A)  Seven

(B)  Nine

(C)  Five

(D)  Eight

(E)  One

Ans: (C)

6. U lives on which of the following floor numbers ?

(A)  Five

(B)  Four

(C)  Nine

(D)  Three

(E)  Other than those given as options

Ans: (C)

7. Four of the following five are alike in a certain way as per the given arrangement and thus form a group. Which of the following does not belong to that group ?

(A)  QU

(B)  VW

(C)  SR

(D)  PX

(E)  XV

Ans: (A)

8. How many persons live below the floor on which W lives ?

(A)  One

(B)  Two

(C)  More than three

(D)  Three

(E)  None of the above

Ans: (C)

9. Which of the following statements is true as per the given arrangement ?

(A)  Only three people live between S and V

(B)  None of the given statements is true

(C)  Q lives immediately above T.

(D)  More than three persons live above R

(E)  U lives on the floor numbered nine

Ans: (E)

10. How many such pairs of digits are there in the number 71864392 (Both in forward and backward directions), each of which has as many digits between them as in the arithmetic series ?

(A)  More than three

(B)  None

(C)  Three

(D)  One

(E)  Two

Ans: (A)

11. Four of the following five are alike in a certain way (based on their positions of alphabets in the English alphabetical series) and hence form a group. Which is the one that does not belong to that group ?

(A)  RQT

(B)  NPL

(C)  FHD

(D)  KMI

(E)  WYU

Ans: (A)

Directions – Study the following information to answer the given question :

Nine persons, B, C, D, E, L, M, N, O and P are seated in a straight line facing north, with equal distance between each other, but not necessarily in the same order.

As many people sit to the left of E as to the right of E. Only one person sits between E and O. L sits third to the left of P. P is not an immediate neighbor of O. Neither P nor D sits at any of the extreme ends of the line. Only three persons sits between B and C. B is not an immediate neighbor of M. The number of people sitting between B and E is double as that between E and N.

12. Who sits third to the right of B ?

(A)  E

(B)  O

(C)  N

(D)  L

(E)  No one as B sits at one of the extreme ends of the line

Ans: (A)

13. Four of the following five are alike in a certain way based on the given arrangement and thus form a group. Which is the one that does not belong to that group ?

(A)  C, E

(B)  M, D

(C)  O, C

(D)  B, L

(E)  D, B

Ans: (E)

14. Which of the following is true with respect to M as per the given arrangement ?

(A)  M sits second to the left of O

(B)  M is an immediate neighbour of L

(C)  More than two people sit between D and M.

(D)  None of the given options is true

(E)  M sits at one of the extreme ends of the line

Ans: (E)

15. Who amongst the following sit exactly between L and P ?

(A)  E, N

(B)  M, O

(C)  B, N

(D)  C, O

(E)  D, E

Ans: (C)

16. In which of the given pairs of people, is odd number of people sitting between them ?

(A)  E, M

(B)  B, M

(C)  L, C

(D)  L, P

(E)  P, O

Ans: (A)

Directions – Study the following information to answer the given question :

Gaurav watches seven movies vis., Gladiator, Braveheart, Titanic Inception, Chinatown, Avatar and Passion on seven different days of the same week, starting from Monday and ending on Sunday, but not necessarily in the same order. Thus on one day the watches only one movie.

Gaurav watches inception on Friday. He watches only one movie between Inception and Titanic. He watches only three movies between Titanic and Gladiator. He watches only two movies between Titanic and Chinatown. Gaurav watches Avatar immediately before the day he watches Titanic. He watches Passion on one of the days after he watches Avatar.

17. How many movies does Gaurav watch between Braveheart and Passion ?

(A)  None

(B)  Three

(C)  One

(D)  Two

(E)  Four

Ans: (D)

18. Four of the following five are alike in a certain way based on the given arrangement and thus form a group. Which is the one that does not belong to that group ?

(A)  Wednesday – Avatar

(B)  Friday – Braveheart

(C)  Thursday – Titanic

(D)  Saturday – Inception

(E)  Sunday – Chinatown

Ans: (B)

19. Which of the following is true as per the given arrangement ?

(A)  Gaurav watches inception on the day immediately after the day on which he watches Braveheart

(B)  None of the given statements are true

(C)  Gaurav watches only two movies between Avatar and Chinatown

(D)  Gaurav watches Passion on Wednesday

(E)  Gaurav watches Gladiator on Sunday

Ans: (E)

20. On which day of the week does Gaurav watch Avatar ?

(A)  Saturday

(B)  Sunday

(C)  Tuesday

(D)  Wednesday

(E)  Thursday

Ans: (C)

21. Which movie does Gaurav watch on Monday ?

(A)  Chinatown

(B)  Gladiator

(C)  Titanic

(D)  Braveheart

(E)  Passion

Ans: (D)

Directions- In this question, relationship between different elements is shown in the statement(s). The statement(s) are followed by conclusions. Study the conclusions based on the given statements and select the appropriate answer and answer as :

(A) Only  conclusion I follows.

(B) Only conclusion II follows.

(C) Either conclusion I or II follows.

(D) Neither conclusion I nor II follows.

(E) Both conclusion I and II follows.

22. Statement : T = V ≤ R < B, T ≤ D

Conclusions : I. D ≥ R

                         II. B ≤ T

Ans: (D)

23. Statement : R < O ≥ E; Y ≥ O > C.

Conclusions : I. Y > R

                         II. E < C

Ans: (A)

24. Statement : P > R = S ≥ Q; M ≤ R

Conclusions : I. M < P

                         II. Q ≤ M

Ans: (A)

25. Statement : H < M ≥ I > N = C ≥ R

Conclusions : I. C < H

                         II. M > R

Ans: (B)

26. Statement : R < O ≥ E; Y ≥ O > C

Conclusions : I. Y = E

                         II. Y > E

Ans: (C)

27. In a certain code language, ‘give me call’ is coded as ‘jo kl mx’ and ‘call for me’ is coded as ‘mx jo st’. How will ‘for’ be coded as in the given code language ? (Note : all codes are two letter codes only)

(A)  Either ‘mx’ or ‘jo’

(B)  Either ‘jo’ or ‘kj’

(C)  kj

(D)  mx

(E)  st

Ans: (E)

Directions – In this question, two/three statements followed by two conclusions are given. You have to take the given statements to be true even if they seem to be at variance from commonly known facts and then decide which of the given conclusions logically follows disregarding commonly known facts and select the appropriate option and answer as :

(A) Only conclusion I follows.

(B) Only conclusion II follows.

(C) Either conclusion I or II follows.

(D) Neither conclusion I nor II follows.

(E) Both conclusion I and II follows.

28. Statement : No horse is a goat. Some goats are deers.

Conclusions :

I. No horse is a deer.

II. Atleast some horses are deers.

Ans: (C)

29. Statement : Some trees are branches. Some branches are roads. No road is a sky.

Conclusions :

I. No tree is a sky.

II. Some branches are definitely not skies.

Ans: (D)

30. Statement : All plates are spoons. All spoons are glasses. Some glasses are mugs.

Conclusions :

I. All plates being mugs is a possibility.

II. All glasses are plates.

Ans: (D)

31. Statement : All plates are spoons. All spoons are glasses. Some glasses are mugs.

Conclusions :

I. Some mugs are spoons.

II. No mug is a spoon.

Ans: (C)

32. Statement : Some trees are branches. Some branches are roads. No road is a sky.

Conclusions :

I. All roads are trees.

II. All skies are branches.

Ans: (D)

Directions – Study the given information carefully to answer the given question :

K and M are the children of G. G is married to R. S is the sister of G. A is the only son of R. P is the son of K.

33. How is M related to P ?

(A)  Father

(B)  Uncle

(C)  Aunt

(D)  Grandmother

(E)  Brother-in-law

Ans: (C)

34. How is S related to R ?

(A)  Sister-in-law

(B)  Daughter-in-law

(C)  Niece

(D)  Daughter

(E)  Granddaughter

Ans: (A)

35. If S does not have any sister, then how is G related to P ?

(A)  Uncle

(B)  Grandmother

(C)  Father-in-law

(D)  Aunt

(E)  Grandfather

Ans: (E)

Directions – Study the given information carefully to answer the given question :

Eight people – M, N, O, P, Q, R, S and T are sitting around a square table (but not necessarily in the same order) in such a way that four of them sit at the corners while four sit in the middle of each of the four sides. The ones sitting at the corners are facing the centre and the ones sitting in the middle of the sides are facing outside. (i.e., opposite to the centre)

O sits in the middle of one of ht sides. Only two people sit between O and R. T sits to the immediate right of R. T and S face the same direction. N sits to the immediate left of S. P is an immediate neighbor of N. M sits third to the right of P.

36. What is the position of P with respect to T ?

(A)  Third to the left

(B)  Second to the left

(C)  Immediate left

(D)  Immediate right

(E)  Second to the right

Ans: (E)

37. Which of the following pairs represent the people sitting between O and the one sitting to the immediate right of P, when counted from the left of O ?

(A)  N, S

(B)  R, N

(C)  S, M

(D)  T, Q

(E)  R, T

Ans: (C)

38. How many people sit between T and M when counted from the right of T ?

(A)  More than three

(B)  Two

(C)  One

(D)  Three

(E)  None of the above

Ans: (A)

39. Which of the following statements is true with respect to the given arrangement ?

(A)  None of the given statements is true

(B)  Q is an immediate neighbor of T

(C)  S sits at one of the corners of the table

(D)  Only two people sit between M and Q

(E)  R and N face opposite directions

Ans: (B)

40. Who sits third to the left of O ?

(A)  R

(B)  M

(C)  P

(D)  N

(E)  S

Ans: (D)

I.B.P.S. Bank Probationary Officers (Pre.) Examination, 2015 Question Paper with Answer Key

IBPS PI.B.P.S. Bank PO Pre

I.B.P.S. Bank Probationary Officers (Pre.) Examination, 2015 Question Paper with Answer Key

I.B.P.S. Bank Probationary Officers (Pre.) Exam., 2015

Held on 3-10-2015

Quantitative Aptitude

 

 

Directions – (Q. 1- 5) Study the table and answer the given questions :

 

Note – Few values are missing in the table (indicated by). A candidate is expected to calculate the missing value, If it is required to answer the given question on the basis of given data and information.

1. Out of the number of qualified candidates from Sate P in 2008, the respective ratio of male and female candidates is 11 : 7. If the number of female qualified candidates from State P in 2008 is 126, what is the number of appeared candidates (both male and female) from the state P in 2008?

(A)  630

(B)  510

(C)  570

(D)  690

(E)  540

Ans: (E)

2. Number of appeared candidates from state Q increased by 100% from 2006 to 2007. If the total number of qualified candidates from State Q in 2006 and 2007 together is 408, what is the number of appeared candidates from State Q in 2006?

(A)  380

(B)  360

(C)  340

(D)  320

(E)  300

Ans: (C)

3. What is the difference between number of qualified candidates from State P in 2006 and that in 2007?

(A)  12

(B)  22

(C)  14

(D)  24

(E)  16

Ans: (A)

4. If the average number of qualified candidates from State Q in 2008, 2009 and 2010 is 210, what is the number of qualified candidates from State Q in 2010 ?

(A)  191

(B)  195

(C)  183

(D)  187

(E)  179

Ans: (D)

5. If the respective ratio between number of qualified candidates from State P in 2009 and 2010 is 14 : 9, what is the number of qualified candidates from State P in 2010?

(A)  252

(B)  207

(C)  216

(D)  234

(E)  198

Ans: (C)

6. 

(A)  15

(B)  10

(C)  7

(D)  4

(E) 12

Ans: (E)

7. 012 – 19.982 – ? = 21.812

(A)  49

(B)  50

(C)  30

(D)  39

(E)  41

Ans: (*)

8. 15 + 2379.85 + 140.01 × 4.99 = ?

(A)  4400

(B)  3900

(C)  3000

(D)  4000

(E)  4300

Ans: (B)

9. 97% of 649.8 ÷ 13.05 = 45.12 – ?

(A)  40

(B)  15

(C)  25

(D)  10

(E)  30

Ans: (C)

10. (674.87 + 59.98) ÷02 = ?

(A)  29

(B)  27

(C)  19

(D)  21

(E)  11

Ans: (D)

Directions (Q. 11-15) : Refer to the graph and answer the given questions.


11. The number of people who travelled by Train B on Friday is 20% more than the people who travelled by the same train on Thursday. What is the respective ratio between the number of people who travelled on Friday and those who travelled on Saturday by the same train ?

(A)  4 : 5

(B)  3 : 4

(C)  5 : 6

(D)  3 : 5

(E)  1 : 4

Ans: (B)

12. What is the difference between the total number of people who travelled by Train B on Monday and Tuesday together and the total number of people who travelled by Train A on Saturday and Sunday together?

(A)  200

(B)  230

(C)  210

(D)  250

(E)  240

Ans: (D)

13. What is the average number of people travelling by Train A on Monday, Tuesday, Wednesday and Thursday ?

(A)  220

(B)  190

(C)  205

(D)  195

(E)  210

Ans: (C)

14. The number of people who travelled by Train A decreased by what per cent from Saturday to Tuesday ?

(A)  35

(B)  40

(C)  30

(D)  42

(E)  33

Ans: (B)

15. The total number of people who travelled by both the given trains together on Sunday is approximately what per cent more than the total number of people who travelled by both the given trains together on Wednesday?

(A)  128

(B)  123

(C)  142

(D)  118

(E)  135

Ans: (B)

16. 6,100 was partly invested in Scheme A at 10% p.a. compound interest (compounded annually) for 2 years and partly in Scheme B at 10% p.a. simple interest for 4 years. Both schemes equal interests. How much was invested in Scheme A ?

(A)  Rs. 3,750

(B)  Rs. 4,500

(C)  Rs. 4,000

(D)  Rs. 3,250

(E)  Rs. 5,000

Ans: (C)

17. ‘A’ bought a certain quantity of oranges at total cost of Rs. 1200. He sold 1/3rd of those oranges at 20% loss. If A earns an overall profit of 10%, at what per cent profit did A set the rest of the oranges ?

(A)  16%

(B)  15%

(C)  22%

(D)  25%

(E)  20%

Ans: (D)

18. Present ages of Kiran and Shyam are in the ratio of 5 : Three years hence, the ratio of their ages will become 11 : 9 respectively. What is Shyam’s present age in years ?

(A)  28

(B)  27

(C)  26

(D)  24

(E)  30

Ans: (D)

Directions – (Q. 19-23) In these questions, two equations numbered I and II are given. You have to solve both the questions and mark the appropriate option.

19. 2x2 + 19x + 45 = 0

      2y2 + 11y + 12 = 0

(A)  x > y

(B)  x ≥ y

(C)  x < y

(D)  Relationship between x and y cannot be determined

(E)  x ≤ y

Ans: (C)

20. 3x2 – 13x + 12 = 0

       2y2 – 15y + 28 = 0

(A)  x > y

(B)  x ≥ y

(C)  x < y

(D)  Relationship between x and y cannot be determined

(E)  x ≤ y

Ans: (C)

21. x2 = 16

      2y2 – 17y + 36 = 0

(A)  x > y

(B)  x ≥ y

(C)  x < y

(D)  Relationship between x and y cannot be determined

(E)  x ≤ y

Ans: (E)

22. 6x2 + 19x + 15 = 0

      3y2 + 11y + 10 = 0

(A)  x > y

(B)  x ≥ y

(C)  x < y

(D)  Relationship between x and y cannot be determined

(E)  x ≤ y

Ans: (B)

23. 2x2 – 11x + 15 = 0

      2y2 – 11y + 14 = 0

(A)  x > y

(B)  x ≥ y

(C)  x < y

(D)  Relationship between x and y cannot be determined

(E)  x ≤ y

Ans: (D)

24. A started a business. After 4 months from the start of the business, B and C joined. The respective ratio between the investments of A, B and C was 4 : 6 : 5. If A’s share in annual profit was Rs. 250 more than C’s share, what was the total annual profit earned ?

(A)  Rs. 3740

(B)  Rs. 3910

(C)  Rs. 4250

(D)  Rs. 5000

(E)  None of these

Ans: (C)

25. A person has to travel from point A to point B in certain time. Travelling at a speed of 5 kmph he reaches 48 minutes late and while travelling at a speed of 8 kmph he reaches 15 minutes early. What is the distance from point A to B?

(A)  15 km

(B)  9 km

(C)  12 km

(D)  18 km

(E)  14 km

Ans: (E)

26. 28 men can complete a piece of work in 15 days and 15 women can complete the same piece of work in 24 days. What is the respective ratio between the amount of work done by 30 men in 1 day and the amount of work done by 18 women in 1 day?

(A)  10 : 7

(B)  3 : 7

(C)  4 : 5

(D)  5 : 4

(E)  4 : 1

Ans: (A)

27. 18 litres of pure water was added to a vessel containing 80 litres of pure milk. 49 litres of the resultant mixture was then sold and some more quantity of pure milk and pure water was added to the vessel in the respective ratio of 2 : 1. If the resultant respective ratio of milk and water in the vessel was 4 : 1, what was the quantity of pure milk added in the vessel ? (in litres).

(A)  4

(B)  8

(C)  10

(D)  12

(E)  2

Ans: (A)

28. A certain sum is divided among A, B and C in such a way that A gets Rs. 40 more than the 1/2 of the sum. B get Rs. 120 less than 3/8th of the sum and C gas Rs. 200. What is the total sum.

(A)  Rs. 1100

(B)  Rs. 850

(C)  Rs. 960

(D)  Rs. 1200

(E)  Rs. 1600

Ans: (C)

Directions-(Q. 29-33) What will come in place of question mark (?) in the given number series ?

29. 123    140    106    55 – 13 ?

(A)  214

(B)  139

(C)  −98

(D)  −69

(E)  −74

Ans: (C)

30. 190    94      46      22      ?        4

(A)  19

(B)  15

(C)  10

(D)  8

(E)  16

Ans: (C)

31. 320   320    314    290    230    ?

(A)  114

(B)  110

(C)  50

(D)  98

(E)  142

Ans: (B)

32. 3    4        9        28      113    ?

(A)  782

(B)  424

(C)  646

(D)  384

(E)  566

Ans: (E)

33. 8    4        6        15      ?        25

(A)  64.5

(B)  84

(C)  52.5

(D)  36

(E)  46

Ans: (C)

34. The respective ratio of curved surface area and total surface area of a cylinder is 4 : 5. If the curved surface area of the cylinder is 1232 cm2, what is the height ? (in cm)

(A)  28

(B)  12

(C)  20

(D)  18

(E)  19

Ans: (A)

35. A bag contains 3 red balls, 5 yellow balls and 7 pink balls. If one ball is drawn at random from the bag, what is the probability that it is other pink or red ?

(A) 

(B) 

(C) 

(D)    

(E) 

Ans: (B)

IBPS PO Exam Results

IBPS PO 2015 Main exam results are to be declared soon:

          The Institute of Banking Personnel Selection (IBPS) was established in the year 1975 as the Personal Selection Services (PSS) whose recruitment takes place either two tier or three tier process.

IBPS RRB and SO recruitment is a two tier process whereas IBPS PO & Clerk consists of three stages such as Common Written Examination (CWE) followed by an interview and final placement.

About Institute of Banking Personnel Selection:

              The Institute of Banking Personnel Selection is an autonomous body which was set up to implement world class assessment and selection of personnel for various client organizations. It provides its services to many public sector banks like SBI, RBI, NABARD, SIDBI, few co-operative banks, public sector undertakings of non-financial sectors, state owned companies, Government departments and so on.

IBPS exam has become one of the famous tests in the country for being a successful “Personnel Selection Test Conducting Agency”. Its strength and ability lies in conducting multiple choices of objective tests for Lakhs and Lakhs of students across 200 test centres in India and also in few foreign locations.

Also, few major Universities and reputed management institutes engage IBPS regularly for conducting their Admission tests.

 

About IBPSPO exam:

                 Institute of Banking Personnel Selection has conducted preliminary examination on October 3rd, 4th, 10th and 11th, 2015 for the selection of Probationary officer (PO) or Management Trainee (MT) positions.

This was an online computer based test and consisted of multiple choices of questions. The duration of preliminary exam was one hour and total number of questions in the question paper was 100.

The results of the preliminary examination were declared on October 17th 2015.

The results of preliminary examination only serve to shortlist the candidates for the final IBPS PO main examination.

Following this preliminary exam, IBPS PO main exam was conducted on October 31st, 2015. The results of the main exam are expected to be declared soon at the official website of IBPS. Thus, students who wish to check their results please be ready with required details and then click on “IBPS PO/MT Results” which will direct you to the respective page showing your results. Candidates please take a print out of this page for future reference.

The examination pattern for IBPS PO 2015 main comprised of 200 objective types of questions based on Reasoning, Quantitative aptitude, and English language, General awareness with special reference to the banking industry and computer knowledge. Candidates will be finally shortlisted on the basis of the performance in preliminary exam, main exam and personal interview.

The official site of IBPS is http://www.ibps.in

 

© Copyright Entrance India - Engineering and Medical Entrance Exams in India | Website Maintained by Firewall Firm - IT Monteur